natural resources set2.classification cases

93
1 NATURAL RESOURCES: CLASSIFICATIONS CASES G.R. No. L-46729 November 19, 1982 LAUSAN AYOG vs. JUDGE VICENTE N. CUSI, JR 1 AQUINO, J.: This case is about the application of section 11, Article XIV of the 1973 Constitution (disqualifying a private corporation from purchasing public lands) to a 1953 sales award made by the Bureau of Lands, for which a sales patent and Torrens title were issued in 1975 , and to the 1964 decision of the trial court, ejecting some of the petitioners from the land purchased, which decision was affirmed in 1975 by the Court of Appeals. That legal question arises under the following facts: On January 21, 1953, the Director of Lands, after a bidding, awarded to Biñan Development Co., Inc. on the basis of its 1951 Sales Application No. V-6834 Cadastral Lot No. 281 located at Barrio Tamugan, Guianga (Baguio District), Davao City with an area of about two hundred fifty hectares. Some occupants of the lot protested against the sale. The Director of Lands in his decision of August 30, 1957 dismissed the protests and ordered the occupants to vacate the lot and remove their improvements. No appeal was made from that decision. The Director found that the protestants (defendants in the 1961 ejectment suit, some of whom are now petitioners herein) entered the land only after it was awarded to the corporation and, therefore, they could not be regarded as bona fide occupants thereof. The Director characterized them as squatters. He found that some claimants were fictitious persons (p. 30, Rollo of L- 43505, Okay vs. CA). He issued a writ of execution but the protestants defied the writ and refused to vacate the land (p. 28, Rollo of L-43505, Okay vs. CA). ** Because the alleged occupants refused to vacate the land, the corporation filed against them on February 27, 1961 in the 1 Classsifications Court of First Instance of Davao, Civil Case No. 3711, an ejectment suit ( accion publiciana ). The forty defendants were Identified as follows: 1. Vicente Abaqueta 21. Eniego Garlic 2. Candido Abella 22. Nicolas Garlic 3. Julio Ayog 23. Rufo Garlic 4. Arcadio Ayong 24. Alfonso Ibales 5. Generoso Bangonan 25. Julian Locacia 6. Lomayong Cabao 26. Filomeno Labantaban 7. Jose Catibring 27. Arcadio Lumantas 8. Teodolfo Chua 28. Santos Militante 9. Guillermo Dagoy 29. Toribio Naquila 10. Anastacia Vda. de Didal 30. Elpidio Okay 11. Alfredo Divinagracia 31. Guillermo Omac 12. Silverio Divinagracia 32. Emilio Padayday 13. Galina Edsa 33. Marcosa Vda. de Rejoy 14. Jesus Emperado 34. Lorenzo Rutsa 15. Porfirio Enoc 35. Ramon Samsa 16. Benito Ente 36. Rebecca Samsa 17. German Flores 37. Alfeao Sante 18. Ciriaco Fuentes 38. Meliton Sante 19. Pulong Gabao 39. Amil Sidaani 20. Constancio Garlic 40. Cosme Villegas That ejectment suit delayed the issuance of the patent. The trial court found that the protests of twenty of the abovenamed defendants were among those that were dismissed by the Director of Lands in his 1957 decision already mentioned. On July 18, 1961 the purchase price of ten thousand pesos was fully paid by Binan Development Co., Inc. On November 10, 1961 , an official of the Bureau of Lands submitted a final investigation report wherein it was stated that the corporation had complied with the cultivation and other requirements under the Public Land Law and had paid the purchase price of the land (p. 248, Rollo).

Upload: kay-frin-chez-tautho

Post on 27-Nov-2015

12 views

Category:

Documents


2 download

DESCRIPTION

wwwgggg

TRANSCRIPT

Page 1: Natural Resources Set2.Classification Cases

1NATURAL RESOURCES: CLASSIFICATIONS CASES

G.R. No. L-46729 November 19, 1982 LAUSAN AYOG vs. JUDGE VICENTE N. CUSI, JR1

AQUINO, J.:

This case is about the application of section 11, Article XIV of the 1973 Constitution (disqualifying a private corporation from purchasing public lands) to a 1953 sales award made by the Bureau of Lands, for which a sales patent and Torrens title were issued in 1975, and to the 1964 decision of the trial court, ejecting some of the petitioners from the land purchased, which decision was affirmed in 1975 by the Court of Appeals. That legal question arises under the following facts:

On January 21, 1953, the Director of Lands, after a bidding, awarded to Biñan Development Co., Inc. on the basis of its 1951 Sales Application No. V-6834 Cadastral Lot No. 281 located at Barrio Tamugan, Guianga (Baguio District), Davao City with an area of about two hundred fifty hectares. Some occupants of the lot protested against the sale. The Director of Lands in his decision of August 30, 1957 dismissed the protests and ordered the occupants to vacate the lot and remove their improvements. No appeal was made from that decision.

The Director found that the protestants (defendants in the 1961 ejectment suit, some of whom are now petitioners herein) entered the land only after it was awarded to the corporation and, therefore, they could not be regarded as bona fide occupants thereof. The Director characterized them as squatters. He found that some claimants were fictitious persons (p. 30, Rollo of L-43505, Okay vs. CA). He issued a writ of execution but the protestants defied the writ and refused to vacate the land (p. 28, Rollo of L-43505, Okay vs. CA). **

Because the alleged occupants refused to vacate the land, the corporation filed against them on February 27, 1961 in the Court of First Instance of Davao, Civil Case No. 3711, an ejectment suit (accion publiciana). The forty defendants were Identified as follows:

1. Vicente Abaqueta 21. Eniego Garlic2. Candido Abella 22. Nicolas Garlic3. Julio Ayog 23. Rufo Garlic4. Arcadio Ayong 24. Alfonso Ibales5. Generoso Bangonan 25. Julian Locacia6. Lomayong Cabao 26. Filomeno Labantaban7. Jose Catibring 27. Arcadio Lumantas8. Teodolfo Chua 28. Santos Militante9. Guillermo Dagoy 29. Toribio Naquila

1 Classsifications

10. Anastacia Vda. de Didal 30. Elpidio Okay11. Alfredo Divinagracia 31. Guillermo Omac12. Silverio Divinagracia 32. Emilio Padayday13. Galina Edsa 33. Marcosa Vda. de Rejoy14. Jesus Emperado 34. Lorenzo Rutsa15. Porfirio Enoc 35. Ramon Samsa16. Benito Ente 36. Rebecca Samsa17. German Flores 37. Alfeao Sante18. Ciriaco Fuentes 38. Meliton Sante19. Pulong Gabao 39. Amil Sidaani20. Constancio Garlic 40. Cosme Villegas

That ejectment suit delayed the issuance of the patent. The trial court found that the protests of twenty of the abovenamed defendants were among those that were dismissed by the Director of Lands in his 1957 decision already mentioned.

On July 18, 1961 the purchase price of ten thousand pesos was fully paid by Binan Development Co., Inc. OnNovember 10, 1961, an official of the Bureau of Lands submitted a final investigation report wherein it was stated that the corporation had complied with the cultivation and other requirements under the Public Land Law and had paid the purchase price of the land (p. 248, Rollo).

It was only more than thirteen years later or on August 14, 1975 when Sales Patent No. 5681 was issued to the corporation for that lot with a reduced area of 175.3 hectares. The patent was registered. Original Certificate of Title No. P-5176 was issued to the patentee.

The Director of Lands in his memorandum dated June 29, 1974 for the Secretary of Natural Resources, recommending approval of the sales patent, pointed out that the purchaser corporation had complied with the said requirements long before the effectivity of the Constitution, that the land in question was free from claims and conflicts and that the issuance of the patent was in conformity with the guidelines prescribed in Opinion No. 64, series of 1973, of Secretary of Justice Vicente Abad Santos and was an exception to the prohibition in section 11, Article XIV of the Constitution (p. 258, Rollo).

Secretary of Natural Resources Jose J. Leido, Jr., in approving the patent on August 14, 1975, noted that the applicant had acquired a nested right to its issuance (p. 259, Rollo).

Before that patent was issued, there was a trial in the ejectment suit. Fifteen defendants (out of forty), namely, Julio Ayog, Guillermo Bagoy, Generoso Bangonan, Jose Catibring, Porfirio Enoc, Jose Emperado, Arcadio Lomanto, Toribio Naquila,

Page 2: Natural Resources Set2.Classification Cases

2NATURAL RESOURCES: CLASSIFICATIONS CASES

Elpidio Okay, Alfeo Sante, Meliton Sante, Ramon Samsa, Rebecca Samsa, Arcadio Sarumines and Felix Tahantahan, testified that they entered the disputed land long before 1951 and that they planted it to coconuts, coffee, jackfruit and other fruit trees. (p. 28, Record on Appeal).

The trial court did not give credence to their testimonies. It believed the report of an official of the Bureau of Lands that in 1953 the land was free from private claims and conflicts and it gave much weight to the decision of the Director of Lands dismissing the protests of the defendants against the sales award (p. 30, Record on Appeal).

Furthermore, the trial court during its ocular inspection of the land on November 8, 1964 found that the plantings on the land could not be more than ten years old, meaning that they were not existing in 1953 when the sales award was made. Hence, the trial court ordered the defendants to vacate the land and to restore the possession thereof to tile company. The Court of Appeals affirmed that judgment on December 5, 1975 in its decision in Binan Development Co., Inc. vs, Sante, CA-G.R. No. 37142- R. The review of the decision was denied by this Court on May 17, 1976 in Elpidio Okay vs. Court of Appeals, L-43505.

After the record was remanded to the trial court, the corporation filed a motion for execution. The defendants, some of whom are now petitioners herein, opposed the motion. They contended that the adoption of the Constitution, which took effect on January 17, 1973, was a supervening fact which rendered it legally impossible to execute the lower court's judgment. They invoked the constitutional prohibition, already mentioned, that "no private corporation or association may hold alienable lands of the public domain except by lease not to exceed one thousand hectares in area."

The lower court suspended action on the motion for execution because of the manifestation of the defendants that they would file a petition for prohibition in this Court. On August 24, 1977, the instant prohibition action was filed. Some of the petitioners were not defendants in the ejectment case.

We hold that the said constitutional prohibition has no retroactive application to the sales application of Biñan Development Co., Inc. because it had already acquired a vested right to the land applied for at the time the 1973 Constitution took effect.

That vested right has to be respected. lt could not be abrogated by the new Constitution. Section 2, Article XIII of the 1935 Constitution allows private corporations to purchase public agricultural lands not exceeding one thousand and twenty-four hectares. Petitioners' prohibition action is barred by the doctrine of vested rights in constitutional law.

"A right is vested when the right to enjoyment has become the property of some particular person or persons as a present interest" (16 C.J.S. 1173). It is "the privilege to enjoy property legally vested, to enforce contracts, and enjoy the rights of property conferred by the existing law" (12 C.J. 955, Note 46, No. 6) or "some right or interest in property which has become fixed and established and is no longer open to doubt or controversy" (Downs vs. Blount 170 Fed. 15, 20, cited in Balboa vs. Farrales, 51 Phil. 498, 502).

The due process clause prohibits the annihilation of vested rights. "A state may not impair vested rights by legislative enactment, by the enactment or by the subsequent repeal of a municipal ordinance, or by a change in the constitution of the State, except in a legitimate exercise of the police power" (16 C.J.S. 1177-78).

It has been observed that, generally, the term "vested right" expresses the concept of present fixed interest, which in right reason and natural justice should be protected against arbitrary State action, or an innately just and imperative right which an enlightened free society, sensitive to inherent and irrefragable individual rights, cannot deny (16 C.J.S. 1174, Note 71, No. 5, citing Pennsylvania Greyhound Lines, Inc. vs. Rosenthal, 192 Atl. 2nd 587).

Secretary of Justice Abad Santos in his 1973 opinion ruled that where the applicant, before the Constitution took effect, had fully complied with all his obligations under the Public Land Act in order to entitle him to a sales patent, there would seem to be no legal or equitable justification for refusing to issue or release the sales patent (p. 254, Rollo).

In Opinion No. 140, series of 1974, he held that as soon as the applicant had fulfilled the construction or cultivation requirements and has fully paid the purchase price, he should be deemed to have acquired by purchase the particular tract of land and to him the area limitation in the new Constitution would not apply.

In Opinion No. 185, series of 1976, Secretary Abad Santos held that where the cultivation requirements were fulfilled before the new Constitution took effect but the full payment of the price was completed after January 17, 1973, the applicant was, nevertheless, entitled to a sales patent (p. 256, Rollo).

Such a contemporaneous construction of the constitutional prohibition by a high executive official carries great weight and should be accorded much respect. It is a correct interpretation of section 11 of Article XIV.

In the instant case, it is incontestable that prior to the effectivity of the 1973 Constitution the right of the corporation to purchase the land in question had become fixed and established and was no longer open to doubt or controversy.

Page 3: Natural Resources Set2.Classification Cases

3NATURAL RESOURCES: CLASSIFICATIONS CASES

Its compliance with the requirements of the Public Land Law for the issuance of a patent had the effect of segregating the said land from the public domain. The corporation's right to obtain a patent for that land is protected by law. It cannot be deprived of that right without due process (Director of Lands vs. CA, 123 Phil. 919).

As we cannot review the factual findings of the trial court and the Court of Appeals, we cannot entertain petitioners' contention that many of them by themselves and through their predecessors-in-interest have possessed portions of land even before the war. They should have filed homestead or free patent applications.

Our jurisdiction is limited to the resolution of the legal issue as to whether the 1973 Constitution is an obstacle to the implementation of the trial court's 1964 final and executory judgment ejecting the petitioners. On that issue, we have no choice but to sustain its enforceability.

Nevertheless, in the interest of social justice, to avoid agrarian unrest and to dispel the notion that the law grinds the faces of the poor, the administrative authorities should find ways and means of accommodating some of the petitioners if  they are landless and are really tillers of the soil who in the words of President Magsaysay deserve a little more food in their stomachs, a little more shelter over their heads and a little more clothing on their backs. The State should endeavor to help the poor who find it difficult to make both ends meet and who suffer privations in the universal struggle for existence.

A tiller of the soil is entitled to enjoy basic human rights, particularly freedom from want. The common man should be assisted in possessing and cultivating a piece of land for his sustenance, to give him social security and to enable him to achieve a dignified existence and become an independent, self-reliant and responsible citizen in our democratic society.

To guarantee him that right is to discourage him from becoming a subversive or from rebelling against a social order where, as the architect of the French Revolution observed, the rich are choking with the superfluities of life but the famished multitude lack the barest necessities.

Indeed, one purpose of the constitutional prohibition against purchases of public agricultural lands by private corporations is to equitably diffuse land ownership or to encourage "owner-cultivatorship and the economic family- size farm" and to prevent a recurrence of cases like the instant case. Huge landholdings by corporations or private persons had owned social unrest.

Petitioners' counsel claims that Biñan Development Co., Inc. seeks to execute the judgment in Civil Case No. 3711, the ejectment suit from which this prohibition case

arose, against some of the petitioners who were not defendants in that suit (p. 126, Rollo).

Those petitioners are not successors-in-interest of the defendants in the ejectment suit. Nor do they derive their right of possession from the said defendants. Those petitioners occupy portions of the disputed land distinct and separate from the portions occupied by the said defendants.

We hold that judgment cannot be enforced against the said petitioners who were not defendants in that litigation or who were not summoned and heard in that case. Generally, "it is an axiom of the law that no man shall be affected by proceedings to which he is a stranger" (Ed. A. Keller & Co. vs Ellerman & Bucknall Steamship Co., 38 Phil. 514, 520).

To enforce the judgment against those who were not parties to the case and who occupy portions of the disputed land distinct and separate from the portions occupied by the defendants in the ejectment suit, would be violative of due process of law, the law which, according to Daniel Webster in his argument in the Dartmouth College case, is the law of the land, a law which hears before it condemns, which proceeds upon inquiry and renders judgment only after trial. "The meaning is, that every citizen shall hold his life, liberty, property, and immunities, under the protection of the general rules which govern society." (Cited in Lopez vs. Director of Lands, 47 Phil. 23, 32. See Gatchalian vs. Arlegui, L-35615 and Tang Tee vs. Arlegui, L-41360, February 17, 1977, 75 SCRA 234 and Berses vs. Villanueva, 25 Phil. 473.)

Contempt incident.-During the pendency of this case, or at about four o'clock in the morning of December 12, 1978, Ciriaco Tebayan, Domingo Nevasca, Rogelio Duterte and Sofronio Etac, employees of the Crown Fruits and Cannery Corporation, plowed or bulldozed with their tractors a portion of the disputed land which was occupied by Melquiades Emberador, one of the petitioners herein. The disputed land was leased by Biñan Development Co., Inc. to the canning corporation.

The four tractor drivers destroyed the improvements thereon worth about five thousand pesos consisting of coffee, coconut and banana plants. Emberador was in the hospital at the time the alleged destruction of the improvements occurred. However, it should be noted that Emberador was not expressly named as a defendant in the ejectment suit. Apparently, he is not included in the trial court's decision although he was joined as a co-petitioner in this prohibition case.

The petitioners in their motion of January 11, 1979 asked that the four tractor drivers and Honesto Garcia, the manager of Biñan Development Co., Inc., be declared in contempt of court for having disregarded the restraining order issued by this Court on August 29, 1977, enjoining specifically Judge Vicente N. Cusi and the provincial

Page 4: Natural Resources Set2.Classification Cases

4NATURAL RESOURCES: CLASSIFICATIONS CASES

sheriff from enforcing the decision in the ejectment suit, Civil Case No. 3711 (pp. 46-47, 138- 141, Rollo).

Garcia and the four drivers answered the motion. The incident was assigned for hearing to Judge Antonio M. Martinez of the Court of First Instance of Davao. Judge Martinez found that the plowing was made at the instance of Garcia who told the barrio captain, petitioner Lausan Ayog, a Bagobo, that he (Garcia) could not wait anymore for the termination of this case.

The record shows that on April 30, 1979 or four months after the said incident, Emberador, in consideration of P3,500, as the value of the improvements on his land, executed a quitclaim in favor of the Crown Fruits and Cannery Corporation (Exh. 1, 2 and 3).

We hold that no contempt was committed. The temporary restraining order was not directed to Biñan Development Co., Inc. its officers, agents or privies. Emberador was not named specifically in the trial court's judgment as one of the occupants to be ejected.

For the redress of whatever wrong or delict was committed against Emberador by reason of the destruction of his improvements, his remedy is not in a contempt proceeding but in some appropriate civil and criminal actions against the destroyer of the improvements.

In resume, we find that there is no merit in the instant prohibition action. The constitutional prohibition relied upon by the petitioners as a ground to stop the execution of the judgment in the ejectment suit has no retroactive application to that case and does not divest the trial court of jurisdiction to enforce that judgment.

WHEREFORE, the petition is dismissed for lack of merit but with the clarification that the said judgment cannot be enforced against those petitioners herein who were not defendants in the ejectment case, Civil Case No. 3711, and over whom the lower court did not acquire jurisdiction. The contempt proceeding is also dismissed. No costs.

SO ORDERED.

REPUBLIC OF THE PHILIPPINES vs. HONORABLE COURT OF APPEALS and IGLESIA NI CRISTO2

2 Classifications

G.R. No. L-60188 December 27, 1982

PLANA, J.:

These are two land registration cases instituted by the Iglesia Ni Cristo (INC), a corporation sole, under Section 48 (b) of the Public Land Act which reads:

Sec 48. The following described citizens of the Philippines occupying lands of the public domain or claiming to own any such lands or an interest therein, but whose titles have not been perfected or completed, may apply to the Court of First Instance of the province where the land is located for confirmation of their claims and the issuance of a certificate of title therefor, under the Land Registration Act, to wit . . .xxx xxx xxx(b) Those who by themselves or through their predecessors-in-interest have been in open, continuous, exclusive and notorious possession and occupation of agricultural lands of the public domain, under a bona fide claim of acquisition or ownership, for at least thirty years immediately preceding the filing of the application for confirmation of title except when prevented by war or force majeure. These shall be conclusively presumed to have performed all the conditions essential to a Government grant and shall be entitled to a certificate of title under the provisions of this chapter. ...

In both cases, the applications for land registration were granted by the lower courts, impelling the Republic to elevate the cases to this forum on this basic legal issue: whether the lands applied for may be registered in the name of INC. in the light of the Constitutional provision that "no private corporation or association may hold alienable lands of the public domain except by lease." (Article XIV, Section 11). An ancillary question refers to the nature of the lands subject to registration proceedings — whether they are private or public.

The issues are not new. In at least two decisions, (Meralco vs. Judge Castro Bartolome et al 114 SCRA 799; Republic vs. Judge Candido P. Villanueva, 114 SCRA 875), this court has ruled on the questions. Specifically, in Republic vs. Judge Villanueva, this court, speaking thru Justice Ramon C. Aquino, said:

As correctly contended by the Solicitor General, the Iglesia Ni Cristo, as a corporation sole or a juridical person, is disqualified to acquire or hold alienable lands of the public domain, like the two lots in question, because of the constitutional prohibition already mentioned and because the said church is not entitled to avail itself of the benefits of section 48 (b) which applies only to Filipino citizens or natural persons. A corporation sole (an "unhappy freak of English law") has no nationality (Roman Catholic Apostolic Adm. of Davao, Inc. vs. Land

Page 5: Natural Resources Set2.Classification Cases

5NATURAL RESOURCES: CLASSIFICATIONS CASES

Registration Commission, 102 Phil. 596. See Register of Deeds vs. Ung Siu Si Temple 97 Phil. 58 and sec. 49 of the Public Land Law).

The contention in the comments of the Iglesia Ni Cristo (its lawyer did not file any brief) that the two lots are private lands, following the rule laid down in Susi vs. Razon and Director of Lands, 48 Phil. 424, is not correct. What was considered private land in the Susi case was a parcel of land possessed by a Filipino citizen since time immemorial, as in Carifio vs. Insular Government, 212 U.S. 449, 531 L. ed. 594, 41 Phil. 935 and 7 Phil. 132. The lots sought to be registered in this case do not fall within that category. They are still public lands. A land registration proceeding under section 48 (b) "presupposes that the land is public" (Mindanao vs. Director of Lands, L-19535, July 10, 1967, 20 SCRA 641, 644).

As held in Oh Cho vs. Director of Lands, 75 Phil. 890, "all lands that were not acquired from the Government, either by purchase or by grant, belong to the public domain. An exception to the rule would be any land that should have been in the possession of an occupant and of his predecessors-in-interest since time immemorial, for such possession would justify the presumption that the land had never been part of the public domain or that it had been a private property even before the Spanish conquest. "

In Uy Un vs. Perez, 71 Phil. 508, it was noted that the right of an occupant of public agricultural land to obtain a confirmation of his title under section 48 (b) of the Public Land Law is a "derecho dominical incoativo" and that before the issuance of the certificate of title the occupant is not in the juridical sense the true owner of the land since it still pertains to the State. (114 SCRA 881-882)

Following the above cases, this court sustains the stand of the Republic, without need to inquire into the veracity of the allegation in G.R. L-59447 that the evidence presented therein do not prove INC. continuity of possession of the land for the period prescribed by law. The appealed decisions of the lower courts are hereby set aside and the INC. applications for registration are hereby dismissed. No costs.

SO ORDERED.

G.R. No. L-49623 June 29, 1982 

MANILA ELECTRIC COMPANY vs. JUDGE FLORENLIANA CASTRO-BARTOLOME3

AQUINO, J.:p

This case involves the prohibition in section 11, Article XIV of the Constitution that "no private coporation or associaiton may hold alienable lands of the

3 Classifications

public domain except by lease not to exceed on ethousand hectares in area". * That prohibition is not found in the 1935 Constitution.

The Manila Electric Company, a domestic corporation organized under Philippine laws, more than sixty percent of whose capital stock is owned by Filipino citizens, in its application filed on December 1, 1976 in the Makati branch of the Court of First Instance of Rizal, prayed for the confirmation of its title to two lots with a total area of one hundred sixty-five square meters, located at Tanay, Rizal with an assessed value of P3,270 (LRC Case No. N-9485, LRC No. N-50801).

The Republic of the Philippines opposed theh application on the grounds that the applicant, as a private corporation,is disqualified to hold alienable public lands and that the applicant and its prredecessors-in-interest have not been in the open, continuous, exclusive and notorious possession and occupation of the land for at least thirty years immediately preceding the filing of the application (pp. 65-66, Rollo).

After the trial had commenced, the Province of rizal and the Municipality of Tanay filed a joint opposition to the application on the ground that one of the lots, Lot No. 1165 of the Tanay cadastre, would be needed for the widening and improvement of Jose Abad Santos and E.Quirino Streetsin the town of Tanay.

The land was possessed by Olimpia ramos before the Pacific war which broke out in 1941. On July 3, 1947, Ramos sold the land to the spouses Rafael Piguing and MInerva Inocencio (Exh. K). The Piguing sapouses constructed a house therereon. Because the Meralco had installed the "anchor guy" of its steel post on the land, the Piguing spouses sold the lot to the Meralco on August 13, 1976.

The said land was included in the1968 cadastral survey made in Tanacy by the Bureau of Lands, Plan AP-04-000902 (Exh. F and H) and was divided into two lots, Lots Nos. 1164 and 1165, so as to segregate Lot No. 1165 which would be used to widen the two street serving as the land's eastern and southern boundaries.

The land was declared for realty tax purposes since 1945 and taxes had been paid thereon up to 1977. It is residential in character as distinguished from a strictly agricultural land. It is not included in any military reservation. Since 1927, it has formed part of the alienable portion of the public domain.

After trial, the lowre court rendered a decision dismissing the application because in its opinion the Meralco is not qualified to apply for the registration of the said land since under section 48(b) of the Public Land Law only Filipino citizens or natural persons can apply for judicial confirmationof their imperfect titles to public

Page 6: Natural Resources Set2.Classification Cases

6NATURAL RESOURCES: CLASSIFICATIONS CASES

land. The Meralco is a juridical person. The trial court assumed that the land which it seeks to register is public land.

From that decision, the Meralco appealed to this Court under Republic Act No. 5440.

In contends that the said land, after having been possessed in the concept of owner by Olimpia Ramos and the Piguing spouses for more than thirty years, had become private land in the hands of the latter, and, therefore, the constitutional prohibition, banning a private corporation from acquiring alienable public land, is not applicable to the said land.

The Meralco further contends that it has invoke section 48(b) of the Public Land Law, not for itself, but for the Piguing spouses who, as Filipino citizens, could secure a judicial confirmation of their imperfect title to the land.

In reply to these contentions, the Solicitor General counters that the said land is not private land because the Meralco and its predecessors-in-interest have no composition title from the Spanish government nor possessory information title or any other means for the acquisition of public lands such as grants or patents (Republic vs. Court of Appeals and De Jesus, L-40912, September 30, 1976, 73 SCRA 146, 157; Director of Lands vs. Reyes, L-27594, November 28, 1975, and Alinsunurin vs. Director of Lands, L-28144, November 28, 1975; 68 SCRA 177; 195; Lee Hong Hok vs. David, L-30389, December 27, 1972, 48 SCRA 372, 378-9; Director of Lands vs. Court of Appeals and Raymundo, L-29575, April 30, 1971, 38 SCRA 634, 639; Padilla vs. Reyes and Director of Lands, 60 Phil. 967, 969; Heirs of Datu Pendatun vs. Director of Lands, 59 Phil. 600, 603).

The Public Land Law provides:

CHAPTER VIII. — Judicial confirmation of imperfect or incomplete titles.xxx xxx xxxSEC. 48. The following described citizens of the Philippines, occupying lands of the public domain or claiming to own any such lands or an interest therein, but whose titles have not been perfected or completed, may apply to the Court of First Instance of the province where the land is located for confirmation of their claims and the issuance of a certificate of title therefor, under the Land Registration Act, to wit:xxx xxx xxx(b) Those who by themselves or through their predecessors in interest have been in open, continuous, exclusive, and notorious possession and occupation of agricultural lands of the public domain, under a bona fide claim of acquisition of ownership, for at least thirty years immediately preceding the filing of the

application for confirmation of title except when prevented by war or force majeure. These shall be conclusively presumed to have performed all the conditions essential to a Government grant and shall be entitled to a certificate of title under the provisions of this chapter. (As amended by Republic Act No. 1942, approved on June 22, 1957.)xxx xxx xxxSEC. 49. No person claiming title to lands of the public domain not in possession of the qualifications specified in the last preceding section may apply for the benefits of this chapter.

We hold that, as between the State and the Meralco, the said land is still public land. It would cease to be public land only upon the issuance of the certificate of title to any Filipino citizen claiming it under section 48(b). Because it is still public land and the Meralco, as a juridical person, is disqualified to apply for its registration under section 48(b), Meralco's application cannot be given due course or has to be dismissed.

This conclusion is supported by the rule announced in Oh Cho vs. Director of Lands, 75 Phil. 890, 892, which rule is a compendious or quintessential precis of a pervasive principle of public land law and land registration law, that "all lands that were not acquired from the Government, either by purchase or by grant, belong to the public domain. An exception to the rule would be any land that should have been in the possession of an occupant and of his predecessors-in-interest since time immemorial, for such possession would justify the presumption that the land had never been part of the public domain or that it had been a private property even before the Spanish conquest." (Cariño vs. Insular Government, 212 U. S. 449, 53 L. ed. 594, 41 Phil. 935 and 7 Phil. 132).

The Meralco relies on the ruling in Susi vs. Razon and Director of Lands, 48 Phil. 424, that "an open, continuous, adverse and public possession of a land of the public domain from time immemorial by a private individual personally and through his predecessors confers an effective title on said possessor, whereby the land ceases to be public" and becomes private property.

That ruling is based on the Cariño case which is about the possession of land by an Igorot and his ancestors since time immemorial or even before the Spanish conquest. The land involved in the Susi case was possessed before 1880 or since a period of time "beyond the reach of memory". That is not the situation in this case. The Meralco does not pretend that the Piguing spouses and their predecessor had been in possession of the land since time immemorial.

In the Susi case, this Court applied section 45(b) of Act No. 2874 which corresponds to what is now section 48(b). It was held that the long possession of the land under a bona fide claim of ownership since July 26, 1894 gave rise to the

Page 7: Natural Resources Set2.Classification Cases

7NATURAL RESOURCES: CLASSIFICATIONS CASES

conclusive presumption that the occupant had complied with all the conditions essential to a Government grant and was thus entitled to a certificate of title.

On the other hand, in Uy Un vs. Perez, 71 Phil. 508, 510-11, it was held that while occupants of public land, who have applied for the confirmation of their title, "teian asimismo a su favor la presuncion juris et de jure de que habian cumplido con todas las condiciones necesarias para la concesion del titulo; pero hasta que el titulo se expida no tenian el concepto juridico de ser los verdaderos dueños del terreno in este dejo de pertenecer a los terrenos publico del Estado susceptibles de enajenacion."

That means that until the certificate of title is issued, a pice of land, over which an imperfect title is sought to be confirmed, remains public land. For that reason in the Uy Un case, it was held that if that land was attached by a judgment creditor of the applicant, while his application for confirmation of his imperfect title was pending in the Bureau of Lands, the levy and execution sald of the land were void.

For that same reason, lands over which an imperfect title is sought to be confirmed are governed by the Public Land Law. Such lands would not be covered by the Public Land Law if they were already private lands. The occupants' right to the said lands is characterized in the Uy Un case, not as ownership in fee simple, but asderecho dominical incoativo.

The Meralco in its concluding argument contends that if the Piguing spouses could ask for the confirmation of their imperfect title to the said lands, then why should the Meralco, as their transferee, be denied the same right to register the said land in its name, there being no legal prohibition for the Piguing spouses from selling the land to the Meralco? This Court is disposing of that same contention in the Oh Cho case said:

The benefits provided in the Public Land Act (meaning the confirmation of an imperfect title under section 48[b]) for applicant's immediate predecessors-in-interest are or constitute a grant or concession by the State; and before they could acquire any right under such benefits, the applicant's immediate predecessors-in-interest should comply with the condition precedent for the grant of such benefits.

The condition precedent is to apply for the registration of the land of which they had been in possession at least since July 26, 1894. This the applicant's immediate predecessors-in-interest (meaning the Piguing spouses in the instant case) failed to do.

They did not have any vested right in the lot amounting to title which was transmissible to the applicant. The only right, if it may thus be called, is their possession of the lot which, tacked to that of their predecessors-in-interest, may

be availed of by a qualified person to apply for its registration but not by a person as the applicant who is disqualified. (75 Phil. 890, 893.)

Finally, it may be observed that the constitutional prohibition makes no distinction between (on one hand) alienable agricultural public lands as to which no occupant has an imperfect title and (on the other hand) alienable lands of the public domain as to which an occupant has an imperfect title subject to judicial confirmation.

Since section 11 of Article XIV does not distinguish, we should not make any distinction or qualification. The prohibition applies to alienable public lands as to which a Torrens title may be secured under section 48(b). The proceeding under section 48(b) "presupposes that the land is public" (Mindanao vs. Director of Lands, L-19535, July 30, 1967, 20 SCRA 641, 644).

The lower court;s judgment dismissing Meralco's application is affirmed. Costs against the petitioner-appellant. SO ORDERED.

G.R. No. 73246 March 2, 1993

DIRECTOR OF LANDS AND DIRECTOR OF FOREST DEVELOPMENT vs. INTERMEDIATE APPELLATE COURT 4

NOCON, J.:

For review before Us is the decision of the Court of Appeals in the land registration case entitled J. Antonio Araneta v. The Director of Lands and Director of Forest Development, AC-G.R. CV. No. 00636,  affirming the lower court's approval of the application for registration of a parcel of land in favor of applicant therein, J. Antonio Araneta.

Evidence show that the land involved is actually an island known as Tambac Island in Lingayen Gulf. Situated in the Municipality of Bani, Pangasinan, the area consists of 187,288 square meters, more or less. The initial application for registration was filed for Pacific Farms, Inc. under the provisions of the Land Registration Act, Act No. 496, as amended.

The Republic of the Philippines, thru the Director of Lands opposed the application alleging that the applicant, Pacific Farms, Inc. does not possess a fee simple title to the land nor did its predecessors possess the land for at least thirty (30) years immediately preceding the filing of application. The opposition likewise specifically alleged that the applicant is a private corporation disqualified under the

4 Classifications

Page 8: Natural Resources Set2.Classification Cases

8NATURAL RESOURCES: CLASSIFICATIONS CASES

(1973) new Philippine Constitution from acquiring alienable lands of the public domain citing Section 11, Article 14. 

The Director of Forest Development also entered its opposition alleging that the land is within the unclassified public land and, hence, inalienable. Other private parties also filed their oppositions, but were subsequently withdrawn.

In an amended application, Pacific Farms, Inc. filed a manifestation-motion to change the applicant from Pacific Farms, Inc. to J. Antonio Araneta. Despite the supposed amendment, there was no republication.

Evidence presented by the applicant include the testimony of Placido Orlando, fishery guard of Pacific Farms, Inc., who said he has known the disputed land since he attained the age of reason for some forty (40) years now; that when he first came to know the property it was then owned by and in the possession of Paulino Castelo, Juan Ambrosio and Julio Castelo, and later on the whole island was bought by Atty. Vicente Castelo who in turn sold it to J. Antonio Araneta.

Deposition by oral examination of Araneta was also presented, together with documents of sale, tax declarations and receipts, and survey of property. Applicant, however, failed to present the tracing cloth plan and instead submitted to the court certified copies thereof.

While this case is pending here in Court, respondent filed an Omnibus Motion for Substitution of private respondent.  Apparently, Antonio Araneta had assigned his rights to and interest in Tambac Island to Amancio R. Garcia   who in turn assigned his rights and interest in the same property to Johnny A. Khonghun whose nationality was not alleged in the pleadings.

On October 4, 1979, the trial court rendered a decision adjudicating the subject property to J. Antonio Araneta. On appeal to the then Intermediate Appellate Court, the decision of the lower court was affirmed on December 12, 1985.

Petitioners raised the following errors:

I. The lower court erred in adjudicating the lands subject of registration to applicant-appellee despite his failure to present the original tracing cloth plan the submission of which is a statutory requirement of mandatory character.

II.II. The lower court erred in not denying registration in favor of J. Antonio Araneta since the amendment of the application was simply an attempt to avoid the application of the constitutional provision disqualifying a private corporation —

the Pacific Farms, Inc. in this case — from acquiring lands of public domain.

III. The lower court erred in not declaring the land known as the "Tambac Island" not subject of registration it being an island formed on the seas.

IV. The lower court erred in adjudicating the land to the applicant under the provisions of Presidential Decree No. 1529, otherwise known as the Property Registration Decree, despite absence of any specific invocation of this law in the original and amended application.

V. The lower court erred in not granting the government's motion for reconsideration at least to enable it to present proof of the status of the land as within the unclassified public forest, and hence beyond the court's jurisdiction to adjudicate as private property.

VI. The lower court erred in not declaring that the applicant has failed to overthrow the presumption that the land is a portion of the public domain belonging to the Republic of the Philippines.

From the foregoing it appears that the more important issues are: 1) whether the presentation of the tracing cloth plan is necessary; and 2) whether the land known as "Tambac Island" can be subject to registration.

By mere consideration of the first assignment of error, We can right away glean the merit of the petition.

Respondent claims that the tracing cloth plan is with the files of the Land Registration Commission, and the only evidence that can be presented to that fact is the request for the issuance of a certified copy thereof and the certified copy issued pursuant to the request.  Respondent further argues that failure of the petitioners to object to the presentation of the certified copy of the tracing cloth plan was the basis of the trial court's denial of petitioner's motion for reconsideration.

In a very recent decision of this Court, entitled The Director of Lands v. The Honorable Intermediate Appellate Court and Lino Anit,  We have ruled that the submission of the tracing cloth plan is a mandatory requirement for registration. Reiterating Our ruling in Director of Lands v. Reyes, 7 We asserted that failure to submit in evidence the original tracing cloth plan is fatal it being a statutory requirement of mandatory character.

It is of no import that petitioner failed to object to the presentation of the certified copy of the said plan. What is required is the original tracing cloth plan of the land applied for and objection to such requirement cannot be waived either expressly or impliedly.  This case is no different from the case of Director of Lands v. Reyes,

Page 9: Natural Resources Set2.Classification Cases

9NATURAL RESOURCES: CLASSIFICATIONS CASES

supra wherein We said that if the original tracing cloth plan was indeed with the Land Registration Commission, there is no reason why the applicant cannot easily retrieve the same and submit it in evidence, it being an essential requirement for registration.

As to the second assignment of error, We are inclined to agree with petitioners that the amendment of the application from the name of Pacific Farms Inc., as applicant, to the name of J. Antonio Araneta Inc., was a mere attempt to evade disqualification. Our Constitution, whether the 1973   or1987, 10 prohibits private corporations or associations from holding alienable lands of the public domain except by lease. Apparently realizing such prohibition, respondent amended its application to conform with the mandates of the law.

However, We cannot go along with petitioners' position that the absence of republication of an amended application for registration is a jurisdictional flaw. We should distinguish. Amendments to the application may be due to change in parties or substantial change in the boundaries or increase in the area of the land applied for.

In the former case, neither the Land Registration Act, as amended, nor Presidential Decree No. 1529, otherwise known as the Property Registration Decree, requires republication and registration may be allowed by the court at any stage of the proceeding upon just and reasonable terms.  On the other hand, republication is required if the amendment is due to substantial change in the boundaries or increase in the area of the land applied for.

As to the fourth assignment of error. We do not see any relevant dispute in the lower court's application of Presidential Decree No. 1529, instead of Act No. 496, in adjudicating the land to the then applicant, assuming that the land involved is registrable. Both laws are existing and can stand together. P.D. 1529 was enacted to codify the various laws relative to registration of property, in order to facilitate effective implementation of said laws. 

The third, fifth and sixth assignment of errors are likewise meritorious and shall be discussed forthwith together.

Respondent asserts that contrary to the allegation of petitioners, the reports of the District Land Officer of Dagupan City, Land Inspector Perfecto Daroy and Supervising Land Examiner Teodoro P. Nieva show that the subject property is an unclassified public land, not forest land. This claim is rather misleading. The report of Supervising Land Examiner Nieva specifically states that the "land is within the unclassified forest land" under the administrative jurisdiction of the then Bureau of Forest Development.  This was based on the reports of Land Inspector Daroy and District Land Officer Feliciano Liggayu.

Lands of the public domain are classified under three main categories, namely: Mineral, Forest and Disposable or Alienable Lands.   Under the Commonwealth Constitution, only agricultural lands were allowed to be alienated. Their disposition was provided for under Commonwealth Act No. 141 (Secs. 6-7), which states that it is only the President, upon the recommendation of the proper department head, who has the authority to classify the lands of the public domain into alienable or disposable, timber and mineral lands. Mineral and Timber or forest lands are not subject to private ownership unless they are first reclassified as agricultural lands and so released for alienation.  In the absence of such classification, the land remains as unclassified land until released therefrom and rendered open to disposition. Courts have no authority to do so. 

This is in consonance with the Regalian doctrine that all lands of the public domain belong to the State, and that the State is the source of any asserted right to ownership in land and charged with the conservation of such patrimony. Under the Regalian Doctrine, all lands not otherwise appearing to be clearly within private ownership are presumed to belong to the State. Hence, a positive act of the government is needed to declassify a forest land into alienable or disposable land for agricultural or other purposes. 

The burden of proof in overcoming the presumption of state ownership of the lands of the public domain is on the person applying for registration that the land subject of the application is alienable or disposable. 

Unless the applicant succeeds in showing by convincing evidence that the property involved was acquired by him or his ancestors either by composition title from the Spanish Government or by possessory information title, or any other means for the proper acquisition of public lands, the property must be held to be part of the public domain. The applicant must present evidence and persuasive proof to substantiate his claim. 

In this particular case, respondent presented proof that as early as 1921, the subject property has been declared for tax purposes with receipts attached, in the names of respondent's predecessors-in-interest. Nevertheless, in that span of time there had been no attempt to register the same either under Act 496 or under the Spanish Mortgage Law. It is also rather intriguing that Vicente Castelo who acquired almost 90% of the property from Alejo Ambrosia, et al. on June 18, 1958 and from Julio Castelo on June 19, 1958 immediately sold the same to applicant J. Antonio Araneta on 3 July 1958.

According to the report of Land Investigator Daroy, the land was declared for taxation purposes in the name of Vicente Castelo only in 1958 and the purported old tax declarations are not on file with the Provincial Assessor's Office.

Page 10: Natural Resources Set2.Classification Cases

10NATURAL RESOURCES: CLASSIFICATIONS CASES

In any case tax declarations and receipts are not conclusive evidence of ownership or of the right to possess land when not supported by evidence.   The fact that the disputed property may have been declared for taxation purposes in the names of the applicants or of their predecessors-in-interest way back in 1921 does not necessarily prove ownership. They are merely indicia of a claim of ownership. 

Respondent's contention that the BFD, LC Map No. 681, certified on August 8, 1927 which was the basis of the report and recommendation of the Land Examiner, is too antiquated; that it cannot be conclusively relied upon and was not even presented in evidence, is not well taken. As We have said in the case of Director of Lands v. CA: 

And the fact that BF Map LC No. 673 dated March 1, 1927 showing subject property to be within unclassified region was not presented in evidence will not operate against the State considering the stipulation between the parties and under the well-settled rule that the State cannot be estopped by the omission, mistake or error of its officials or agents, if omission there was, in fact.

Respondent even admitted that Tambac Island is still an unclassified public land as of 1927 and remains to be unclassified.

Since the subject property is still unclassified, whatever possessionthe applicant may have had and however long, cannot ripen into private ownership.  The conversion of subject property does not automatically render the property as alienable and disposable.

In effect what the courts a quo have done is to release the subject property from the unclassified category, which is beyond their competence and jurisdiction. We reiterate that the classification of public lands is an exclusive prerogative of the Executive Department of the Government and not of the Courts. In the absence of such classification, the land remains unclassified until released therefrom and rendered open to disposition. 

In fairness to respondent, the petitioners should seriously consider the matter of the reclassification of the land in question. The attempt of people to have disposable lands they have been tilling for generations titled in their name should not only be viewed with understanding attitude, but as a matter of policy encouraged. 

WHEREFORE, the petition is hereby GRANTED and the decisions of the courts a quo are REVERSED. SO ORDERED.

G.R. No. L-44237 February 28, 1989

VICTORIA ONG DE OCSIO vs. COURT OF APPEALS 5

NARVASA, J.:

From the adverse judgment of the Court of Appeals,  affirming in toto that of the Trial Court,  the petitioner has come to this Court on an appeal by certiorari to plead for reversal of (1) the factual determination that she had sold the lot in controversy to private respondent, and (2) the legal conclusion that neither the 1973 nor the 1987 Constitution disqualifies the corporation known as the Religious of the Virgin Mary, from acquiring the land in question and registering it in its name. In light of the time-honored rule that findings of fact of the Court of Appeals are generally final, and the doctrine lately laid down by this Court on the precise legal issue now raised by petitioner, her appeal must fail.

The controversy at bar arose in connection with cadastral proceedings initiated by the Director of Lands, in behalf of the Republic, for the settlement and adjudication of title to a large tract of land measuring 261.5791 hectares, divided into 1,419 lots, situated in the City of Iligan. 

Victoria Ong de Ocsio (herein petitioner) seasonably presented an answer to the petition. She alleged that she was the owner, by purchase, of two (2) parcels of land with specific boundaries comprehended in the cadastral proceeding: Lot No. 1272, measuring 256 square meters, and Lot 1273 a road lot, measuring 21 square meters; and that as owner, she had been in possession of both lots for fifteen (15) years, and her predecessors-in-interest, for sixty (60) years.  Title to the same parcels of land was however claimed by the Religious of the Virgin Mary.  In its answer, it averred that it had bought the lots from Victoria Ong de Ocsio and had been in possession as owner thereof for over four years, and its possession and that of its predecessors was immemorial.

Evidence was received on these conflicting assertions after which the Cadastral Court rendered judgment, declaring that the evidence satisfactorily established that Victoria Ong de Ocsio had in truth sold Lot No. 1272 to the Religious of the Virgin Mary in virtue of a deed of sale dated April 12, 1956 (Exhibit 1), and Lot No. 1273 was a road right of way granted to the City of Iligan. The judgment contained the following dispositive portion, viz:

WHEREFORE, the court renders judgment adjudicating Cadastral Lot 1272, Iligan Cadastre, to the Religious of the Virgin Mary, a duly registered domestic religious corporation, the members of which are all Filipino citizens, with main office in the City of Manila, but the building existing thereon is hereby declared to be the property of claimant Victoria Ong de Ocsio who is hereby ordered to

5 Classifications

Page 11: Natural Resources Set2.Classification Cases

11NATURAL RESOURCES: CLASSIFICATIONS CASES

remove Said building out of the premises within 90 days from date hereof. The claim of Victoria Ong de Ocsio with respect to said cadastral lot is dismiss. No pronouncement is made as to costs.

Let the corresponding decree issue 30 days after this decision shall have become final.

As aforestated, the Court of Appeals affirmed the cadastral court's decision in toto. So, too, will this Court.

Both the cadastral Court and the Court of Appeals came to the conclusion, after analysing and weighing the testimonial and documentary evidence adduced by the parties, that Virginia Ong de Ocsio's version of the facts was not true-that it was another property, not Lot No. 1272, that she had conveyed to the religious corporation but that it was indeed Lot No. 1272 that was subject of the sale and had indeed been transferred to the latter. Now, findings of fact of this sort, contained in a decision of the Court of Appeals are by long and uniformly observed rule conclusive on the parties and on the Supreme Court, as well;  subject only to a few specified exceptions,  none of which obtains here, said findings may not be reviewed on appeal.

As regards the issue of law raised by her, petitioner fares no better. Citing Manila Electric Co. v. Castro-Bartolome, 114 SCRA 799 (1982) and Republic v. Villanueva, 114 SCRA 875 (1982), in relation to Section 11, Article XIV of the 1973 Constitution, she asserts that as the private respondent is a religious corporation, it is disqualified to obtain judicial confirmation of an imperfect title under Section 48(b) of the Public Land Act which grants that right only to natural persons. The cited rulings no longer control. Current doctrine, first announced by the Court en banc in Director of Lands v. I.A.C. 146 SCRA 509 (1986), is that open, continuous and exclusive possession of alienable public land for at least thirty (30) years in accordance with the Public Land Act ipso jureconverts the land to private property, and a juridical person who thereafter acquires the same may have title thereto confirmed in its name. Virtually the same state of facts obtained in said case that now obtain here. A private corporation had purchased the land originally of the public domain from parties who had, by themselves and through their predecessors-in-interest, possessed and occupied it since time immemorial. It had thereafter instituted proceedings for confirmation of title under Section 48(b) of the Public Land Act. In upholding its right to do so, the court held that the fact that the proceedings had been instituted by said purchaser in its own name and not in the name of the transferors was "xx simply xx (an) accidental circumstance, productive of a defect hardly more than procedural and in nowise affecting the substance and merits of the right of ownership sought to be confirmed." The ruling was reaffirmed in two later cases, Director of Lands v. Manila Electric Co., 153 SCRA 686 (September 11, 1987), and Republic v. C.A., 156 SCRA 344 (October 30, 1987) where the same question of law was raised. In the latter it was expressly held that the prohibitions in the 1973 and 1987 Constitutions against

acquisition or registration of lands by or in behalf of private corporations do not apply to public lands already converted to private ownership by natural persons under the provisions of the Public Land Act. In the present case, Virginia Ong de Ocsio and her predecessors-in-interest having possessed Lot No. 1272 for the period and under the conditions prescribed by law for acquisition of ownership of disposable public land prior to the sale of the property to the Religious of the Virgin Mary, confirmation of title thereto in the latter's name is, under the precedents referred to, entirely in order.

WHEREFORE, the judgment of the Court of Appeals subject of the petition for review on certiorari is AFFIRMEDin toto. Costs against the petitioner.

G.R. No. 129682      March 21, 2002NESTOR PAGKATIPUNAN vs. THE COURT OF APPEALS 6

YNARES-SANTIAGO, J.:

This is a petition for review of the decision of the Court of Appeals nullifying the decision of the Court of First Instance of Gumaca, Quezon which confirmed petitioners’ title over the lots subject of the instant petition. Petitioners further seek to annul and set aside the resolutions of the Court of Appeals denying their urgent motion to recall the judgment entered in the land registration case.

The antecedent facts are as follows:

Sometime in November 1960, petitioners’ predecessors-in-interest, spouses Getulio Pagkatipunan and Lucrecia Esquires, filed with the Court of First Instance of Gumaca, Quezon an application for judicial confirmation and registration of their title to Lots 1 and 2 of Plan Psu-174406 and Lots 1 and 2 of Plan Psu-112066, all located in San Narciso, Quezon.

On May 4, 1961, the Court of First Instance entered an order of default against the whole world, except spouses Felicisimo Almace and Teodulo Medenilla who were given ten (10) days to file their written opposition as regards Lot No. 2 of Plan Psu-174406. Upon motion of petitioner’s predecessors, Lot No. 2 of Plan Psu-174406 was removed from the coverage of the application. The remaining parcel of land covered by Lot No. 1 has an area of 3,804.261 square meters.

On June 15, 1967, the Court of First Instance promulgated a decision confirming petitioners’ title to the property. On October 23, 1967, OCT No. O-12665 was issued in the name of petitioners.

6 Classifications

Page 12: Natural Resources Set2.Classification Cases

12NATURAL RESOURCES: CLASSIFICATIONS CASES

Almost eighteen (18) years later, or on September 12, 1985, the Republic of the Philippines filed with the Intermediate Appellate Court an action to declare the proceedings in LRC Case No. 91-G, LRC Record No. N-19930 before the Court of First Instance of Gumaca, Quezon null and void, and to cancel Original Certificate of Title No. 0-12665 and titles derived therefrom as null and void, to direct the register of deeds to annul said certificates of title, and to confirm the subject land as part of the public domain.

The Republic claimed that at the time of filing of the land registration case and of rendition of the decision on June 15, 1967, the subject land was classified as timberland under LC Project No. 15-B of San Narciso, Quezon, as shown in BF Map No. LC-1180; hence inalienable and not subject to registration. Moreover, petitioners’ title thereto can not be confirmed for lack of showing of possession and occupation of the land in the manner and for the length of time required by Section 48(b), Commonwealth Act No. 141, as amended. Neither did petitioners have any fee simple title which may be registered under Act No. 496, as amended. Consequently, the Court of First Instance did not acquire jurisdiction over the res and any proceedings had therein were null and void.

On the other hand, petitioners raised the special defenses of indefeasibility of title and res judicata. They argued that due to the lapse of a considerable length of time, the judgment of the Court of First Instance of Quezon in the land registration case has become final and conclusive against the Republic. Moreover, the action for reversion of the land to the public domain is barred by prior judgment.

In a decision promulgated on June 27, 1986, the Intermediate Appellate Court held that the land in question was forestral land; hence not registrable. There was no evidence on record to show that the land was actually and officially delimited and classified as alienable or disposable land of the public domain. Therefore, the Court of First Instance did not acquire jurisdiction to take cognizance of the application for registration and to decide the same. Consequently, the action to declare null and void the June 15, 1967 decision for lack of jurisdiction did not prescribe. The dispositive portion of the appellate court’s decision reads:

WHEREFORE, judgment is rendered in favor of petitioner and against respondents, and as prayed for:(a) The Decision dated June 15, 1967 in LRC Case No. 91-G, LRC Record No. N-19930 is hereby declared null and void, and accordingly set aside;(b) Original Certificate of Title No. O-12665, and Transfer Certificates of Title Nos. T-84439, T-93857 and T-117618 deriving therefrom, as well as any other derivative titles, are declared null and void;(c) The respondent Register of Deeds for Quezon Province is ordered to cancel said titles; and(d) The parcels of land covered thereby are ordered reverted to the State.

Without pronouncement as to costs."

On July 16, 1986, petitioners moved for the reconsideration of the afore-cited decision reiterating that the land in question was agricultural because it was possessed and cultivated as such long before its classification as timberland by the Bureau of Forestry in 1955. Petitioners and their predecessors-in-interest have been in open, continuous, exclusive, notorious possession and occupation of said land for agricultural and cattle raising purposes as far back as the Spanish regime. Following the doctrine in Oracoy v. Director of Lands, private interest had intervened and petitioners acquired vested rights which can no longer be impaired by the subsequent classification of the land as timberland by the Director of Forestry.

On August 20, 1986, the appellate court denied the motion for reconsideration for lack of merit. On December 12, 1986, the decision of June 27, 1986 attained finality and judgment was entered in the book of entries of judgments.

On April 2, 1987, petitioners filed an urgent motion to set aside entry of judgment on the ground that Atty. Cirilo E. Doronila, petitioners’ counsel of record, was not furnished a copy of the resolution denying the motion for reconsideration.14 In the absence of such notice, the decision of the appellate court did not become final and executory.

On October 22, 1987, the Court of Appeals set aside and lifted the entry of judgment in CA-G. R. SP No. 07115 and directed the clerk of court to furnish petitioners’ counsel a copy of the August 20, 1986 resolution.

For petitioners’ inaction despite service of the August 20, 1986 resolution, the June 27, 1986 decision became final and executory. On March 2, 1988, entry of judgment was again made in the land registration case.

On September 4, 1995, Atty. Doronila withdrew his appearance as counsel for petitioners.

On April 1, 1996, petitioners, through their new counsel, Atty. George I. Howard, filed with the Court of Appeals an urgent motion to recall the entry of judgment, which was denied by the appellate court on December 16, 1996.

The motion for reconsideration was likewise denied on the ground that it raised arguments already discussed and resolved in the urgent motion to recall entry of judgment.

Hence, the instant petition for review.

Page 13: Natural Resources Set2.Classification Cases

13NATURAL RESOURCES: CLASSIFICATIONS CASES

Petitioners claim that their title to the land became incontrovertible and indefeasible one (1) year after issuance of the decree of registration. Hence, the Republic’s cause of action was barred by prescription and res judicata, proceedings having been initiated only after about 18 years from the time the decree of registration was made. Contrary to the appellate court’s findings, the land is agricultural and the inclusion and classification thereof by the Bureau of Forestry in 1955 as timberland can not impair the vested rights acquired by petitioners’ predecessors-in-interest who have been in open, continuous, adverse and public possession of the land in question since time immemorial and for more than thirty (30) years prior to the filing of the application for registration in 1960. Hence, the Court of Appeals committed grave error when it denied their motion to set aside entry of judgment in the land registration case.

The petition lacks merit.

Unless public land is shown to have been reclassified or alienated to a private person by the State, it remains part of the inalienable public domain. Occupation thereof in the concept of owner, no matter how long, cannot ripen into ownership and be registered as a title.

Evidence extant on record showed that at the time of filing of the application for land registration and issuance of the certificate of title over the disputed land in the name of petitioners, the same was timberland and formed part of the public domain, as per certification issued by the Bureau of Forest Development on April 1, 1985, thus:

TO WHOM IT MAY CONCERN:This is to certify that the tract of land situated in Vigo Cantidang, San Narciso, Quezon, containing an area of 3,804.261 square meters as described in Transfer Certificate of Title No. T-117618 x x x registered in the name of Spouses Nestor E. Pagkatipunan and Rosalina Mañgas is verified to be within the Timberland Block -B, Project No. 15-B of San Narciso, Quezon, certified and declared as such on August 25, 1955 per BFD Map LC-1880. The land is, therefore, within the administrative jurisdiction and control of the Bureau of Forest Development, and not subject to disposition under the Public Land Law.[Sgd.]ARMANDO CRUZSupervising Cartographer

This fact was even admitted by petitioners during the proceedings before the court a quo on March 10, 1986, when they confirmed that the land has been classified as forming part of forest land, albeit only on August 25, 1955. Since no imperfect title can be confirmed over lands not yet classified as disposable or alienable, the title issued to herein petitioners is considered void ab initio.

Under the Regalian doctrine, all lands of the public domain belong to the State, and the State is the source of any asserted right to ownership in land and charged with the conservation of such patrimony. This same doctrine also states that all lands not otherwise appearing to be clearly within private ownership are presumed to belong to the State. To overcome such presumption, incontrovertible evidence must be shown by the applicant that the land subject of the application is alienable or disposable.

In the case at bar, there was no evidence showing that the land has been reclassified as disposable or alienable. Before any land may be declassified from the forest group and converted into alienable or disposable land for agricultural or other purposes, there must be a positive act from the government. Even rules on the confirmation of imperfect titles do not apply unless and until the land classified as forest land is released in an official proclamation to that effect so that it may form part of the disposable agricultural lands of the public domain. Declassification of forest land is an express and positive act of Government. It cannot be presumed. Neither should it be ignored nor deemed waived. It calls for proof.

The court a quo found registrable title in favor of petitioners based on the Republic’s failure to show that the land is more valuable as forest land than for agricultural purposes, a finding which is based on a wrong concept of what is forest land.

There is a big difference between "forest" as defined in the dictionary and "forest or timber land" as a classification of land of the public domain in the Constitution. One is descriptive of what appears on the land while the other is a legal status, a classification for legal purposes. The "forest land" started out as a "forest" or vast tracts of wooded land with dense growths of trees and underbrush. However, the cutting down of trees and the disappearance of virgin forest do not automatically convert the land of the public domain from forest or timber land to alienable agricultural land.

The classification of forest land, or any land for that matter, is descriptive of its legal nature or status, and does not have to be descriptive of what the land actually looks like.32 A person cannot enter into forest land and by the simple act of cultivating a portion of that land, earn credits towards an eventual confirmation of imperfect title. The Government must first declare the forest land to be alienable and disposable agricultural land before the year of entry, cultivation, and exclusive and adverse possession can be counted for purposes of an imperfect title.

As ruled in the case of Heirs of Jose Amunategui v. Director of Forestry:

Page 14: Natural Resources Set2.Classification Cases

14NATURAL RESOURCES: CLASSIFICATIONS CASES

A forested area classified as forest land of the public domain does not lose such classification simply because loggers or settlers may have stripped it of its forest cover. Parcels of land classified as forest land may actually be covered with grass or planted to crops by kaingin cultivators or other farmers. "Forest lands" do not have to be on mountains or in out of the way places. Swampy areas covered by mangrove trees, nipa palms, and other trees growing in brackish or sea water may also be classified as forest land. The classification is descriptive of its legal nature or status and does not have to be descriptive of what the land actually looks like. Unless and until the land classified as "forest" is released in an official proclamation to that effect so that it may form part of the disposable agricultural lands of the public domain, the rules on confirmation of imperfect title do not apply.

Moreover, the original text of Section 48 (b), Chapter VIII of the Public Land Act, which took effect on December 1, 1936, expressly provided that only agricultural land of the public domain are subject to acquisitive prescription, to wit:

Section 48. x x x(a) x x x(b) Those who by themselves or through their predecessors-in-interest have been in open, continuous, exclusive, and notorious possession and occupation of agricultural lands of the public domain, under a bona fide claim of acquisition of ownership, except as against the Government, since July twenty-six, eighteen hundred and ninety-four, except when prevented by war or force majeure. These shall be conclusively presumed to have performed all the conditions essential to a Government grant and shall be entitled to a certificate of title under the provisions of this Chapter. (Emphasis supplied)

Thus, it is clear that the applicant must prove not only his open, continuous, exclusive and notorious possession and occupation of the land either since time immemorial or for the period prescribed therein, but most importantly, he must prove that the land is alienable public land. In the case at bar, petitioners failed to do so.

Petitioners’ contention that the Republic is now barred from questioning the validity of the certificate of title issued to them considering that it took the government almost eighteen (18) years to assail the same is erroneous. It is a basic precept that prescription does not run against the State. The lengthy occupation of the disputed land by petitioners cannot be counted in their favor, as it remained part of the patrimonial property of the State, which property, as stated earlier, is inalienable and indisposable.

In light of the foregoing, the Court of Appeals did not err when it set aside the June 15, 1967 decision of the courta quo and ordered that the subject lot be reverted back to the public domain. Since the land in question is unregistrable, the

land registration court did not acquire jurisdiction over the same. Any proceedings had or judgment rendered therein is void and is not entitled to the respect accorded to a valid judgment.

Consequently, the Court of Appeals rightfully denied petitioners’ motion to set aside the judgment rendered on December 12, 1986, in the land registration case.

WHEREFORE, in view of the foregoing, the decision of the Court of Appeals dated June 27, 1986 in AC-G.R. SP No. 07115, is hereby AFFIRMED in toto. Without pronouncement as to costs. SO ORDERED.

G.R. No. 127296 January 22, 1998EDUBIGIS GORDULA vs. THE HONORABLE COURT OF APPEALS and REPUBLIC OF THE PHILIPPINES7

PUNO, J.:

Before us is a petition to affirm the Decision of the Regional Trial Court, Branch 27, Sta. Cruz, Laguna, which was reversed by the respondent Court of Appeals in its Decision  dated June 20, 1996 in C.A.-G.R. CV No. 45466. Petitioners' Motion for Reconsideration was denied by respondent court on November 15, 1996. 

The facts show that on June 26, 1969, former President Ferdinand E. Marcos issued Proclamation No. 573  withdrawing from sale and settlement and setting aside as permanent forest reserves, subject to private rights, certain parcels of the public domain denominated as follows:

Parcel No. 1. — Magat River Forest ReserveParcel No. 2 — Chico River Forest ReserveParcel No. 3 — Abulug River Forest ReserveParcel No. 4 — Penaranda River Forest ReserveParcel No. 5 — Angat River-Bustos Dam Forest ReserveParcel No. 6 — Ambayawan River Forest ReserveParcel No. 7 — Umiray River Forest ReserveParcel No. 8 — Kaliwa River Forest ReserveParcel No. 9 — Caliraya-Lumot River Forest ReserveParcel No. 10 — Barit River-Lake Buhi Forest ReserveParcel No. 11 — Jalaur River Forest Reserve

7 Classifications

Page 15: Natural Resources Set2.Classification Cases

15NATURAL RESOURCES: CLASSIFICATIONS CASES

They were primarily for use as watershed area. Their development was to be undertaken by the Bureau of Forestry, with the cooperation of, among other government agencies, the National Power Corporation (Napocor).

Located in Talaongan, Cavinti, Laguna with an area of Twenty Nine Thousand Seven Hundred Seven (29,707) square meters, and bearing the following boundaries:

North — National Power CorporationSouth — Road and Baldomero HaliliWest — National Power CorporationEast — National Power Corporation

the parcel of land subject of the case at bar is, by petitioners' explicit admission,  within Parcel No. 9, the Caliraya-Lumot River Forest Reserve.

More than three years after the land was segregated as part of the Caliraya-Lumot River Forest Reserve, or on January 9, 1973, petitioner Edubigis Gordula, a native of Cavinti, Laguna, filed with the Bureau of Lands, an Application   for a Free Patent over the land. Manuel Fernandez and several others also filed free patent applications covering other parcels of land in the area.

On February 5, 1973, petitioner Gordula declared the land for taxation purposes in his name as shown in Tax Declaration No. 0429.

The Regional Director of the Bureau of Lands referred the free patent applications of petitioner Gordula, Fernandez, and several others to Mr. Ravanal Ravanzo, then the General Manager of the Napocor. Mr. Ravanzo responded through the following letter:

October 24, 1973The DirectorRegional Lands Office No. IV757 Gen. Solano St.San Miguel, ManilaSir:

This refers to the Free Patent Application[s] of Manuel Fernandez, et al., of Barrio Talaongan, Cavinti, Laguna, which were referred to this Office for clearance it having been found that they are within the Caliraya-Lumot Watershed Reservation under Proclamation No. 573 dated June 26, 1969.Investigation conducted by this Office reveals that applicants have sufficient ground to establish "priority rights" over the areas claimed and that agricultural improvements introduced thereon are not detrimental to the watershed.

In view thereof, this Office interpose[s] no objection to the application by the applicants contained in your letter dated October 2, 1973.

On January 30, 1974, the Register of Deeds of Laguna issued, on the basis of Free Patent No. 693, Original Certificate of Title No. P-1405 in the name of petitioner Gordula.

He declared the land anew for taxation purposes under Tax Declaration No. 6498. He paid its real estate taxes from 1975 to 1979.

In the meantime, respondent Republic, through the Napocor, contracted the Certeza Surveying Company to survey the area constituting the Caliraya-Lumot River Forest Reserve. The survey plans were approved by the Regional Director of the Bureau of Lands on October 27, 1975.

The said survey plans, as well as the Cadastral Map of Talaongan and the Geological Plan of the Caliraya-Lumot River Forest Reserve, show that petitioner Gordula's land is located in the saddle area of the watershed recreation for the hydro-electric reservoir.

On January 22, 1979, petitioner Gordula sold the land to petitioner Celso V. Fernandez, Jr. for six thousand pesos (P6,000.00). The Register of Deeds of Laguna cancelled Original Certificate of Title No. P-1405 and issued, in lieu thereof, Transfer Certificate of Title No. T-85445 in the name of petitioner Fernandez, Jr. The latter declared the land for taxation purposes in his name.

On March 12, 1979, Fernandez, Jr. executed a Deed of Absolute Sale over the land in favor of petitioner Celso A. Fernandez for six thousand five hundred pesos (P6,500.00). Transfer Certificate of Title No. T-85445 was cancelled and Transfer Certificate of Title No. 85594 was issued on March 21, 1979 in the name of petitioner Fernandez.

As approved by the Bureau of Lands in Psd-Plan 04-014230, petitioner Fernandez subdivided the land into nine (9) lots. On August 16, 1985, the Register of Deeds of Laguna issued Transfer Certificates of Title Nos. 102492 to 102500 in his name covering the nine (9) subdivision lots.

On August 29, 1985, he sold the lots to petitioner Nora Ellen Estrellado for twenty one thousand pesos (P21,000.00). Transfer Certificates of Title Nos. 102492 to 102500 were cancelled, and in lieu thereof, Transfer Certificates of Title Nos. T-103404 to T-103412 were issued to petitioner Estrellado.

Page 16: Natural Resources Set2.Classification Cases

16NATURAL RESOURCES: CLASSIFICATIONS CASES

On October 17, 1986, petitioner Estrellado mortgaged to petitioner Development Bank of the Philippines (DBP) four  (4) of the (9) lots. Another lot, covered by Transfer Certificate of Title No. 103408, was sold to petitioner J.F. Festejo Company, Inc. to whom was issued, in lieu of the former, Transfer Certificate of Title No. 106495.

On July 16, 1987, former President Corazon Aquino issued Executive Order (E.O.) No. 224  vesting in the Napocor "complete jurisdiction, control and regulation" over the "Caliraya-Lumot Watershed Reservation as covered by Proclamation No. 573".

On July 26, 1987, Mr. Antonio Aquino, Jr., the Civil Security Officer of the Cavinti reservoir complex, sent a Memorandum to the President of the Napocor informing him of the fences and roads being constructed in the saddle area, more particularly, in the lots sold by petitioner Fernandez to petitioner Estrellado.

On July 28, 1987, Mr. A. Coronado, the Manager of the Cavinti reservoir complex, asked petitioner Fernandez to remove all the improvements made in the Estrellado lots. In reply, petitioner Fernandez claimed that the roads being constructed would not adversely affect the reservoir area in case of heavy floods because the Estrellado lots were elevated at a height of around fifty (50) feet.

In view of petitioner Fernandez's, refusal, the Napocor assigned two (2) security guards over the lot. The guards ordered the construction workers to leave their posts and barred their return without permission from the Napocor.

On October 18, 1987, petitioner Fernandez, as attorney-in-fact and counsel of petitioner Estrellado, wrote to the President of the Napocor threatening to file a multi-million damage suit if the guards were not removed within fifteen (15) days.

On November 18, 1987, respondent Republic, through the Napocor, filed against petitioners a Complaint for Annulment of Free Patent and Cancellation of Titles and Reversion with Writ of Preliminary Injunction in the RTC of Sta. Cruz, Laguna. 8 On January 29, 1988, the trial court issued a writ of preliminary injunction upon a bond of one hundred thousand pesos (P100,000.00).

On December 28, 1993, the trial court rendered judgment in favor of petitioners. The dispositive portion of its decision states:

WHEREFORE, judgment is hereby rendered in favor of the defendants and against plaintiff:(1) Dismissing plaintiff's complaint and dissolving the writ of preliminary injunction issued in this case;

(2) Ordering National Power Corporation to pay defendant Celso A. Fernandez P300,000.00 as actual damages and P30,000.00 as attorney's fees; andWith costs against the plaintiff.SO ORDERED. 

Respondent Republic, through the Napocor, elevated the case to the respondent Court of Appeals.

On June 20, 1996, the respondent Court of Appeals ruled against petitioners. It held, inter alia, viz:

The kernel and primal issue to be resolved by the Court is whether or not Free Patent No. IV-5 (693) and Original Certificate of Title No. P-1405 and all derivative titles thereafter issued to the Appellees . . . are null and void. The Appellant avers that the parcel of land covered by the aforesaid Free Patent issued to Gordula is a portion of the vast track of land reserved by former President Marcos as permanent forest under Proclamation No. 573 dated June 26, 196[9] . . . and hence, non-disposable and inalienable, pursuant to Section 88 in relation to Section 83 of Commonwealth Act [No.] 141, as amended. In contrast the Court a quo dismissed Appellant's complaint, in the light of the exclusionary clause in Proclamation No. 573 . . . that the setting up of the permanent forest reserves over the Caliraya-Lumot Watershed area was "subject to private rights" if there be any and the letter-clearance of the then General Manager of [Napocor] . . . dated October 24, 1973, interposing no objection to the Application for a free patent of Manuel Fernandez, et al.

xxx xxx xxx

We are convinced, beyond cavil, that the parcel of land subject of the Free Patent issued to Gordula on January 17, 1974 and covered by Original Certificate of Title No. P-1405 issued on January 30, 1974 . . . as the two (2) parcels of land purportedly purchased by the [Republic] from Perez and Glorioso in 1941, were public disposable and alienable lands before the issuance, by the former President, of Proclamation No. 573, on June 26, 196[9] . . . The property was, however, later reserved, under Proclamation No. 573, as a permanent forest, on June 26, 196[9]. Since then, the property became non-disposable and inalienable public land. . . .

xxx xxx xxx

At the time Gordula filed his application for a Free Patent, on January 9, 1973, the parcel of land . . . was already reserved as a permanent forest under Proclamation No. 573. Since the property was already a forest reservation as of June 26, 196[9], the same could no longer be disposed of or alienated in favor of private individuals . . . .

Page 17: Natural Resources Set2.Classification Cases

17NATURAL RESOURCES: CLASSIFICATIONS CASES

xxx xxx xxx

We do not agree with Appellees' and the Court a quo's pose that Gordula's property was exempt from the application of Proclamation No. 573 because, by express provision thereof, the reservation was "subject to private rights, if there be any" . . . .

Appellees failed to adduce proof that, as of June 26, 196[9], Gordula had acquired ownership or title to the aforesaid property either by deed or by any other mode of acquisition from the State by operation of law for that matter such as for instance, alienable public land held by a possessor personally, or through his predecessors-in-interest, openly, continuously and exclusively for the prescribed period of thirty (30) years, is converted into private property by mere lapse of period ipso jure . . . . In the present recourse, Gordula, as of 196[9], had been in possession of the property for only [twenty-five (25) years] years since 1944 when he commenced, as can be gleaned from his application . . . for a free patent, possession of the property. The period of Gordula's occupancy after 196[9] should not be tacked to the period from 1944 because by then theproperty was not susceptible of occupancy, disposition, conveyance or alienation. . . .

xxx xxx xxx

The Appellees cannot find refuge in the letter of the then General Manager of [Napocor], Ravanal Ravanzo, on October 24, 1973 . . . .

In the first place, Ravanzo made no explicit and unequivocal statement, in said letter, that Gordula had priority rights to the property. What he merely declared was that "applicants have sufficient ground to establish priority rights over the areas claimed . . ." . Even if it may be conceded, for the nonce, that indeed, Ravanzo declared that Gordula had priority rights over the property claimed by him, such declaration is irrefragably erroneous. Munda and the Director of Lands erred in recommending the approval of Gordula's application in the same manner that the then Secretary of Agriculture and Natural Resources erred in issuing the patent to Gordula. But then, well-settled is the doctrine, enunciated by the Supreme Court, in a catena of cases, that the State cannot be bound and estopped by the errors or mistakes of its agents or officials . . . .

The General Manager of the Appellant is not vested with authority to allow the occupancy or acquisition, by private individuals, of such properties, whether still needed by the Appellant or not, reserved by the President of the Philippines for permanent forests. Only the President or [the] Congress, by statutory fiat, can revert the property to the disposable or alienable portion of the public domain.

Anent Appellees' plea that they are buyers of the property in good faith, they must harken to the Decision of the Supreme Court in Republic of the Philippines vs. Court of Appeals, et al., 148 SCRA 480 that:

. . . even assuming that the transferees are innocent purchasers for value, their titles to said lands derived from the titles of private respondents which were not validly issued as they cover lands still a part of the public domain, may be cancelled. . . .

We do not agree with Appellees' claim that Appellant's suit was barred by prescription and by the purported indefeasibility of their title. Prescription, basically, does not run against the State. The right of the State for the reversion of unlawfully acquired property is not barred by prescription nor by the perceived indefeasibility of Appellees' title for that matter. . . . 

Thus states the dispositive portion of the decision of respondent appellate court:

IN THE LIGHT OF ALL THE FOREGOING, the assailed Decision is hereby REVERSED and SET ASIDE. Another Decision is hereby rendered as follows:

1. Free Patent No. IV-5-693 and Original Certificate of Title No. P1405 issued under the name of Edubigis Gordula and all derivative titles issued to the Appellees are hereby declared null and void;2. The parcel of land covered by said titles is hereby declared reverted to the Government under the jurisdiction, control and supervision of the [Napocor] under Executive Order No. 224 of former President Corazon C. Aquino;3. The Appellees and all those acting for and in their behalf are hereby prohibited from intruding into and disturbing the Appellant of its possession and dominion of the subject property; [and]4. Appellees' counterclaims are DISMISSED. No pronouncement as to costs.

SO ORDERED. 

Hence, this petition anchored on the following grounds:

FIRSTRESPONDENT COURT OF APPEALS ERRED TANTAMOUNT TO LACK OF JURISDICTION WHEN IT CONCLUDED THAT THE SUBJECT LAND IS WITHIN THE LANDS BOUGHT BY THE NPC EITHER FROM GERONIMO PEREZ ON MARCH 10, 1941 AND/OR FROM CELERINO GLORIOSO ON SEPTEMBER 26, 1941;

SECOND

Page 18: Natural Resources Set2.Classification Cases

18NATURAL RESOURCES: CLASSIFICATIONS CASES

ON [sic] THE LAST PARAGRAPH OF PAGE 19 UP TO PAGE 23, LAST PAGE OF THE DECISION, THE RESPONDENT COURT WENT BEYOND THE ISSUES OF THE CASE WHICH RESULTED [IN THE] REVERSAL OF THE DECISION OF THE LOWER COURT . . . ;

THIRDTHE FACTUAL FINDINGS AND CONCLUSION OF THE TRIAL COURT ARE IN CONFLICT WITH THE FINDINGS OF THE RESPONDENT COURT CONCERNING THE ISSUE OF WHETHER OR NOT PETITIONER EDUBIGIS GORDULA HAD ACQUIRED "PRIVATE RIGHTS" ON THE SUBJECT LAND, WHICH IS AN EXCEPTION UNDER PROCLAMATION NO. 573. HENCE, THIS CASE IS A QUESTION OF FACTS AND OF LAW . . . ;FOURTH

THERE IS NO QUESTION THAT THE SUBJECT LAND IS WITHIN THE AREA OF PROCLAMATION NO. 573. HOWEVER THE RESPONDENT [COURT] GRAVELY ERRED TANTAMOUNT TO LACK OF JURISDICTION WHEN IT WENT TO THE EXTENT OF DISCUSSING ON [sic] THE CIRCUMSTANCES AND INVESTIGATION RELATIVE TO THE ISSUANCE OF THE TITLE TO PETITIONER EDUBIGIS GORDULA AND AFTERWARD DECLARED THAT GORDULAWHO HAS A TITLE ON THE SUBJECT LAND HAS NOT ACQUIRED "PRIVATE RIGHTS" ON THE LAND DESPITE OF [sic] THE FACT THAT SAID RESPONDENT COURT IS ALREADY PRECLUDED FROM DISCUSSING THE FACTS RELATIVE ON [sic] THE ISSUANCE OF THE TITLE BY AUTHORITY OF THE PRESIDENT OF THE PHILIPPINES, MORE SO ITS FINDINGS AND CONCLUSION IS [sic] AGAINST THE LAW, JUSTICE AND EQUITY. THIS IS AGAINST THE RULING IN ESPINOSA VS. MAKALINTAL, 79 PHIL. 134 and ORTUA VS. SINGSON ENCARNACION, 5[9] PHIL. 440; and

FIFTHTHE RESPONDENT COURT GRAVELY ERRED IN CONCLUDING THAT THEN GENERAL MANAGER RAVANZO OF NPC AND UNDERSECRETARY OF AGRICULTURE BY AUTHORITY OF THE PRESIDENT OF THE PHILIPPINES ERRED IN ISSUING THE PATENT TO PETITIONER GORDULA. THIS IS AGAINST THE RULING IN ESPINOSA VS. MAKALINTAL, 79 PHIL. 134 and ORTUA VS. SINGSON ENCARNACION, 5[9] PHIL. 440. 

We affirm the Court of Appeals.

We start with the proposition that the sovereign people, represented by their lawfully constituted government, have untrammeled dominion over the forests on their native soil. Forest lands, being the self-replenishing, versatile and all-important natural resource that they are, need to be reserved and saved to promote the people's welfare. By their very nature  or by executive or statutory fiat, they are outside the commerce of man, unsusceptible of private appropriation in any form,   and

inconvertible into any character less than of inalienable public domain, regardless of their actual state, for as long as the reservation subsists and is not revoked by a subsequent valid declassification. 

Once again, we reiterate the rule enunciated by this Court in Director of Forestry vs. Munoz and consistently adhered to in a long line of cases the more recent of which is Republic vs. Court Appeals, that forest lands or forest reserves are incapable of private appropriation, and possession thereof, however long, cannot convert them into private properties. This ruling is premised on the Regalian doctrine enshrined not only in the 1935 and 1973 Constitution but also in the 1987 Constitution. 

Petitioners do not contest the nature of the land in the case at bar. It is admitted that it lies in the heart of the Caliraya-Lumot River Forest Reserve, which Proclamation No. 573 classified as inalienable and indisposable. Its control was vested in the NAPOCOR under E.O. No. 224.

Petitioners, however, contend that Proclamation No. 573 itself recognizes private rights of landowners prior to the reservation. They claim to have established their private rights to the subject land.

We do not agree. No public land can be acquired by private persons without any grant, express or implied from the government; it is indispensable that there be a showing of the title from the state. 17 The facts show that petitioner Gordula, did not acquire title to the subject land prior to its reservation under Proclamation No. 573. He filed his application for free patent only in January, 1973, more than three (3) years after the issuance of Proclamation No. 573 in June, 1969. At that time, the land, as part of the Caliraya-Lumot River Forest Reserve, was no longer open to private ownership as it has been classified as public forest reserve for the public good.

Nonetheless, petitioners insist that the term, "private rights", in Proclamation No. 573, should not be interpreted as requiring a title. They opine that it suffices if the claimant "had occupied and cultivated the property for so may number of years, declared the land for taxation purposes, [paid] the corresponding real estate taxes [which are] accepted by the government, and [his] occupancy and possession [is] continuos, open and unmolested and recognized by the government". 18 Prescinding from this premise, petitioners urge that the 25-year possession by petitioner Gordula from 1944 to 1969, albeit five (5) years short of the 30-year possession required under Commonwealth Act (C.A.) No. 141, as amended, is enough to vest upon petitioner Gordula the "private rights" recognized and respected in Proclamation No. 573.

Page 19: Natural Resources Set2.Classification Cases

19NATURAL RESOURCES: CLASSIFICATIONS CASES

The case law does not support this submission. In Director of Lands v. Reyes,  we held that a settler claiming the protection of "private rights" to exclude his land from a military or forest reservation must show ". . . by clear and convincing evidence that the property in question was acquired by [any] . . . means for the acquisition of public lands".

In fine, one claiming "private rights" must prove that he has complied with C. A. No. 141, as amended, otherwise known as the Public Land Act, which prescribes the substantive as well as the procedural requirements for acquisition of public lands. This law requires at least thirty (30) years of open, continuous, exclusive and notorious possession and occupation of agricultural lands of the public domain, under a bona fide claim of acquisition, immediately preceding the filing of the application for free patent. The rationale for the 30-year period lies in the presumption that the land applied for pertains to the State, and that the occupants and/or possessors claim an interest therein only by virtue of their imperfect title or continuous, open and notorious possession.

Indeed, the possession of public agricultural land, however long the period may have extended, never confers title thereto upon the possessor.   The reason, to reiterate our ruling, is because the statute of limitations with regard to public agricultural land does not operate against the State, unless the occupant can prove possession and occupation of the same under claim of ownership for the required number of years to constitute a grant from the State. 

In the case at bar, petitioners have failed to comply with the mandatory 30-year period of possession. Their 25-year possession of the land prior to its reservation as part of the Caliraya-Lumot River Forest Reserve cannot be considered compliance with C.A. No. 141, as amended. The Court has no authority to lower this requirement for it cannot amend the law.

Next, petitioners contend that their "private rights" have been recognized by the government itself. They point to (1) the letter dated October 24, 1973 of then NAPOCOR General Manager, Ravanal Ravanzo, (2) the action of the Bureau of Lands which after investigation, declared him qualified to acquire the land; and (3) the Free Patent issued on January 17, 1974 by the Undersecretary of Agriculture and Natural Resources, by authority of the President of the Philippines. Petitioners urge that the findings and conclusions of the aforementioned government agencies and/or officers are conclusive and binding upon the courts, as held in the cases of Ortua v. Singson Encarnacion  and Espinosa v. Makalintal. 

The submissions are unconvincing.

In the first place, there is nothing in Espinosa v. Makalintal that is relevant to petitioners' claims. On the other hand, our ruling in Ortua v. Singson Encarnacion that "a decision rendered by the Director of Lands and approved by the Secretary of Agriculture and Commerce, upon a question of fact is conclusive and not subject to be reviewed by the courts,"   was made subject to the categorical caveat "in the absence of a showing that such decision was rendered in consequence of fraud, imposition, or mistake". 

Undoubtedly, then General Manager Ravanzo erred in holding that petitioner Gordula "ha[d] sufficient ground to establish 'priority rights' over the areas claimed". This error mothered the subsequent error of the Bureau of Lands which culminated in the erroneous grant of a free patent on January 17, 1974. The perpetration of these errors does not have the effect of converting a forest reserve into public alienable land. It is well-settled that forest land is incapable of registration, and its inclusion in a title nullifies that title.  To be sure, the defense of indefeasibility of a certificate of title issued pursuant to a free patent does not lie against the state in an action for reversion of the land covered thereby when such land is a part of a public forest or of a forest reservation, the patent covering forest land being void ab initio.  Nor can the mistake or error of its officials or agents in this regard be invoked against the government. Finally, the conversion of a forest reserve into public alienable land, requires no less than a categorical act of declassification by the President, upon the recommendation of the proper department head who has the authority to classify the lands of the public domain into alienable or disposable, timber and mineral lands. There is none such in this case.

IN VIEW WHEREOF, the petition is HEREBY DENIED. No costs. SO ORDERED.

G.R. No. 105630           February 23, 2000REPUBLIC OF THE PHILIPPINES vs. ENRIQUE P. DE GUZMAN8

PARDO, J.:

This is a petition for review assailing the decision of the Court of Appeals, reversing and setting aside that of the Regional Trial Court, Branch 22, General Santos City, and dismissing the complaint for reversion of lot 5249, Ts-217, situated at Dadiangas. General Santos City and cancellation of titles, for lack of merit.

The facts are as follows:

After public bidding held on March 18, 1950, the Board of Liquidators, awarded Lot 5249 Ts-217, a 450 square meter land situated in Dadiangas, General Santos City, to Eusebio Diones of Takurong, Bubon, Cotabato. On March 11, 1955, Eusebio Diono transferred his rights over the lot to Enrique P. de

8 Classifications

Page 20: Natural Resources Set2.Classification Cases

20NATURAL RESOURCES: CLASSIFICATIONS CASES

Guzman (de Guzman, for brevity) for P700.00, evidenced by an Agreement of Transfer of Right. On November 12, 1956, the Board of Liquidators cancelled the award previously given to Eusebio Diones.

From the time he purchased the lot, de Guzman did not occupy it. In 1963, Lucena Ong Ante, another claimant of Lot 5249 Ts-217, authorized Carmen Ty to occupy the land. Ong Ante paid the corresponding real estate taxes from 1963 until 1980. Carmen Ty remained the occupant of the land until this time.

On August 12, 1967, de Guzman filed with the Board of Liquidators, Miscellaneous Sales Application No. 00222-E, and submitted supporting documents.

On August 29, 1967, the Director of Lands ordered the awards and issuance of a patent in favor of de Guzman. On September 5, 1967, the Department of Agriculture and Natural Resources approved and issued Miscellaneous Sales Patent No. 814 to de Guzman. On September 26, 1967, the Register of Deeds, General Santos, issued Original Certificate of Title No. P-29712 in the name of de Guzman.9

Sometime in 1973, de Guzman sold the lot to his married daughter and her husband, Carolina R. de Guzman and Rio Rivera for P5,000.00. The covering deed of sale could not be located and Rio Rivera admitted that his father-in-law Enrique P. de Guzman was not in occupation of the lot in question. On September 4, 1973, the Register of Deeds of General Santos City issued Transfer Certificate of Title No. T-7203 to spouses Rio Rivera and Carolina R. de Guzman. On March 21, 1974, Lucena Ong-Ante's adverse claim was annotated on the title of the lot.

Meanwhile, on February 13, 1974, spouses Rivera and Hoeschst, Phils., Inc., as mortgagor and mortgagee, respectively, executed a deed of real estate mortgage involving the lot. For failure to settle their obligation, on October 29, 1977, mortgagee Hoechst Phils., Inc., foreclosed on the mortgage and acquired the lot at the foreclosure sale. A certificate of sale was issued in favor of Hoechst Phils., Inc. However, for unknown reasons, the real estate mortgage and certificate of sale were not registered with the Register of Deeds. Thus, the transfer certificate of title remained in the name of spouses Rivera.

On January 14, 1981, petitioner Republic of the Philippines filed with the Court of First Instance, 16th Judicial District, General Santos City re-amended complaint for reversion of Lot 5249 Ts-217 and cancellation of titles against Enrique P. de Guzman, spouses Rio Rivera and Carolina R. de Guzman, the City Registrar of Deeds, General Santos City and Hoechst Phils., Inc.

In its re-amended complaint, petitioner Republic of the Philippines averred that Enrique P. de Guzman obtained Original Certificate of Title No. P-29712 through

fraudulent means. Petitioner contended since Lot 5249 Ts-217 was awarded to Eusebio Diones, hence, Eusebio Diono had no right to execute a deed of transfer in favor of Enrique de Guzman. Petitioner maintained that the documents presented by de Guzman to support his miscellaneous sales application were either issued by fictitious persons who were not employees of the Board of Liquidators, or contained inconsistencies that cast doubt on their authenticity.

De Guzman was neither in actual possession of the land, nor made improvements thereto, as he alleged in his sales application. Actual possession of the land by the applicant and making improvements thereto were among the legal requirements to be complied with by an applicant. Thus, Miscellaneous Sales Patent No. 814 and Original Certificate of Title No. P-29712 issued to de Guzman pursuant thereto were null and void. Also, Transfer Certificate of Title No. T-2703 in the name of spouses Rivera was null and void for they were not innocent purchasers for value. Admittedly, they knew that their vendor de Guzman was not in possession of the lot. Petitioner asserted that Lot 5249 Ts-217 must be reverted back to the mass of public domain.

On July 9, 1987, the trial court rendered decision in favor of petitioner. It held that the supporting documents submitted by de Guzman were falsified, hence, OCT No. P-29712 issued in his name was obtained through fraudulent means. Furthermore, the fact that de Guzman was not in possession of the property disqualified him from being awarded the sales patent. The trial court ruled that Rio Rivera and Carolina R. de Guzman were not innocent purchasers for value since their close relationship with Enrique P. de Guzman put them on notice of knowledge of a defect in the acquisition of title to the land. The trial Court ordered the reversion of the land to the mass of public domain.

The dispositive portion decreed as follows:

WHEREFORE, IN VIEW OF ALL THE FOREGOING, plaintiff having preponderantly proven the allegations of the complaint, judgment is hereby rendered:

1. Declaring the Miscellaneous Sales Patent No. 814 and OCT No. P-29712 in the name of Enrique de Guzman covering Lot 5249, Ts-217, situated at Dadiangas, General Santos City, and TCT No. T-7203 in the name of Rio Rivera and Carolina de Guzman and the Certificate of Sale executed by the City Sheriff, General Santos City, dated October 29, 1977, in favor (sic) Hoechst Philippines, Inc., null and void;

2. Ordering Rio Rivera and Carolina de Guzman and/or Hoechst Philippines, Inc., to Surrender to the Register of Deeds, General Santos City, the Owner's Duplicate of said TCT No. T-7203 or any subsequent transfer certificate of title

Page 21: Natural Resources Set2.Classification Cases

21NATURAL RESOURCES: CLASSIFICATIONS CASES

issued in the name of Hoechst Philippines, Inc. if any, and once surrendered, ordering the Register of Deeds, General Santos City, to cancel the owner's duplicate original of said Title No. P-29712 in the name of Enrique de Guzman and owner's duplicate of TCT No. T-2703 in the name of Rio Rivera and Carolina de Guzman on file with this (Register of Deeds) Office;

3. Ordering the Register of Deeds, General Santos City, to cancel OCT No. P-29712 and TCT No. T-7203 in the name of Enrique de Guzman and Rio Rivera, respectively, and all other subsequent transfer certificate of titles derived therefrom should said defendants Enrique P. de Guzman, Rio Rivera and Carolina de Guzman or Hoechst Philippines, Inc. fail to surrender their respective certificate of titles over Lot No. 5249, Ts-217;

4. Ordering the reversion of Lot No. 5249, Ts-217 situated at Dadiangas, General Santos City, to the mass of public domain, subject to the administration and disposition of the Director of Lands or the Board of Liquidators as the case maybe, giving preference to qualified and actual occupant; and

5. Dismissing the counterclaim with costs against the defendants.SO ORDERED.General Santos City, July 9, 1987.(s/t) ABEDNEGO O. ADRE Judge

On November 10, 1988, spouses Rivera appealed the decision to the Court of Appeals.

On May 25, 1992, the Court of Appeals rendered decision reversing the decision of the trial court. The Court of Appeals ruled that when Enrique P. de Guzman obtained Original Certificate of Title No. P-29712, the land ceased to be part of the public domain. The land became registered under the Torrens system, converted into a private registered land, and governed by the Property Registration Decree (P.D. 1529). Being a private land, the Director of Lands had neither control nor jurisdiction over the land. Furthermore, the title became indefeasible after the expiration of one (1) year from issuance thereof.

The Court of Appeals found that no fraud attended the issuance of the patent and Original Certificate of Title to de Guzman. It stated that the discrepancy in the name Diono and Diones appearing in the records was a mere typographical error.

The appellate court gave little credit to the investigation report relied upon by the trial court. It held that no other evidence, whether testimonial or documentary, was presented to prove that the documents presented by de Guzman were issued by fictitious persons or entirely fabricated.

However, the Court of Appeals sustained the finding of the trial court that Enrique P. de Guzman and spouses Rivera were not in possession of the property. The Court of Appeals concluded that de Guzman misrepresented facts in his application since he was not the possessor at the time he applied for the sales patent. Nonetheless, the Court of Appeals ruled that title founded on fraud or misrepresentation could not be assailed since more than one year had lapsed from the issuance of the public land patent.

At any rate, the Court of Appeals held that the land has passed to innocent purchasers for value, namely, spouses Rivera. The Court of Appeals argued that mere relationship to de Guzman, without any other proof of bad faith on the part of spouses Rivera, did not dispute the presumption that they were innocent purchasers for value.

On August 19, 1992, the Republic of the Philippines filed with this Court, a petition for review on certiorari of the Court of Appeals' decision.

On February 17, 1993, we gave due course to the petition and required the parties to file their respective memoranda. The parties have complied.

Petitioner Republic of the Philippines contends that Enrique de Guzman obtained the Miscellaneous Sales Patent No. 814 and OCT No. P-29712 through fraudulent means. Petitioner avers that the supporting documents submitted by de Guzman together with his sales application, were either fabricated or issued by fictitious persons. Thus, both the sales application and original certificate of title issued in favor of Enrique P. de Guzman were null and void. Petitioner avers that respondent spouses Rivera being related by consanguinity and affinity to de Guzman were not innocent purchasers for value.

We grant the petition.

Initially, we resolve the question of whether or not the Director of Lands loses authority over the land the moment an original certificate of title is issued covering the same. The Court of Appeals ruled that the issuance of the original certificate of title converted the lot into a private land, thereby placing it beyond the authority of the Director of Lands.

We disagree. The authority of the Director of Lands to investigate conflicts over public lands is derived from Section 91 of the Public Land Act. In fact, it is not merely his right but his specific duty to conduct investigations of alleged fraud in securing patents and the corresponding titles thereto. While title issued on the basis of a patent is as indefeasible as one judicially secured, such indefeasibility is not a bar to an investigation by the Director of Lands as to how such title had been

Page 22: Natural Resources Set2.Classification Cases

22NATURAL RESOURCES: CLASSIFICATIONS CASES

acquired, if the purpose of such investigation is to determine whether or not fraud had been committed in securing such title, in order that the appropriate action for reversion may be filed by the Government.

The next issue to determine is whether or not Enrique P. de Guzman validly obtained the sales patent and the original certificate of title.

We rule in the negative. There is no question that de Guzman was not in possession of the property. Hence, de Guzman misrepresented facts in his application for sales patent. Even the Court of Appeals sustained the factual finding of the trial court on this point. However, the Court of Appeals held that an action for cancellation of patent or title could not be maintained after the lapse of one year from the date of issuance thereof. As heretofore stated, the ruling is erroneous.

The next issue is whether or not the validity of the patent and the original certificate of title can still be assailed after the lapse of one year from the issuance of the disputed title.

We rule that the State can assail a patent fraudulently issued by the Director of Lands. "Where public land is acquired by an applicant through fraud and misrepresentation, the State may institute reversion proceedings even after the lapse of one year." "The indefeasibility of a title does not attach to titles secured by fraud and misrepresentation."

The last issue to resolve is whether the spouses Rivera are innocent purchasers for value.

We agree with the trial court that spouses Rivera are not innocent purchasers for value. Spouses Rivera are related by consanguinity and affinity to Enrique P. de Guzman knew that de Guzman was not in possession of the land. In fact, Rio Rivera testified that his father-in-law was not in possession of the lot in question. Carmen Ty was in possession of the land since 1963 and paid the real estate taxes thereon. We do not agree with the Court of Appeals that the presumption of a buyer in good faith must prevail. "The burden of proving the status of a purchaser in good faith and for value lies upon him who asserts that status. In discharging the burden, it is not enough to invoke the ordinary presumption of good faith." "The rule is settled that a buyer of real property which is in the possession of persons other than the seller must be wary and should investigate the rights of those in possession. Otherwise, without such inquiry, the buyer can hardly be regarded as buyer in good faith."

"A purchaser or mortgagee cannot close his eyes to facts which should put a reasonable man upon his guard, and then claim that he acted in good faith under the

belief that there was no defect in the title of the vendor or mortgagor. His mere refusal to believe that such defect exists, or his willful closing of his eyes to the possibility of the existence of a defect in the vendors or mortgagor's title, will not make him an innocent purchaser or mortgagee for value, if it afterwards develops that the title was in fact defective, and it appears that he had such notice of the defects as would have led to its discovery had he acted with the measure of precaution which may be required of a prudent man in a like situation."

WHEREFORE, we GRANT the petition and REVERSE the decision of the Court of Appeals. We declare Miscellaneous Sales Patent No. 814 and Original Certificate of Title No. P-29712 in the name of Enrique P. de Guzman, and Transfer Certificate of Title No. T-7203 in the name of spouses Rio Rivera and Carolina R. de Guzman, and all subsequent transfer certificates of title derived therefrom, as null and void. We order the reversion to the mass of public domain of Lot 5249, Ts-217, located in Dadiangas, General Santos City. No costs. SO ORDERED.

G.R. No. 133250           July 9, 2002FRANCISCO I. CHAVEZ vs. PUBLIC ESTATES AUTHORITY 9

CARPIO, J.:

This is an original Petition for Mandamus with prayer for a writ of preliminary injunction and a temporary restraining order. The petition seeks to compel the Public Estates Authority ("PEA" for brevity) to disclose all facts on PEA's then on-going renegotiations with Amari Coastal Bay and Development Corporation ("AMARI" for brevity) to reclaim portions of Manila Bay. The petition further seeks to enjoin PEA from signing a new agreement with AMARI involving such reclamation.

The Facts

On November 20, 1973, the government, through the Commissioner of Public Highways, signed a contract with the Construction and Development Corporation of the Philippines ("CDCP" for brevity) to reclaim certain foreshore and offshore areas of Manila Bay. The contract also included the construction of Phases I and II of the Manila-Cavite Coastal Road. CDCP obligated itself to carry out all the works in consideration of fifty percent of the total reclaimed land.

On February 4, 1977, then President Ferdinand E. Marcos issued Presidential Decree No. 1084 creating PEA. PD No. 1084 tasked PEA "to reclaim land, including foreshore and submerged areas," and "to develop, improve, acquire, x x x lease and sell any and all kinds of lands." On the same date, then President Marcos issued Presidential Decree No. 1085 transferring to PEA the "lands reclaimed

9 Classifications

Page 23: Natural Resources Set2.Classification Cases

23NATURAL RESOURCES: CLASSIFICATIONS CASES

in the foreshore and offshore of the Manila Bay" under the Manila-Cavite Coastal Road and Reclamation Project (MCCRRP).

On December 29, 1981, then President Marcos issued a memorandum directing PEA to amend its contract with CDCP, so that "[A]ll future works in MCCRRP x x x shall be funded and owned by PEA." Accordingly, PEA and CDCP executed a Memorandum of Agreement dated December 29, 1981, which stated:

"(i) CDCP shall undertake all reclamation, construction, and such other works in the MCCRRP as may be agreed upon by the parties, to be paid according to progress of works on a unit price/lump sum basis for items of work to be agreed upon, subject to price escalation, retention and other terms and conditions provided for in Presidential Decree No. 1594. All the financing required for such works shall be provided by PEA.x x x(iii) x x x CDCP shall give up all its development rights and hereby agrees to cede and transfer in favor of PEA, all of the rights, title, interest and participation of CDCP in and to all the areas of land reclaimed by CDCP in the MCCRRP as of December 30, 1981 which have not yet been sold, transferred or otherwise disposed of by CDCP as of said date, which areas consist of approximately Ninety-Nine Thousand Four Hundred Seventy Three (99,473) square meters in the Financial Center Area covered by land pledge No. 5 and approximately Three Million Three Hundred Eighty Two Thousand Eight Hundred Eighty Eight (3,382,888) square meters of reclaimed areas at varying elevations above Mean Low Water Level located outside the Financial Center Area and the First Neighborhood Unit."

On January 19, 1988, then President Corazon C. Aquino issued Special Patent No. 3517, granting and transferring to PEA "the parcels of land so reclaimed under the Manila-Cavite Coastal Road and Reclamation Project (MCCRRP) containing a total area of one million nine hundred fifteen thousand eight hundred ninety four (1,915,894) square meters." Subsequently, on April 9, 1988, the Register of Deeds of the Municipality of Parañaque issued Transfer Certificates of Title Nos. 7309, 7311, and 7312, in the name of PEA, covering the three reclaimed islands known as the "Freedom Islands" located at the southern portion of the Manila-Cavite Coastal Road, Parañaque City. The Freedom Islands have a total land area of One Million Five Hundred Seventy Eight Thousand Four Hundred and Forty One (1,578,441) square meters or 157.841 hectares.

On April 25, 1995, PEA entered into a Joint Venture Agreement ("JVA" for brevity) with AMARI, a private corporation, to develop the Freedom Islands. The JVA also required the reclamation of an additional 250 hectares of submerged areas surrounding these islands to complete the configuration in the Master Development Plan of the Southern Reclamation Project-MCCRRP. PEA and AMARI entered into

the JVA through negotiation without public bidding. On April 28, 1995, the Board of Directors of PEA, in its Resolution No. 1245, confirmed the JVA. On June 8, 1995, then President Fidel V. Ramos, through then Executive Secretary Ruben Torres, approved the JVA.

On November 29, 1996, then Senate President Ernesto Maceda delivered a privilege speech in the Senate and denounced the JVA as the "grandmother of all scams." As a result, the Senate Committee on Government Corporations and Public Enterprises, and the Committee on Accountability of Public Officers and Investigations, conducted a joint investigation. The Senate Committees reported the results of their investigation in Senate Committee Report No. 560 dated September 16, 1997.7 Among the conclusions of their report are: (1) the reclaimed lands PEA seeks to transfer to AMARI under the JVA are lands of the public domain which the government has not classified as alienable lands and therefore PEA cannot alienate these lands; (2) the certificates of title covering the Freedom Islands are thus void, and (3) the JVA itself is illegal.

On December 5, 1997, then President Fidel V. Ramos issued Presidential Administrative Order No. 365 creating a Legal Task Force to conduct a study on the legality of the JVA in view of Senate Committee Report No. 560. The members of the Legal Task Force were the Secretary of Justice, the Chief Presidential Legal Counsel,and the Government Corporate Counsel. The Legal Task Force upheld the legality of the JVA, contrary to the conclusions reached by the Senate Committees.

On April 4 and 5, 1998, the Philippine Daily Inquirer and Today published reports that there were on-going renegotiations between PEA and AMARI under an order issued by then President Fidel V. Ramos. According to these reports, PEA Director Nestor Kalaw, PEA Chairman Arsenio Yulo and retired Navy Officer Sergio Cruz composed the negotiating panel of PEA.

On April 13, 1998, Antonio M. Zulueta filed before the Court a Petition for Prohibition with Application for the Issuance of a Temporary Restraining Order and Preliminary Injunction docketed as G.R. No. 132994 seeking to nullify the JVA. The Court dismissed the petition "for unwarranted disregard of judicial hierarchy, without prejudice to the refiling of the case before the proper court."

On April 27, 1998, petitioner Frank I. Chavez ("Petitioner" for brevity) as a taxpayer, filed the instant Petition for Mandamus with Prayer for the Issuance of a Writ of Preliminary Injunction and Temporary Restraining Order. Petitioner contends the government stands to lose billions of pesos in the sale by PEA of the reclaimed lands to AMARI. Petitioner prays that PEA publicly disclose the terms of any renegotiation of the JVA, invoking Section 28, Article II, and Section 7, Article III, of the 1987 Constitution on the right of the people to information on matters of public concern. Petitioner assails the sale to AMARI of lands of the public domain as a

Page 24: Natural Resources Set2.Classification Cases

24NATURAL RESOURCES: CLASSIFICATIONS CASES

blatant violation of Section 3, Article XII of the 1987 Constitution prohibiting the sale of alienable lands of the public domain to private corporations. Finally, petitioner asserts that he seeks to enjoin the loss of billions of pesos in properties of the State that are of public dominion.

After several motions for extension of time, PEA and AMARI filed their Comments on October 19, 1998 and June 25, 1998, respectively. Meanwhile, on December 28, 1998, petitioner filed an Omnibus Motion: (a) to require PEA to submit the terms of the renegotiated PEA-AMARI contract; (b) for issuance of a temporary restraining order; and (c) to set the case for hearing on oral argument. Petitioner filed a Reiterative Motion for Issuance of a TRO dated May 26, 1999, which the Court denied in a Resolution dated June 22, 1999.

In a Resolution dated March 23, 1999, the Court gave due course to the petition and required the parties to file their respective memoranda.

On March 30, 1999, PEA and AMARI signed the Amended Joint Venture Agreement ("Amended JVA," for brevity). On May 28, 1999, the Office of the President under the administration of then President Joseph E. Estrada approved the Amended JVA.

Due to the approval of the Amended JVA by the Office of the President, petitioner now prays that on "constitutional and statutory grounds the renegotiated contract be declared null and void."

The Issues

The issues raised by petitioner, PEA and AMARI are as follows:

I. WHETHER THE PRINCIPAL RELIEFS PRAYED FOR IN THE PETITION ARE MOOT AND ACADEMIC BECAUSE OF SUBSEQUENT EVENTS;

II. WHETHER THE PETITION MERITS DISMISSAL FOR FAILING TO OBSERVE THE PRINCIPLE GOVERNING THE HIERARCHY OF COURTS;III. WHETHER THE PETITION MERITS DISMISSAL FOR NON-EXHAUSTION OF ADMINISTRATIVE REMEDIES;IV. WHETHER PETITIONER HAS LOCUS STANDI TO BRING THIS SUIT;V. WHETHER THE CONSTITUTIONAL RIGHT TO INFORMATION INCLUDES OFFICIAL INFORMATION ON ON-GOING NEGOTIATIONS BEFORE A FINAL AGREEMENT;

VI. WHETHER THE STIPULATIONS IN THE AMENDED JOINT VENTURE AGREEMENT FOR THE TRANSFER TO AMARI OF CERTAIN LANDS,

RECLAIMED AND STILL TO BE RECLAIMED, VIOLATE THE 1987 CONSTITUTION; AND

VII. WHETHER THE COURT IS THE PROPER FORUM FOR RAISING THE ISSUE OF WHETHER THE AMENDED JOINT VENTURE AGREEMENT IS GROSSLY DISADVANTAGEOUS TO THE GOVERNMENT.

The Court's Ruling

First issue: whether the principal reliefs prayed for in the petition are moot and academic because of subsequent events.

The petition prays that PEA publicly disclose the "terms and conditions of the on-going negotiations for a new agreement." The petition also prays that the Court enjoin PEA from "privately entering into, perfecting and/or executing any new agreement with AMARI."

PEA and AMARI claim the petition is now moot and academic because AMARI furnished petitioner on June 21, 1999 a copy of the signed Amended JVA containing the terms and conditions agreed upon in the renegotiations. Thus, PEA has satisfied petitioner's prayer for a public disclosure of the renegotiations. Likewise, petitioner's prayer to enjoin the signing of the Amended JVA is now moot because PEA and AMARI have already signed the Amended JVA on March 30, 1999. Moreover, the Office of the President has approved the Amended JVA on May 28, 1999.

Petitioner counters that PEA and AMARI cannot avoid the constitutional issue by simply fast-tracking the signing and approval of the Amended JVA before the Court could act on the issue. Presidential approval does not resolve the constitutional issue or remove it from the ambit of judicial review.

We rule that the signing of the Amended JVA by PEA and AMARI and its approval by the President cannot operate to moot the petition and divest the Court of its jurisdiction. PEA and AMARI have still to implement the Amended JVA. The prayer to enjoin the signing of the Amended JVA on constitutional grounds necessarily includes preventing its implementation if in the meantime PEA and AMARI have signed one in violation of the Constitution. Petitioner's principal basis in assailing the renegotiation of the JVA is its violation of Section 3, Article XII of the Constitution, which prohibits the government from alienating lands of the public domain to private corporations. If the Amended JVA indeed violates the Constitution, it is the duty of the Court to enjoin its implementation, and if already implemented, to annul the effects of such unconstitutional contract.

Page 25: Natural Resources Set2.Classification Cases

25NATURAL RESOURCES: CLASSIFICATIONS CASES

The Amended JVA is not an ordinary commercial contract but one which seeks to transfer title and ownership to 367.5 hectares of reclaimed lands and submerged areas of Manila Bay to a single private corporation. It now becomes more compelling for the Court to resolve the issue to insure the government itself does not violate a provision of the Constitution intended to safeguard the national patrimony. Supervening events, whether intended or accidental, cannot prevent the Court from rendering a decision if there is a grave violation of the Constitution. In the instant case, if the Amended JVA runs counter to the Constitution, the Court can still prevent the transfer of title and ownership of alienable lands of the public domain in the name of AMARI. Even in cases where supervening events had made the cases moot, the Court did not hesitate to resolve the legal or constitutional issues raised to formulate controlling principles to guide the bench, bar, and the public.

Also, the instant petition is a case of first impression. All previous decisions of the Court involving Section 3, Article XII of the 1987 Constitution, or its counterpart provision in the 1973 Constitution, covered agricultural lands sold to private corporations which acquired the lands from private parties. The transferors of the private corporations claimed or could claim the right to judicial confirmation of their imperfect titles under Title IIof Commonwealth Act. 141 ("CA No. 141" for brevity). In the instant case, AMARI seeks to acquire from PEA, a public corporation, reclaimed lands and submerged areas for non-agricultural purposes by purchase under PD No. 1084 (charter of PEA) and Title III of CA No. 141. Certain undertakings by AMARI under the Amended JVA constitute the consideration for the purchase. Neither AMARI nor PEA can claim judicial confirmation of their titles because the lands covered by the Amended JVA are newly reclaimed or still to be reclaimed. Judicial confirmation of imperfect title requires open, continuous, exclusive and notorious occupation of agricultural lands of the public domain for at least thirty years since June 12, 1945 or earlier. Besides, the deadline for filing applications for judicial confirmation of imperfect title expired on December 31, 1987.

Lastly, there is a need to resolve immediately the constitutional issue raised in this petition because of the possible transfer at any time by PEA to AMARI of title and ownership to portions of the reclaimed lands. Under the Amended JVA, PEA is obligated to transfer to AMARI the latter's seventy percent proportionate share in the reclaimed areas as the reclamation progresses. The Amended JVA even allows AMARI to mortgage at any time the entire reclaimed area to raise financing for the reclamation project.

Second issue: whether the petition merits dismissal for failing to observe the principle governing the hierarchy of courts.

PEA and AMARI claim petitioner ignored the judicial hierarchy by seeking relief directly from the Court. The principle of hierarchy of courts applies generally to cases involving factual questions. As it is not a trier of facts, the Court cannot

entertain cases involving factual issues. The instant case, however, raises constitutional issues of transcendental importance to the public.22 The Court can resolve this case without determining any factual issue related to the case. Also, the instant case is a petition for mandamus which falls under the original jurisdiction of the Court under Section 5, Article VIII of the Constitution. We resolve to exercise primary jurisdiction over the instant case.

Third issue: whether the petition merits dismissal for non-exhaustion of administrative remedies.

PEA faults petitioner for seeking judicial intervention in compelling PEA to disclose publicly certain information without first asking PEA the needed information. PEA claims petitioner's direct resort to the Court violates the principle of exhaustion of administrative remedies. It also violates the rule that mandamus may issue only if there is no other plain, speedy and adequate remedy in the ordinary course of law.

PEA distinguishes the instant case from Tañada v. Tuvera where the Court granted the petition for mandamus even if the petitioners there did not initially demand from the Office of the President the publication of the presidential decrees. PEA points out that in Tañada, the Executive Department had an affirmative statutoryduty under Article 2 of the Civil Code and Section 1 of Commonwealth Act No. 638 to publish the presidential decrees. There was, therefore, no need for the petitioners in Tañada to make an initial demand from the Office of the President. In the instant case, PEA claims it has no affirmative statutory duty to disclose publicly information about its renegotiation of the JVA. Thus, PEA asserts that the Court must apply the principle of exhaustion of administrative remedies to the instant case in view of the failure of petitioner here to demand initially from PEA the needed information.

The original JVA sought to dispose to AMARI public lands held by PEA, a government corporation. Under Section 79 of the Government Auditing Code, the disposition of government lands to private parties requires public bidding. PEA was under a positive legal duty to disclose to the public the terms and conditions for the sale of its lands. The law obligated PEA to make this public disclosure even without demand from petitioner or from anyone. PEA failed to make this public disclosure because the original JVA, like the Amended JVA, was the result of a negotiated contract, not of a public bidding. Considering that PEA had an affirmative statutory duty to make the public disclosure, and was even in breach of this legal duty, petitioner had the right to seek direct judicial intervention.

Moreover, and this alone is determinative of this issue, the principle of exhaustion of administrative remedies does not apply when the issue involved is a purely legal or constitutional question. The principal issue in the instant case is the capacity of AMARI to acquire lands held by PEA in view of the constitutional ban prohibiting the alienation of lands of the public domain to private corporations. We

Page 26: Natural Resources Set2.Classification Cases

26NATURAL RESOURCES: CLASSIFICATIONS CASES

rule that the principle of exhaustion of administrative remedies does not apply in the instant case.

Fourth issue: whether petitioner has locus standi to bring this suit

PEA argues that petitioner has no standing to institute mandamus proceedings to enforce his constitutional right to information without a showing that PEA refused to perform an affirmative duty imposed on PEA by the Constitution. PEA also claims that petitioner has not shown that he will suffer any concrete injury because of the signing or implementation of the Amended JVA. Thus, there is no actual controversy requiring the exercise of the power of judicial review.

The petitioner has standing to bring this taxpayer's suit because the petition seeks to compel PEA to comply with its constitutional duties. There are two constitutional issues involved here. First is the right of citizens to information on matters of public concern. Second is the application of a constitutional provision intended to insure the equitable distribution of alienable lands of the public domain among Filipino citizens. The thrust of the first issue is to compel PEA to disclose publicly information on the sale of government lands worth billions of pesos, information which the Constitution and statutory law mandate PEA to disclose. The thrust of the second issue is to prevent PEA from alienating hundreds of hectares of alienable lands of the public domain in violation of the Constitution, compelling PEA to comply with a constitutional duty to the nation.

Moreover, the petition raises matters of transcendental importance to the public. In Chavez v. PCGG, the Court upheld the right of a citizen to bring a taxpayer's suit on matters of transcendental importance to the public, thus -

"Besides, petitioner emphasizes, the matter of recovering the ill-gotten wealth of the Marcoses is an issue of 'transcendental importance to the public.' He asserts that ordinary taxpayers have a right to initiate and prosecute actions questioning the validity of acts or orders of government agencies or instrumentalities, if the issues raised are of 'paramount public interest,' and if they 'immediately affect the social, economic and moral well being of the people.'

Moreover, the mere fact that he is a citizen satisfies the requirement of personal interest, when the proceeding involves the assertion of a public right, such as in this case. He invokes several decisions of this Court which have set aside the procedural matter of locus standi, when the subject of the case involved public interest.x x x

In Tañada v. Tuvera, the Court asserted that when the issue concerns a public right and the object of mandamus is to obtain the enforcement of a public duty, the people are regarded as the real parties in interest; and because it is

sufficient that petitioner is a citizen and as such is interested in the execution of the laws, he need not show that he has any legal or special interest in the result of the action. In the aforesaid case, the petitioners sought to enforce their right to be informed on matters of public concern, a right then recognized in Section 6, Article IV of the 1973 Constitution, in connection with the rule that laws in order to be valid and enforceable must be published in the Official Gazette or otherwise effectively promulgated. In ruling for the petitioners' legal standing, the Court declared that the right they sought to be enforced 'is a public right recognized by no less than the fundamental law of the land.'

Legaspi v. Civil Service Commission, while reiterating Tañada, further declared that 'when a mandamus proceeding involves the assertion of a public right, the requirement of personal interest is satisfied by the mere fact that petitioner is a citizen and, therefore, part of the general 'public' which possesses the right.'

Further, in Albano v. Reyes, we said that while expenditure of public funds may not have been involved under the questioned contract for the development, management and operation of the Manila International Container Terminal, 'public interest [was] definitely involved considering the important role [of the subject contract] . . . in the economic development of the country and the magnitude of the financial consideration involved.' We concluded that, as a consequence, the disclosure provision in the Constitution would constitute sufficient authority for upholding the petitioner's standing.

Similarly, the instant petition is anchored on the right of the people to information and access to official records, documents and papers — a right guaranteed under Section 7, Article III of the 1987 Constitution. Petitioner, a former solicitor general, is a Filipino citizen. Because of the satisfaction of the two basic requisites laid down by decisional law to sustain petitioner's legal standing, i.e. (1) the enforcement of a public right (2) espoused by a Filipino citizen, we rule that the petition at bar should be allowed."

We rule that since the instant petition, brought by a citizen, involves the enforcement of constitutional rights - to information and to the equitable diffusion of natural resources - matters of transcendental public importance, the petitioner has the requisite locus standi.

Fifth issue: whether the constitutional right to information includes official information on on-going negotiations before a final agreement.

Section 7, Article III of the Constitution explains the people's right to information on matters of public concern in this manner:

"Sec. 7. The right of the people to information on matters of public concern shall be recognized. Access to official records, and to documents, and papers pertaining to official acts, transactions, or decisions, as well as to

Page 27: Natural Resources Set2.Classification Cases

27NATURAL RESOURCES: CLASSIFICATIONS CASES

government research data used as basis for policy development, shall be afforded the citizen, subject to such limitations as may be provided by law." (Emphasis supplied)

The State policy of full transparency in all transactions involving public interest reinforces the people's right to information on matters of public concern. This State policy is expressed in Section 28, Article II of the Constitution, thus:

"Sec. 28. Subject to reasonable conditions prescribed by law, the State adopts and implements a policy of full public disclosure of all its transactions involving public interest." (Emphasis supplied)

These twin provisions of the Constitution seek to promote transparency in policy-making and in the operations of the government, as well as provide the people sufficient information to exercise effectively other constitutional rights. These twin provisions are essential to the exercise of freedom of expression. If the government does not disclose its official acts, transactions and decisions to citizens, whatever citizens say, even if expressed without any restraint, will be speculative and amount to nothing. These twin provisions are also essential to hold public officials "at all times x x x accountable to the people," for unless citizens have the proper information, they cannot hold public officials accountable for anything. Armed with the right information, citizens can participate in public discussions leading to the formulation of government policies and their effective implementation. An informed citizenry is essential to the existence and proper functioning of any democracy. As explained by the Court in Valmonte v. Belmonte, Jr. –

"An essential element of these freedoms is to keep open a continuing dialogue or process of communication between the government and the people. It is in the interest of the State that the channels for free political discussion be maintained to the end that the government may perceive and be responsive to the people's will. Yet, this open dialogue can be effective only to the extent that the citizenry is informed and thus able to formulate its will intelligently. Only when the participants in the discussion are aware of the issues and have access to information relating thereto can such bear fruit."

PEA asserts, citing Chavez v. PCGG, that in cases of on-going negotiations the right to information is limited to "definite propositions of the government." PEA maintains the right does not include access to "intra-agency or inter-agency recommendations or communications during the stage when common assertions are still in the process of being formulated or are in the 'exploratory stage'."

Also, AMARI contends that petitioner cannot invoke the right at the pre-decisional stage or before the closing of the transaction. To support its contention, AMARI cites the following discussion in the 1986 Constitutional Commission:

"Mr. Suarez. And when we say 'transactions' which should be distinguished from contracts, agreements, or treaties or whatever, does the Gentleman refer to the steps leading to the consummation of the contract, or does he refer to the contract itself?Mr. Ople: The 'transactions' used here, I suppose is generic and therefore, it can cover both steps leading to a contract and already a consummated contract, Mr. Presiding Officer.Mr. Suarez: This contemplates inclusion of negotiations leading to the consummation of the transaction.Mr. Ople: Yes, subject only to reasonable safeguards on the national interest.Mr. Suarez: Thank you." (Emphasis supplied)

AMARI argues there must first be a consummated contract before petitioner can invoke the right. Requiring government officials to reveal their deliberations at the pre-decisional stage will degrade the quality of decision-making in government agencies. Government officials will hesitate to express their real sentiments during deliberations if there is immediate public dissemination of their discussions, putting them under all kinds of pressure before they decide.

We must first distinguish between information the law on public bidding requires PEA to disclose publicly, and information the constitutional right to information requires PEA to release to the public. Before the consummation of the contract, PEA must, on its own and without demand from anyone, disclose to the public matters relating to the disposition of its property. These include the size, location, technical description and nature of the property being disposed of, the terms and conditions of the disposition, the parties qualified to bid, the minimum price and similar information. PEA must prepare all these data and disclose them to the public at the start of the disposition process, long before the consummation of the contract, because the Government Auditing Code requires public bidding. If PEA fails to make this disclosure, any citizen can demand from PEA this information at any time during the bidding process.

Information, however, on on-going evaluation or review of bids or proposals being undertaken by the bidding or review committee is not immediately accessible under the right to information. While the evaluation or review is still on-going, there are no "official acts, transactions, or decisions" on the bids or proposals. However, once the committee makes its official recommendation, there arises a "definite proposition" on the part of the government. From this moment, the public's right to

Page 28: Natural Resources Set2.Classification Cases

28NATURAL RESOURCES: CLASSIFICATIONS CASES

information attaches, and any citizen can access all the non-proprietary information leading to such definite proposition. In Chavez v. PCGG,33 the Court ruled as follows:

"Considering the intent of the framers of the Constitution, we believe that it is incumbent upon the PCGG and its officers, as well as other government representatives, to disclose sufficient public information on any proposed settlement they have decided to take up with the ostensible owners and holders of ill-gotten wealth. Such information, though, must pertain to definite propositions of the government, not necessarily to intra-agency or inter-agency recommendations or communications during the stage when common assertions are still in the process of being formulated or are in the "exploratory" stage. There is need, of course, to observe the same restrictions on disclosure of information in general, as discussed earlier – such as on matters involving national security, diplomatic or foreign relations, intelligence and other classified information." (Emphasis supplied)

Contrary to AMARI's contention, the commissioners of the 1986 Constitutional Commission understood that the right to information "contemplates inclusion of negotiations leading to the consummation of the transaction." Certainly, a consummated contract is not a requirement for the exercise of the right to information. Otherwise, the people can never exercise the right if no contract is consummated, and if one is consummated, it may be too late for the public to expose its defects.1âwphi1.nêt

Requiring a consummated contract will keep the public in the dark until the contract, which may be grossly disadvantageous to the government or even illegal, becomes a fait accompli. This negates the State policy of full transparency on matters of public concern, a situation which the framers of the Constitution could not have intended. Such a requirement will prevent the citizenry from participating in the public discussion of anyproposed contract, effectively truncating a basic right enshrined in the Bill of Rights. We can allow neither an emasculation of a constitutional right, nor a retreat by the State of its avowed "policy of full disclosure of all its transactions involving public interest."

The right covers three categories of information which are "matters of public concern," namely: (1) official records; (2) documents and papers pertaining to official acts, transactions and decisions; and (3) government research data used in formulating policies. The first category refers to any document that is part of the public records in the custody of government agencies or officials. The second category refers to documents and papers recording, evidencing, establishing, confirming, supporting, justifying or explaining official acts, transactions or decisions of government agencies or officials. The third category refers to research data, whether raw, collated or processed, owned by the government and used in formulating government policies.

The information that petitioner may access on the renegotiation of the JVA includes evaluation reports, recommendations, legal and expert opinions, minutes of meetings, terms of reference and other documents attached to such reports or minutes, all relating to the JVA. However, the right to information does not compel PEA to prepare lists, abstracts, summaries and the like relating to the renegotiation of the JVA. The right only affords access to records, documents and papers, which means the opportunity to inspect and copy them. One who exercises the right must copy the records, documents and papers at his expense. The exercise of the right is also subject to reasonable regulations to protect the integrity of the public records and to minimize disruption to government operations, like rules specifying when and how to conduct the inspection and copying.

The right to information, however, does not extend to matters recognized as privileged information under the separation of powers. The right does not also apply to information on military and diplomatic secrets, information affecting national security, and information on investigations of crimes by law enforcement agencies before the prosecution of the accused, which courts have long recognized as confidential. The right may also be subject to other limitations that Congress may impose by law.

There is no claim by PEA that the information demanded by petitioner is privileged information rooted in the separation of powers. The information does not cover Presidential conversations, correspondences, or discussions during closed-door Cabinet meetings which, like internal deliberations of the Supreme Court and other collegiate courts, or executive sessions of either house of Congress, are recognized as confidential. This kind of information cannot be pried open by a co-equal branch of government. A frank exchange of exploratory ideas and assessments, free from the glare of publicity and pressure by interested parties, is essential to protect the independence of decision-making of those tasked to exercise Presidential, Legislative and Judicial power. This is not the situation in the instant case.

We rule, therefore, that the constitutional right to information includes official information on on-going negotiations before a final contract. The information, however, must constitute definite propositions by the government and should not cover recognized exceptions like privileged information, military and diplomatic secrets and similar matters affecting national security and public order. Congress has also prescribed other limitations on the right to information in several legislations.41

Sixth issue: whether stipulations in the Amended JVA for the transfer to AMARI of lands, reclaimed or to be reclaimed, violate the Constitution.

The Regalian Doctrine

Page 29: Natural Resources Set2.Classification Cases

29NATURAL RESOURCES: CLASSIFICATIONS CASES

The ownership of lands reclaimed from foreshore and submerged areas is rooted in the Regalian doctrine which holds that the State owns all lands and waters of the public domain. Upon the Spanish conquest of the Philippines, ownership of all "lands, territories and possessions" in the Philippines passed to the Spanish Crown.42 The King, as the sovereign ruler and representative of the people, acquired and owned all lands and territories in the Philippines except those he disposed of by grant or sale to private individuals.

The 1935, 1973 and 1987 Constitutions adopted the Regalian doctrine substituting, however, the State, in lieu of the King, as the owner of all lands and waters of the public domain. The Regalian doctrine is the foundation of the time-honored principle of land ownership that "all lands that were not acquired from the Government, either by purchase or by grant, belong to the public domain."43 Article 339 of the Civil Code of 1889, which is now Article 420 of the Civil Code of 1950, incorporated the Regalian doctrine.

Ownership and Disposition of Reclaimed Lands

The Spanish Law of Waters of 1866 was the first statutory law governing the ownership and disposition of reclaimed lands in the Philippines. On May 18, 1907, the Philippine Commission enacted Act No. 1654 which provided for the lease, but not the sale, of reclaimed lands of the government to corporations and individuals . Later, on November 29, 1919, the Philippine Legislature approved Act No. 2874, the Public Land Act, which authorized the lease, but not the sale, of reclaimed lands of the government to corporations and individuals. On November 7, 1936, the National Assembly passed Commonwealth Act No. 141, also known as the Public Land Act, which authorized the lease, but not the sale, of reclaimed lands of the government to corporations and individuals. CA No. 141 continues to this day as the general law governing the classification and disposition of lands of the public domain.

The Spanish Law of Waters of 1866 and the Civil Code of 1889

Under the Spanish Law of Waters of 1866, the shores, bays, coves, inlets and all waters within the maritime zone of the Spanish territory belonged to the public domain for public use. The Spanish Law of Waters of 1866 allowed the reclamation of the sea under Article 5, which provided as follows:

"Article 5. Lands reclaimed from the sea in consequence of works constructed by the State, or by the provinces, pueblos or private persons, with proper permission, shall become the property of the party constructing such works, unless otherwise provided by the terms of the grant of authority."

Under the Spanish Law of Waters, land reclaimed from the sea belonged to the party undertaking the reclamation, provided the government issued the necessary permit and did not reserve ownership of the reclaimed land to the State.

Article 339 of the Civil Code of 1889 defined property of public dominion as follows:

"Art. 339. Property of public dominion is –1. That devoted to public use, such as roads, canals, rivers, torrents, ports and bridges constructed by the State, riverbanks, shores, roadsteads, and that of a similar character;2. That belonging exclusively to the State which, without being of general public use, is employed in some public service, or in the development of the national wealth, such as walls, fortresses, and other works for the defense of the territory, and mines, until granted to private individuals."

Property devoted to public use referred to property open for use by the public. In contrast, property devoted to public service referred to property used for some specific public service and open only to those authorized to use the property.

Property of public dominion referred not only to property devoted to public use, but also to property not so used but employed to develop the national wealth. This class of property constituted property of public dominion although employed for some economic or commercial activity to increase the national wealth.

Article 341 of the Civil Code of 1889 governed the re-classification of property of public dominion into private property, to wit:

"Art. 341. Property of public dominion, when no longer devoted to public use or to the defense of the territory, shall become a part of the private property of the State."

This provision, however, was not self-executing. The legislature, or the executive department pursuant to law, must declare the property no longer needed for public use or territorial defense before the government could lease or alienate the property to private parties.

Act No. 1654 of the Philippine Commission

On May 8, 1907, the Philippine Commission enacted Act No. 1654 which regulated the lease of reclaimed and foreshore lands. The salient provisions of this law were as follows:

Page 30: Natural Resources Set2.Classification Cases

30NATURAL RESOURCES: CLASSIFICATIONS CASES

"Section 1. The control and disposition of the foreshore as defined in existing law, and the title to all Government or public lands made or reclaimed by the Government by dredging or filling or otherwise throughout the Philippine Islands, shall be retained by the Government without prejudice to vested rights and without prejudice to rights conceded to the City of Manila in the Luneta Extension.Section 2. (a) The Secretary of the Interior shall cause all Government or public lands made or reclaimed by the Government by dredging or filling or otherwise to be divided into lots or blocks, with the necessary streets and alleyways located thereon, and shall cause plats and plans of such surveys to be prepared and filed with the Bureau of Lands.(b) Upon completion of such plats and plans the Governor-General shall give notice to the public that such parts of the lands so made or reclaimed as are not needed for public purposes will be leased for commercial and business purposes, x x x.x x x(e) The leases above provided for shall be disposed of to the highest and best bidder therefore, subject to such regulations and safeguards as the Governor-General may by executive order prescribe." (Emphasis supplied)

Act No. 1654 mandated that the government should retain title to all lands reclaimed by the government. The Act also vested in the government control and disposition of foreshore lands. Private parties could lease lands reclaimed by the government only if these lands were no longer needed for public purpose. Act No. 1654 mandated public bidding in the lease of government reclaimed lands. Act No. 1654 made government reclaimed lands sui generis in that unlike other public lands which the government could sell to private parties, these reclaimed lands were available only for lease to private parties.

Act No. 1654, however, did not repeal Section 5 of the Spanish Law of Waters of 1866. Act No. 1654 did not prohibit private parties from reclaiming parts of the sea under Section 5 of the Spanish Law of Waters. Lands reclaimed from the sea by private parties with government permission remained private lands.

Act No. 2874 of the Philippine Legislature

On November 29, 1919, the Philippine Legislature enacted Act No. 2874, the Public Land Act. The salient provisions of Act No. 2874, on reclaimed lands, were as follows:

"Sec. 6. The Governor-General, upon the recommendation of the Secretary of Agriculture and Natural Resources, shall from time to time classify the lands of the public domain into –

(a) Alienable or disposable,

(b) Timber, and(c) Mineral lands, x x x.

Sec. 7. For the purposes of the government and disposition of alienable or disposable public lands, the Governor-General, upon recommendation by the Secretary of Agriculture and Natural Resources, shall from time to time declare what lands are open to disposition or concession under this Act."

Sec. 8. Only those lands shall be declared open to disposition or concession which have been officially delimited or classified x x x.

x x xSec. 55. Any tract of land of the public domain which, being neither timber nor mineral land, shall be classified as suitable for residential purposes or for commercial, industrial, or other productive purposes other than agricultural purposes, and shall be open to disposition or concession, shall be disposed of under the provisions of this chapter, and not otherwise.Sec. 56. The lands disposable under this title shall be classified as follows:

(a) Lands reclaimed by the Government by dredging, filling, or other means;(b) Foreshore;(c) Marshy lands or lands covered with water bordering upon the shores or banks of navigable lakes or rivers;(d) Lands not included in any of the foregoing classes.

x x x.Sec. 58. The lands comprised in classes (a), (b), and (c) of section fifty-six shall be disposed of to private parties by lease only and not otherwise, as soon as the Governor-General, upon recommendation by the Secretary of Agriculture and Natural Resources, shall declare that the same are not necessary for the public service and are open to disposition under this chapter. The lands included in class (d) may be disposed of by sale or lease under the provisions of this Act." (Emphasis supplied)

Section 6 of Act No. 2874 authorized the Governor-General to "classify lands of the public domain into x x x alienable or disposable" lands. Section 7 of the Act empowered the Governor-General to "declare what lands are open to disposition or concession." Section 8 of the Act limited alienable or disposable lands only to those lands which have been "officially delimited and classified."

Section 56 of Act No. 2874 stated that lands "disposable under this title48 shall be classified" as government reclaimed, foreshore and marshy lands, as well as other lands. All these lands, however, must be suitable for residential, commercial, industrial or other productive non-agricultural purposes. These provisions vested upon the Governor-General the power to classify inalienable lands of the public domain into disposable lands of the public domain. These provisions also empowered

Page 31: Natural Resources Set2.Classification Cases

31NATURAL RESOURCES: CLASSIFICATIONS CASES

the Governor-General to classify further such disposable lands of the public domain into government reclaimed, foreshore or marshy lands of the public domain, as well as other non-agricultural lands.

Section 58 of Act No. 2874 categorically mandated that disposable lands of the public domain classified as government reclaimed, foreshore and marshy lands "shall be disposed of to private parties by lease only and not otherwise." The Governor-General, before allowing the lease of these lands to private parties, must formally declare that the lands were "not necessary for the public service." Act No. 2874 reiterated the State policy to lease and not to sell government reclaimed, foreshore and marshy lands of the public domain, a policy first enunciated in 1907 in Act No. 1654. Government reclaimed, foreshore and marshy lands remained sui generis, as the only alienable or disposable lands of the public domain that the government could not sell to private parties.

The rationale behind this State policy is obvious. Government reclaimed, foreshore and marshy public lands for non-agricultural purposes retain their inherent potential as areas for public service. This is the reason the government prohibited the sale, and only allowed the lease, of these lands to private parties. The State always reserved these lands for some future public service.

Act No. 2874 did not authorize the reclassification of government reclaimed, foreshore and marshy lands into other non-agricultural lands under Section 56 (d). Lands falling under Section 56 (d) were the only lands for non-agricultural purposes the government could sell to private parties. Thus, under Act No. 2874, the government could not sell government reclaimed, foreshore and marshy lands to private parties, unless the legislature passed a law allowing their sale.

Act No. 2874 did not prohibit private parties from reclaiming parts of the sea pursuant to Section 5 of the Spanish Law of Waters of 1866. Lands reclaimed from the sea by private parties with government permission remained private lands.

Dispositions under the 1935 Constitution

On May 14, 1935, the 1935 Constitution took effect upon its ratification by the Filipino people. The 1935 Constitution, in adopting the Regalian doctrine, declared in Section 1, Article XIII, that –

"Section 1. All agricultural, timber, and mineral lands of the public domain, waters, minerals, coal, petroleum, and other mineral oils, all forces of potential energy and other natural resources of the Philippines belong to the State, and their disposition, exploitation, development, or utilization shall be limited to citizens of the Philippines or to corporations or associations at least sixty per

centum of the capital of which is owned by such citizens, subject to any existing right, grant, lease, or concession at the time of the inauguration of the Government established under this Constitution. Natural resources, with the exception of public agricultural land, shall not be alienated, and no license, concession, or lease for the exploitation, development, or utilization of any of the natural resources shall be granted for a period exceeding twenty-five years, renewable for another twenty-five years, except as to water rights for irrigation, water supply, fisheries, or industrial uses other than the development of water power, in which cases beneficial use may be the measure and limit of the grant." (Emphasis supplied)

The 1935 Constitution barred the alienation of all natural resources except public agricultural lands, which were the only natural resources the State could alienate. Thus, foreshore lands, considered part of the State's natural resources, became inalienable by constitutional fiat, available only for lease for 25 years, renewable for another 25 years. The government could alienate foreshore lands only after these lands were reclaimed and classified as alienable agricultural lands of the public domain. Government reclaimed and marshy lands of the public domain, being neither timber nor mineral lands, fell under the classification of public agricultural lands. However, government reclaimed and marshy lands, although subject to classification as disposable public agricultural lands, could only be leased and not sold to private parties because of Act No. 2874.

The prohibition on private parties from acquiring ownership of government reclaimed and marshy lands of the public domain was only a statutory prohibition and the legislature could therefore remove such prohibition. The 1935 Constitution did not prohibit individuals and corporations from acquiring government reclaimed and marshy lands of the public domain that were classified as agricultural lands under existing public land laws. Section 2, Article XIII of the 1935 Constitution provided as follows:

"Section 2. No private corporation or association may acquire, lease, or hold public agricultural lands in excess of one thousand and twenty four hectares, nor may any individual acquire such lands by purchase in excess of one hundred and forty hectares, or by lease in excess of one thousand and twenty-four hectares, or by homestead in excess of twenty-four hectares. Lands adapted to grazing, not exceeding two thousand hectares, may be leased to an individual, private corporation, or association." (Emphasis supplied)

Still, after the effectivity of the 1935 Constitution, the legislature did not repeal Section 58 of Act No. 2874 to open for sale to private parties government reclaimed and marshy lands of the public domain. On the contrary, the legislature continued the long established State policy of retaining for the government title and ownership of government reclaimed and marshy lands of the public domain.

Page 32: Natural Resources Set2.Classification Cases

32NATURAL RESOURCES: CLASSIFICATIONS CASES

Commonwealth Act No. 141 of the Philippine National Assembly

On November 7, 1936, the National Assembly approved Commonwealth Act No. 141, also known as the Public Land Act, which compiled the then existing laws on lands of the public domain. CA No. 141, as amended, remains to this day the existing general law governing the classification and disposition of lands of the public domain other than timber and mineral lands.

Section 6 of CA No. 141 empowers the President to classify lands of the public domain into "alienable or disposable" lands of the public domain, which prior to such classification are inalienable and outside the commerce of man. Section 7 of CA No. 141 authorizes the President to "declare what lands are open to disposition or concession." Section 8 of CA No. 141 states that the government can declare open for disposition or concession only lands that are "officially delimited and classified." Sections 6, 7 and 8 of CA No. 141 read as follows:

"Sec. 6. The President, upon the recommendation of the Secretary of Agriculture and Commerce, shall from time to time classify the lands of the public domain into –(a) Alienable or disposable,(b) Timber, and(c) Mineral lands,and may at any time and in like manner transfer such lands from one class to another,53 for the purpose of their administration and disposition.

Sec. 7. For the purposes of the administration and disposition of alienable or disposable public lands, the President, upon recommendation by the Secretary of Agriculture and Commerce, shall from time to time declare what lands are open to disposition or concession under this Act.

Sec. 8. Only those lands shall be declared open to disposition or concession which have been officially delimited and classified and, when practicable, surveyed, and which have not been reserved for public or quasi-public uses, nor appropriated by the Government, nor in any manner become private property, nor those on which a private right authorized and recognized by this Act or any other valid law may be claimed, or which, having been reserved or appropriated, have ceased to be so. x x x."

Thus, before the government could alienate or dispose of lands of the public domain, the President must first officially classify these lands as alienable or disposable, and then declare them open to disposition or concession. There must be no law reserving these lands for public or quasi-public uses.

The salient provisions of CA No. 141, on government reclaimed, foreshore and marshy lands of the public domain, are as follows:

"Sec. 58. Any tract of land of the public domain which, being neither timber nor mineral land, is intended to be used for residential purposes or for commercial, industrial, or other productive purposes other than agricultural, and is open to disposition or concession, shall be disposed of under the provisions of this chapter and not otherwise.

Sec. 59. The lands disposable under this title shall be classified as follows:(a) Lands reclaimed by the Government by dredging, filling, or other means;(b) Foreshore;(c) Marshy lands or lands covered with water bordering upon the shores or banks of navigable lakes or rivers;(d) Lands not included in any of the foregoing classes.

Sec. 60. Any tract of land comprised under this title may be leased or sold, as the case may be, to any person, corporation, or association authorized to purchase or lease public lands for agricultural purposes. x x x.Sec. 61. The lands comprised in classes (a), (b), and (c) of section fifty-nine shall be disposed of to private parties by lease only and not otherwise, as soon as the President, upon recommendation by the Secretary of Agriculture, shall declare that the same are not necessary for the public service and are open to disposition under this chapter. The lands included in class (d) may be disposed of by sale or lease under the provisions of this Act." (Emphasis supplied)

Section 61 of CA No. 141 readopted, after the effectivity of the 1935 Constitution, Section 58 of Act No. 2874 prohibiting the sale of government reclaimed, foreshore and marshy disposable lands of the public domain. All these lands are intended for residential, commercial, industrial or other non-agricultural purposes. As before, Section 61 allowed only the lease of such lands to private parties. The government could sell to private parties only lands falling under Section 59 (d) of CA No. 141, or those lands for non-agricultural purposes not classified as government reclaimed, foreshore and marshy disposable lands of the public domain. Foreshore lands, however, became inalienable under the 1935 Constitution which only allowed the lease of these lands to qualified private parties.

Section 58 of CA No. 141 expressly states that disposable lands of the public domain intended for residential, commercial, industrial or other productive purposes other than agricultural "shall be disposed of under the provisions of this chapter and not otherwise." Under Section 10 of CA No. 141, the term "disposition" includes lease of the land. Any disposition of government reclaimed, foreshore and marshy

Page 33: Natural Resources Set2.Classification Cases

33NATURAL RESOURCES: CLASSIFICATIONS CASES

disposable lands for non-agricultural purposes must comply with Chapter IX, Title III of CA No. 141, unless a subsequent law amended or repealed these provisions.

In his concurring opinion in the landmark case of Republic Real Estate Corporation v. Court of Appeals,Justice Reynato S. Puno summarized succinctly the law on this matter, as follows:

"Foreshore lands are lands of public dominion intended for public use. So too are lands reclaimed by the government by dredging, filling, or other means. Act 1654 mandated that the control and disposition of the foreshore and lands under water remained in the national government. Said law allowed only the 'leasing' of reclaimed land. The Public Land Acts of 1919 and 1936 also declared that the foreshore and lands reclaimed by the government were to be "disposed of to private parties by lease only and not otherwise." Before leasing, however, the Governor-General, upon recommendation of the Secretary of Agriculture and Natural Resources, had first to determine that the land reclaimed was not necessary for the public service. This requisite must have been met before the land could be disposed of. But even then, the foreshore and lands under water were not to be alienated and sold to private parties. The disposition of the reclaimed land was only by lease. The land remained property of the State." (Emphasis supplied)

As observed by Justice Puno in his concurring opinion, "Commonwealth Act No. 141 has remained in effect at present."

The State policy prohibiting the sale to private parties of government reclaimed, foreshore and marshy alienable lands of the public domain, first implemented in 1907 was thus reaffirmed in CA No. 141 after the 1935 Constitution took effect. The prohibition on the sale of foreshore lands, however, became a constitutional edict under the 1935 Constitution. Foreshore lands became inalienable as natural resources of the State, unless reclaimed by the government and classified as agricultural lands of the public domain, in which case they would fall under the classification of government reclaimed lands.

After the effectivity of the 1935 Constitution, government reclaimed and marshy disposable lands of the public domain continued to be only leased and not sold to private parties. These lands remained sui generis, as the only alienable or disposable lands of the public domain the government could not sell to private parties.

Since then and until now, the only way the government can sell to private parties government reclaimed and marshy disposable lands of the public domain is for the legislature to pass a law authorizing such sale. CA No. 141 does not authorize the President to reclassify government reclaimed and marshy lands into other non-

agricultural lands under Section 59 (d). Lands classified under Section 59 (d) are the only alienable or disposable lands for non-agricultural purposes that the government could sell to private parties.

Moreover, Section 60 of CA No. 141 expressly requires congressional authority before lands under Section 59 that the government previously transferred to government units or entities could be sold to private parties. Section 60 of CA No. 141 declares that –

"Sec. 60. x x x The area so leased or sold shall be such as shall, in the judgment of the Secretary of Agriculture and Natural Resources, be reasonably necessary for the purposes for which such sale or lease is requested, and shall not exceed one hundred and forty-four hectares: Provided, however, That this limitation shall not apply to grants, donations, or transfers made to a province, municipality or branch or subdivision of the Government for the purposes deemed by said entities conducive to the public interest;but the land so granted, donated, or transferred to a province, municipality or branch or subdivision of the Government shall not be alienated, encumbered, or otherwise disposed of in a manner affecting its title, except when authorized by Congress: x x x." (Emphasis supplied)

The congressional authority required in Section 60 of CA No. 141 mirrors the legislative authority required in Section 56 of Act No. 2874.

One reason for the congressional authority is that Section 60 of CA No. 141 exempted government units and entities from the maximum area of public lands that could be acquired from the State. These government units and entities should not just turn around and sell these lands to private parties in violation of constitutional or statutory limitations. Otherwise, the transfer of lands for non-agricultural purposes to government units and entities could be used to circumvent constitutional limitations on ownership of alienable or disposable lands of the public domain. In the same manner, such transfers could also be used to evade the statutory prohibition in CA No. 141 on the sale of government reclaimed and marshy lands of the public domain to private parties. Section 60 of CA No. 141 constitutes by operation of law a lien on these lands.

In case of sale or lease of disposable lands of the public domain falling under Section 59 of CA No. 141, Sections 63 and 67 require a public bidding. Sections 63 and 67 of CA No. 141 provide as follows:

"Sec. 63. Whenever it is decided that lands covered by this chapter are not needed for public purposes, the Director of Lands shall ask the Secretary of Agriculture and Commerce (now the Secretary of Natural Resources) for

Page 34: Natural Resources Set2.Classification Cases

34NATURAL RESOURCES: CLASSIFICATIONS CASES

authority to dispose of the same. Upon receipt of such authority, the Director of Lands shall give notice by public advertisement in the same manner as in the case of leases or sales of agricultural public land, x x x.Sec. 67. The lease or sale shall be made by oral bidding; and adjudication shall be made to the highest bidder. x x x." (Emphasis supplied)

Thus, CA No. 141 mandates the Government to put to public auction all leases or sales of alienable or disposable lands of the public domain.

Like Act No. 1654 and Act No. 2874 before it, CA No. 141 did not repeal Section 5 of the Spanish Law of Waters of 1866. Private parties could still reclaim portions of the sea with government permission. However, thereclaimed land could become private land only if classified as alienable agricultural land of the public domain open to disposition under CA No. 141. The 1935 Constitution prohibited the alienation of all natural resources except public agricultural lands.

The Civil Code of 1950

The Civil Code of 1950 readopted substantially the definition of property of public dominion found in the Civil Code of 1889. Articles 420 and 422 of the Civil Code of 1950 state that –

"Art. 420. The following things are property of public dominion:(1) Those intended for public use, such as roads, canals, rivers, torrents, ports and bridges constructed by the State, banks, shores, roadsteads, and others of similar character;(2) Those which belong to the State, without being for public use, and are intended for some public service or for the development of the national wealth.x x x.Art. 422. Property of public dominion, when no longer intended for public use or for public service, shall form part of the patrimonial property of the State."

Again, the government must formally declare that the property of public dominion is no longer needed for public use or public service, before the same could be classified as patrimonial property of the State. In the case of government reclaimed and marshy lands of the public domain, the declaration of their being disposable, as well as the manner of their disposition, is governed by the applicable provisions of CA No. 141.

Like the Civil Code of 1889, the Civil Code of 1950 included as property of public dominion those properties of the State which, without being for public use, are intended for public service or the "development of the national wealth." Thus, government reclaimed and marshy lands of the State, even if not employed for public

use or public service, if developed to enhance the national wealth, are classified as property of public dominion.

Dispositions under the 1973 Constitution

The 1973 Constitution, which took effect on January 17, 1973, likewise adopted the Regalian doctrine. Section 8, Article XIV of the 1973 Constitution stated that –

"Sec. 8. All lands of the public domain, waters, minerals, coal, petroleum and other mineral oils, all forces of potential energy, fisheries, wildlife, and other natural resources of the Philippines belong to the State.With the exception of agricultural, industrial or commercial, residential, and resettlement lands of the public domain, natural resources shall not be alienated, and no license, concession, or lease for the exploration, development, exploitation, or utilization of any of the natural resources shall be granted for a period exceeding twenty-five years, renewable for not more than twenty-five years, except as to water rights for irrigation, water supply, fisheries, or industrial uses other than the development of water power, in which cases, beneficial use may be the measure and the limit of the grant." (Emphasis supplied)

The 1973 Constitution prohibited the alienation of all natural resources with the exception of "agricultural, industrial or commercial, residential, and resettlement lands of the public domain." In contrast, the 1935 Constitution barred the alienation of all natural resources except "public agricultural lands." However, the term "public agricultural lands" in the 1935 Constitution encompassed industrial, commercial, residential and resettlement lands of the public domain.60 If the land of public domain were neither timber nor mineral land, it would fall under the classification of agricultural land of the public domain. Both the 1935 and 1973 Constitutions, therefore, prohibited the alienation of all natural resources except agricultural lands of the public domain.

The 1973 Constitution, however, limited the alienation of lands of the public domain to individuals who were citizens of the Philippines. Private corporations, even if wholly owned by Philippine citizens, were no longer allowed to acquire alienable lands of the public domain unlike in the 1935 Constitution. Section 11, Article XIV of the 1973 Constitution declared that –

"Sec. 11. The Batasang Pambansa, taking into account conservation, ecological, and development requirements of the natural resources, shall determine by law the size of land of the public domain which may be developed, held or acquired by, or leased to, any qualified individual, corporation, or association, and the conditions therefor. No private

Page 35: Natural Resources Set2.Classification Cases

35NATURAL RESOURCES: CLASSIFICATIONS CASES

corporation or association may hold alienable lands of the public domain except by lease not to exceed one thousand hectares in area nor may any citizen hold such lands by lease in excess of five hundred hectares or acquire by purchase, homestead or grant, in excess of twenty-four hectares. No private corporation or association may hold by lease, concession, license or permit, timber or forest lands and other timber or forest resources in excess of one hundred thousand hectares. However, such area may be increased by the Batasang Pambansa upon recommendation of the National Economic and Development Authority." (Emphasis supplied)

Thus, under the 1973 Constitution, private corporations could hold alienable lands of the public domain only through lease. Only individuals could now acquire alienable lands of the public domain, and private corporations became absolutely barred from acquiring any kind of alienable land of the public domain. The constitutional ban extended to all kinds of alienable lands of the public domain, while the statutory ban under CA No. 141 applied only to government reclaimed, foreshore and marshy alienable lands of the public domain.

PD No. 1084 Creating the Public Estates Authority

On February 4, 1977, then President Ferdinand Marcos issued Presidential Decree No. 1084 creating PEA, a wholly government owned and controlled corporation with a special charter. Sections 4 and 8 of PD No. 1084, vests PEA with the following purposes and powers:

"Sec. 4. Purpose. The Authority is hereby created for the following purposes:(a) To reclaim land, including foreshore and submerged areas, by dredging, filling or other means, or to acquire reclaimed land;(b) To develop, improve, acquire, administer, deal in, subdivide, dispose, lease and sell any and all kinds of lands, buildings, estates and other forms of real property, owned, managed, controlled and/or operated by the government;(c) To provide for, operate or administer such service as may be necessary for the efficient, economical and beneficial utilization of the above properties.Sec. 5. Powers and functions of the Authority. The Authority shall, in carrying out the purposes for which it is created, have the following powers and functions:(a)To prescribe its by-laws.x x x(i) To hold lands of the public domain in excess of the area permitted to private corporations by statute.(j) To reclaim lands and to construct work across, or otherwise, any stream, watercourse, canal, ditch, flume x x x.x x x

(o) To perform such acts and exercise such functions as may be necessary for the attainment of the purposes and objectives herein specified." (Emphasis supplied)

PD No. 1084 authorizes PEA to reclaim both foreshore and submerged areas of the public domain. Foreshore areas are those covered and uncovered by the ebb and flow of the tide. Submerged areas are those permanently under water regardless of the ebb and flow of the tide. Foreshore and submerged areas indisputably belong to the public domain and are inalienable unless reclaimed, classified as alienable lands open to disposition, and further declared no longer needed for public service.

The ban in the 1973 Constitution on private corporations from acquiring alienable lands of the public domain did not apply to PEA since it was then, and until today, a fully owned government corporation. The constitutional ban applied then, as it still applies now, only to "private corporations and associations." PD No. 1084 expressly empowers PEA "to hold lands of the public domain" even "in excess of the area permitted to private corporations by statute." Thus, PEA can hold title to private lands, as well as title to lands of the public domain.

In order for PEA to sell its reclaimed foreshore and submerged alienable lands of the public domain, there must be legislative authority empowering PEA to sell these lands. This legislative authority is necessary in view of Section 60 of CA No.141, which states –

"Sec. 60. x x x; but the land so granted, donated or transferred to a province, municipality, or branch or subdivision of the Government shall not be alienated, encumbered or otherwise disposed of in a manner affecting its title, except when authorized by Congress; x x x." (Emphasis supplied)

Without such legislative authority, PEA could not sell but only lease its reclaimed foreshore and submerged alienable lands of the public domain. Nevertheless, any legislative authority granted to PEA to sell its reclaimed alienable lands of the public domain would be subject to the constitutional ban on private corporations from acquiring alienable lands of the public domain. Hence, such legislative authority could only benefit private individuals.

Dispositions under the 1987 Constitution

The 1987 Constitution, like the 1935 and 1973 Constitutions before it, has adopted the Regalian doctrine. The 1987 Constitution declares that all natural resources are "owned by the State," and except for alienable agricultural lands of the public

Page 36: Natural Resources Set2.Classification Cases

36NATURAL RESOURCES: CLASSIFICATIONS CASES

domain, natural resources cannot be alienated. Sections 2 and 3, Article XII of the 1987 Constitution state that –

"Section 2. All lands of the public domain, waters, minerals, coal, petroleum and other mineral oils, all forces of potential energy, fisheries, forests or timber, wildlife, flora and fauna, and other natural resources are owned by the State. With the exception of agricultural lands, all other natural resources shall not be alienated. The exploration, development, and utilization of natural resources shall be under the full control and supervision of the State. x x x.Section 3. Lands of the public domain are classified into agricultural, forest or timber, mineral lands, and national parks. Agricultural lands of the public domain may be further classified by law according to the uses which they may be devoted. Alienable lands of the public domain shall be limited to agricultural lands. Private corporations or associations may not hold such alienable lands of the public domain except by lease, for a period not exceeding twenty-five years, renewable for not more than twenty-five years, and not to exceed one thousand hectares in area. Citizens of the Philippines may lease not more than five hundred hectares, or acquire not more than twelve hectares thereof by purchase, homestead, or grant.Taking into account the requirements of conservation, ecology, and development, and subject to the requirements of agrarian reform, the Congress shall determine, by law, the size of lands of the public domain which may be acquired, developed, held, or leased and the conditions therefor." (Emphasis supplied)

The 1987 Constitution continues the State policy in the 1973 Constitution banning private corporations fromacquiring any kind of alienable land of the public domain. Like the 1973 Constitution, the 1987 Constitution allows private corporations to hold alienable lands of the public domain only through lease. As in the 1935 and 1973 Constitutions, the general law governing the lease to private corporations of reclaimed, foreshore and marshy alienable lands of the public domain is still CA No. 141.

The Rationale behind the Constitutional Ban

The rationale behind the constitutional ban on corporations from acquiring, except through lease, alienable lands of the public domain is not well understood. During the deliberations of the 1986 Constitutional Commission, the commissioners probed the rationale behind this ban, thus:

"FR. BERNAS: Mr. Vice-President, my questions have reference to page 3, line 5 which says:

`No private corporation or association may hold alienable lands of the public domain except by lease, not to exceed one thousand hectares in area.'If we recall, this provision did not exist under the 1935 Constitution, but this was introduced in the 1973 Constitution. In effect, it prohibits private corporations from acquiring alienable public lands. But it has not been very clear in jurisprudence what the reason for this is. In some of the cases decided in 1982 and 1983, it was indicated that the purpose of this is to prevent large landholdings. Is that the intent of this provision?

MR. VILLEGAS: I think that is the spirit of the provision.

FR. BERNAS: In existing decisions involving the Iglesia ni Cristo, there were instances where the Iglesia ni Cristo was not allowed to acquire a mere 313-square meter land where a chapel stood because the Supreme Court said it would be in violation of this." (Emphasis supplied)

In Ayog v. Cusi, the Court explained the rationale behind this constitutional ban in this way:

"Indeed, one purpose of the constitutional prohibition against purchases of public agricultural lands by private corporations is to equitably diffuse land ownership or to encourage 'owner-cultivatorship and the economic family-size farm' and to prevent a recurrence of cases like the instant case. Huge landholdings by corporations or private persons had spawned social unrest."

However, if the constitutional intent is to prevent huge landholdings, the Constitution could have simply limited the size of alienable lands of the public domain that corporations could acquire. The Constitution could have followed the limitations on individuals, who could acquire not more than 24 hectares of alienable lands of the public domain under the 1973 Constitution, and not more than 12 hectares under the 1987 Constitution.

If the constitutional intent is to encourage economic family-size farms, placing the land in the name of a corporation would be more effective in preventing the break-up of farmlands. If the farmland is registered in the name of a corporation, upon the death of the owner, his heirs would inherit shares in the corporation instead of subdivided parcels of the farmland. This would prevent the continuing break-up of farmlands into smaller and smaller plots from one generation to the next.

In actual practice, the constitutional ban strengthens the constitutional limitation on individuals from acquiring more than the allowed area of alienable lands of the public domain. Without the constitutional ban, individuals who already acquired the maximum area of alienable lands of the public domain could easily set up

Page 37: Natural Resources Set2.Classification Cases

37NATURAL RESOURCES: CLASSIFICATIONS CASES

corporations to acquire more alienable public lands. An individual could own as many corporations as his means would allow him. An individual could even hide his ownership of a corporation by putting his nominees as stockholders of the corporation. The corporation is a convenient vehicle to circumvent the constitutional limitation on acquisition by individuals of alienable lands of the public domain.

The constitutional intent, under the 1973 and 1987 Constitutions, is to transfer ownership of only a limited area of alienable land of the public domain to a qualified individual. This constitutional intent is safeguarded by the provision prohibiting corporations from acquiring alienable lands of the public domain, since the vehicle to circumvent the constitutional intent is removed. The available alienable public lands are gradually decreasing in the face of an ever-growing population. The most effective way to insure faithful adherence to this constitutional intent is to grant or sell alienable lands of the public domain only to individuals. This, it would seem, is the practical benefit arising from the constitutional ban.

The Amended Joint Venture Agreement

The subject matter of the Amended JVA, as stated in its second Whereas clause, consists of three properties, namely:

1. "[T]hree partially reclaimed and substantially eroded islands along Emilio Aguinaldo Boulevard in Paranaque and Las Pinas, Metro Manila, with a combined titled area of 1,578,441 square meters;"2. "[A]nother area of 2,421,559 square meters contiguous to the three islands;" and3. "[A]t AMARI's option as approved by PEA, an additional 350 hectares more or less to regularize the configuration of the reclaimed area."

PEA confirms that the Amended JVA involves "the development of the Freedom Islands and further reclamation of about 250 hectares x x x," plus an option "granted to AMARI to subsequently reclaim another 350 hectares x x x."

In short, the Amended JVA covers a reclamation area of 750 hectares. Only 157.84 hectares of the 750-hectare reclamation project have been reclaimed, and the rest of the 592.15 hectares are still submerged areas forming part of Manila Bay.

Under the Amended JVA, AMARI will reimburse PEA the sum of P1,894,129,200.00 for PEA's "actual cost" in partially reclaiming the Freedom Islands. AMARI will also complete, at its own expense, the reclamation of the Freedom Islands. AMARI will further shoulder all the reclamation costs of all the other areas, totaling 592.15 hectares, still to be reclaimed. AMARI and PEA will share, in the

proportion of 70 percent and 30 percent, respectively, the total net usable area which is defined in the Amended JVA as the total reclaimed area less 30 percent earmarked for common areas. Title to AMARI's share in the net usable area, totaling 367.5 hectares, will be issued in the name of AMARI. Section 5.2 (c) of the Amended JVA provides that –

"x x x, PEA shall have the duty to execute without delay the necessary deed of transfer or conveyance of the title pertaining to AMARI's Land share based on the Land Allocation Plan. PEA, when requested in writing by AMARI, shall then cause the issuance and delivery of the proper certificates of title covering AMARI's Land Share in the name of AMARI, x x x; provided, that if more than seventy percent (70%) of the titled area at any given time pertains to AMARI, PEA shall deliver to AMARI only seventy percent (70%) of the titles pertaining to AMARI, until such time when a corresponding proportionate area of additional land pertaining to PEA has been titled." (Emphasis supplied)

Indisputably, under the Amended JVA AMARI will acquire and own a maximum of 367.5 hectares of reclaimed land which will be titled in its name.

To implement the Amended JVA, PEA delegated to the unincorporated PEA-AMARI joint venture PEA's statutory authority, rights and privileges to reclaim foreshore and submerged areas in Manila Bay. Section 3.2.a of the Amended JVA states that –

"PEA hereby contributes to the joint venture its rights and privileges to perform Rawland Reclamation and Horizontal Development as well as own the Reclamation Area, thereby granting the Joint Venture the full and exclusive right, authority and privilege to undertake the Project in accordance with the Master Development Plan."

The Amended JVA is the product of a renegotiation of the original JVA dated April 25, 1995 and its supplemental agreement dated August 9, 1995.

The Threshold Issue

The threshold issue is whether AMARI, a private corporation, can acquire and own under the Amended JVA 367.5 hectares of reclaimed foreshore and submerged areas in Manila Bay in view of Sections 2 and 3, Article XII of the 1987 Constitution which state that:

"Section 2. All lands of the public domain, waters, minerals, coal, petroleum, and other mineral oils, all forces of potential energy, fisheries, forests or timber, wildlife, flora and fauna, and other natural resources are owned by the

Page 38: Natural Resources Set2.Classification Cases

38NATURAL RESOURCES: CLASSIFICATIONS CASES

State. With the exception of agricultural lands, all other natural resources shall not be alienated. x x x.x x xSection 3. x x x Alienable lands of the public domain shall be limited to agricultural lands. Private corporations or associations may not hold such alienable lands of the public domain except by lease, x x x."(Emphasis supplied)

Classification of Reclaimed Foreshore and Submerged Areas

PEA readily concedes that lands reclaimed from foreshore or submerged areas of Manila Bay are alienable or disposable lands of the public domain. In its Memorandum, PEA admits that –

"Under the Public Land Act (CA 141, as amended), reclaimed lands are classified as alienable and disposable lands of the public domain:'Sec. 59. The lands disposable under this title shall be classified as follows:(a) Lands reclaimed by the government by dredging, filling, or other means;x x x.'" (Emphasis supplied)

Likewise, the Legal Task Force constituted under Presidential Administrative Order No. 365 admitted in its Report and Recommendation to then President Fidel V. Ramos, "[R]eclaimed lands are classified as alienable and disposable lands of the public domain." The Legal Task Force concluded that –

"D. Conclusion

Reclaimed lands are lands of the public domain. However, by statutory authority, the rights of ownership and disposition over reclaimed lands have been transferred to PEA, by virtue of which PEA, as owner, may validly convey the same to any qualified person without violating the Constitution or any statute.The constitutional provision prohibiting private corporations from holding public land, except by lease (Sec. 3, Art. XVII, 1987 Constitution), does not apply to reclaimed lands whose ownership has passed on to PEA by statutory grant."

Under Section 2, Article XII of the 1987 Constitution, the foreshore and submerged areas of Manila Bay are part of the "lands of the public domain, waters x x x and other natural resources" and consequently "owned by the State." As such, foreshore and submerged areas "shall not be alienated," unless they are classified as "agricultural lands" of the public domain. The mere reclamation of these areas by PEA does not convert these inalienable natural resources of the State into alienable or disposable lands of the public domain. There must be a law or presidential proclamation officially classifying these reclaimed lands as alienable or disposable

and open to disposition or concession. Moreover, these reclaimed lands cannot be classified as alienable or disposable if the law has reserved them for some public or quasi-public use.

Section 8 of CA No. 141 provides that "only those lands shall be declared open to disposition or concession which have been officially delimited and classified." The President has the authority to classify inalienable lands of the public domain into alienable or disposable lands of the public domain, pursuant to Section 6 of CA No. 141. In Laurel vs. Garcia, the Executive Department attempted to sell the Roppongi property in Tokyo, Japan, which was acquired by the Philippine Government for use as the Chancery of the Philippine Embassy. Although the Chancery had transferred to another location thirteen years earlier, the Court still ruled that, under Article 422 of the Civil Code, a property of public dominion retains such character until formally declared otherwise. The Court ruled that –

"The fact that the Roppongi site has not been used for a long time for actual Embassy service does not automatically convert it to patrimonial property. Any such conversion happens only if the property is withdrawn from public use (Cebu Oxygen and Acetylene Co. v. Bercilles, 66 SCRA 481 [1975]. A property continues to be part of the public domain, not available for private appropriation or ownership 'until there is a formal declaration on the part of the government to withdraw it from being such' (Ignacio v. Director of Lands, 108 Phil. 335 [1960]." (Emphasis supplied)

PD No. 1085, issued on February 4, 1977, authorized the issuance of special land patents for lands reclaimed by PEA from the foreshore or submerged areas of Manila Bay. On January 19, 1988 then President Corazon C. Aquino issued Special Patent No. 3517 in the name of PEA for the 157.84 hectares comprising the partially reclaimed Freedom Islands. Subsequently, on April 9, 1999 the Register of Deeds of the Municipality of Paranaque issued TCT Nos. 7309, 7311 and 7312 in the name of PEA pursuant to Section 103 of PD No. 1529 authorizing the issuance of certificates of title corresponding to land patents. To this day, these certificates of title are still in the name of PEA.

PD No. 1085, coupled with President Aquino's actual issuance of a special patent covering the Freedom Islands, is equivalent to an official proclamation classifying the Freedom Islands as alienable or disposable lands of the public domain. PD No. 1085 and President Aquino's issuance of a land patent also constitute a declaration that the Freedom Islands are no longer needed for public service. The Freedom Islands are thus alienable or disposable lands of the public domain, open to disposition or concession to qualified parties.

At the time then President Aquino issued Special Patent No. 3517, PEA had already reclaimed the Freedom Islands although subsequently there were partial

Page 39: Natural Resources Set2.Classification Cases

39NATURAL RESOURCES: CLASSIFICATIONS CASES

erosions on some areas. The government had also completed the necessary surveys on these islands. Thus, the Freedom Islands were no longer part of Manila Bay but part of the land mass. Section 3, Article XII of the 1987 Constitution classifies lands of the public domain into "agricultural, forest or timber, mineral lands, and national parks." Being neither timber, mineral, nor national park lands, the reclaimed Freedom Islands necessarily fall under the classification of agricultural lands of the public domain. Under the 1987 Constitution, agricultural lands of the public domain are the only natural resources that the State may alienate to qualified private parties. All other natural resources, such as the seas or bays, are "waters x x x owned by the State" forming part of the public domain, and are inalienable pursuant to Section 2, Article XII of the 1987 Constitution.

AMARI claims that the Freedom Islands are private lands because CDCP, then a private corporation, reclaimed the islands under a contract dated November 20, 1973 with the Commissioner of Public Highways. AMARI, citing Article 5 of the Spanish Law of Waters of 1866, argues that "if the ownership of reclaimed lands may be given to the party constructing the works, then it cannot be said that reclaimed lands are lands of the public domain which the State may not alienate."  Article 5 of the Spanish Law of Waters reads as follows:

"Article 5. Lands reclaimed from the sea in consequence of works constructed by the State, or by the provinces, pueblos or private persons, with proper permission, shall become the property of the party constructing such works, unless otherwise provided by the terms of the grant of authority." (Emphasis supplied)

Under Article 5 of the Spanish Law of Waters of 1866, private parties could reclaim from the sea only with "proper permission" from the State. Private parties could own the reclaimed land only if not "otherwise provided by the terms of the grant of authority." This clearly meant that no one could reclaim from the sea without permission from the State because the sea is property of public dominion. It also meant that the State could grant or withhold ownership of the reclaimed land because any reclaimed land, like the sea from which it emerged, belonged to the State. Thus, a private person reclaiming from the sea without permission from the State could not acquire ownership of the reclaimed land which would remain property of public dominion like the sea it replaced.76 Article 5 of the Spanish Law of Waters of 1866 adopted the time-honored principle of land ownership that "all lands that were not acquired from the government, either by purchase or by grant, belong to the public domain."

Article 5 of the Spanish Law of Waters must be read together with laws subsequently enacted on the disposition of public lands. In particular, CA No. 141 requires that lands of the public domain must first be classified as alienable or disposable before the government can alienate them. These lands must not be

reserved for public or quasi-public purposes. Moreover, the contract between CDCP and the government was executed after the effectivity of the 1973 Constitution which barred private corporations from acquiring any kind of alienable land of the public domain. This contract could not have converted the Freedom Islands into private lands of a private corporation.

Presidential Decree No. 3-A, issued on January 11, 1973, revoked all laws authorizing the reclamation of areas under water and revested solely in the National Government the power to reclaim lands. Section 1 of PD No. 3-A declared that –

"The provisions of any law to the contrary notwithstanding, the reclamation of areas under water, whether foreshore or inland, shall be limited to the National Government or any person authorized by it under a proper contract. (Emphasis supplied)x x x."

PD No. 3-A repealed Section 5 of the Spanish Law of Waters of 1866 because reclamation of areas under water could now be undertaken only by the National Government or by a person contracted by the National Government. Private parties may reclaim from the sea only under a contract with the National Government, and no longer by grant or permission as provided in Section 5 of the Spanish Law of Waters of 1866.

Executive Order No. 525, issued on February 14, 1979, designated PEA as the National Government's implementing arm to undertake "all reclamation projects of the government," which "shall be undertaken by the PEA or through a proper contract executed by it with any person or entity." Under such contract, a private party receives compensation for reclamation services rendered to PEA. Payment to the contractor may be in cash, or in kind consisting of portions of the reclaimed land, subject to the constitutional ban on private corporations from acquiring alienable lands of the public domain. The reclaimed land can be used as payment in kind only if the reclaimed land is first classified as alienable or disposable land open to disposition, and then declared no longer needed for public service.

The Amended JVA covers not only the Freedom Islands, but also an additional 592.15 hectares which are still submerged and forming part of Manila Bay. There is no legislative or Presidential act classifying these submerged areas as alienable or disposable lands of the public domain open to disposition. These submerged areas are not covered by any patent or certificate of title. There can be no dispute that these submerged areas form part of the public domain, and in their present state are inalienable and outside the commerce of man. Until reclaimed from the sea, these submerged areas are, under the Constitution, "waters x x x owned by the State," forming part of the public domain and consequently inalienable. Only when actually reclaimed from the sea can these

Page 40: Natural Resources Set2.Classification Cases

40NATURAL RESOURCES: CLASSIFICATIONS CASES

submerged areas be classified as public agricultural lands, which under the Constitution are the only natural resources that the State may alienate. Once reclaimed and transformed into public agricultural lands, the government may then officially classify these lands as alienable or disposable lands open to disposition. Thereafter, the government may declare these lands no longer needed for public service. Only then can these reclaimed lands be considered alienable or disposable lands of the public domain and within the commerce of man.

The classification of PEA's reclaimed foreshore and submerged lands into alienable or disposable lands open to disposition is necessary because PEA is tasked under its charter to undertake public services that require the use of lands of the public domain. Under Section 5 of PD No. 1084, the functions of PEA include the following: "[T]o own or operate railroads, tramways and other kinds of land transportation, x x x; [T]o construct, maintain and operate such systems of sanitary sewers as may be necessary; [T]o construct, maintain and operate such storm drains as may be necessary." PEA is empowered to issue "rules and regulations as may be necessary for the proper use by private parties of any or all of the highways, roads, utilities, buildings and/or any of its properties and to impose or collect fees or tolls for their use." Thus, part of the reclaimed foreshore and submerged lands held by the PEA would actually be needed for public use or service since many of the functions imposed on PEA by its charter constitute essential public services.

Moreover, Section 1 of Executive Order No. 525 provides that PEA "shall be primarily responsible for integrating, directing, and coordinating all reclamation projects for and on behalf of the National Government." The same section also states that "[A]ll reclamation projects shall be approved by the President upon recommendation of the PEA, and shall be undertaken by the PEA or through a proper contract executed by it with any person or entity; x x x." Thus, under EO No. 525, in relation to PD No. 3-A and PD No.1084, PEA became the primary implementing agency of the National Government to reclaim foreshore and submerged lands of the public domain. EO No. 525 recognized PEA as the government entity "to undertake the reclamation of lands and ensure their maximum utilization in promoting public welfare and interests." Since large portions of these reclaimed lands would obviously be needed for public service, there must be a formal declaration segregating reclaimed lands no longer needed for public service from those still needed for public service.

Section 3 of EO No. 525, by declaring that all lands reclaimed by PEA "shall belong to or be owned by the PEA," could not automatically operate to classify inalienable lands into alienable or disposable lands of the public domain. Otherwise, reclaimed foreshore and submerged lands of the public domain would automatically become alienable once reclaimed by PEA, whether or not classified as alienable or disposable.

The Revised Administrative Code of 1987, a later law than either PD No. 1084 or EO No. 525, vests in the Department of Environment and Natural Resources ("DENR" for brevity) the following powers and functions:

"Sec. 4. Powers and Functions. The Department shall:(1) x x xx x x(4) Exercise supervision and control over forest lands, alienable and disposable public lands, mineral resources and, in the process of exercising such control, impose appropriate taxes, fees, charges, rentals and any such form of levy and collect such revenues for the exploration, development, utilization or gathering of such resources;x x x(14) Promulgate rules, regulations and guidelines on the issuance of licenses, permits, concessions, lease agreements and such other privileges concerning the development, exploration and utilization of the country's marine, freshwater, and brackish water and over all aquatic resources of the country and shall continue to oversee, supervise and police our natural resources; cancel or cause to cancel such privileges upon failure, non-compliance or violations of any regulation, order, and for all other causes which are in furtherance of the conservation of natural resources and supportive of the national interest;(15) Exercise exclusive jurisdiction on the management and disposition of all lands of the public domain and serve as the sole agency responsible for classification, sub-classification, surveying and titling of lands in consultation with appropriate agencies." (Emphasis supplied)

As manager, conservator and overseer of the natural resources of the State, DENR exercises "supervision and control over alienable and disposable public lands." DENR also exercises "exclusive jurisdiction on the management and disposition of all lands of the public domain." Thus, DENR decides whether areas under water, like foreshore or submerged areas of Manila Bay, should be reclaimed or not. This means that PEA needs authorization from DENR before PEA can undertake reclamation projects in Manila Bay, or in any part of the country.

DENR also exercises exclusive jurisdiction over the disposition of all lands of the public domain. Hence, DENR decides whether reclaimed lands of PEA should be classified as alienable under Sections 6 and 7 of CA No. 141. Once DENR decides that the reclaimed lands should be so classified, it then recommends to the President the issuance of a proclamation classifying the lands as alienable or disposable lands of the public domain open to disposition. We note that then DENR Secretary Fulgencio S. Factoran, Jr. countersigned Special Patent No. 3517 in compliance with the Revised Administrative Code and Sections 6 and 7 of CA No. 141.

Page 41: Natural Resources Set2.Classification Cases

41NATURAL RESOURCES: CLASSIFICATIONS CASES

In short, DENR is vested with the power to authorize the reclamation of areas under water, while PEA is vested with the power to undertake the physical reclamation of areas under water, whether directly or through private contractors. DENR is also empowered to classify lands of the public domain into alienable or disposable lands subject to the approval of the President. On the other hand, PEA is tasked to develop, sell or lease the reclaimed alienable lands of the public domain.

Clearly, the mere physical act of reclamation by PEA of foreshore or submerged areas does not make the reclaimed lands alienable or disposable lands of the public domain, much less patrimonial lands of PEA. Likewise, the mere transfer by the National Government of lands of the public domain to PEA does not make the lands alienable or disposable lands of the public domain, much less patrimonial lands of PEA.

Absent two official acts – a classification that these lands are alienable or disposable and open to disposition and a declaration that these lands are not needed for public service, lands reclaimed by PEA remain inalienable lands of the public domain. Only such an official classification and formal declaration can convert reclaimed lands into alienable or disposable lands of the public domain, open to disposition under the Constitution, Title I and Title III of CA No. 141 and other applicable laws.

PEA's Authority to Sell Reclaimed Lands

PEA, like the Legal Task Force, argues that as alienable or disposable lands of the public domain, the reclaimed lands shall be disposed of in accordance with CA No. 141, the Public Land Act. PEA, citing Section 60 of CA No. 141, admits that reclaimed lands transferred to a branch or subdivision of the government "shall not be alienated, encumbered, or otherwise disposed of in a manner affecting its title, except when authorized by Congress: x x x." (Emphasis by PEA)

In Laurel vs. Garcia, the Court cited Section 48 of the Revised Administrative Code of 1987, which states that –

"Sec. 48. Official Authorized to Convey Real Property. Whenever real property of the Government is authorized by law to be conveyed, the deed of conveyance shall be executed in behalf of the government by the following: x x x."

Thus, the Court concluded that a law is needed to convey any real property belonging to the Government. The Court declared that -

"It is not for the President to convey real property of the government on his or her own sole will. Any such conveyance must be authorized and approved by a

law enacted by the Congress. It requires executive and legislative concurrence." (Emphasis supplied)

PEA contends that PD No. 1085 and EO No. 525 constitute the legislative authority allowing PEA to sell its reclaimed lands. PD No. 1085, issued on February 4, 1977, provides that –

"The land reclaimed in the foreshore and offshore area of Manila Bay pursuant to the contract for the reclamation and construction of the Manila-Cavite Coastal Road Project between the Republic of the Philippines and the Construction and Development Corporation of the Philippines dated November 20, 1973 and/or any other contract or reclamation covering the same area is hereby transferred, conveyed and assigned to the ownership and administration of the Public Estates Authority established pursuant to PD No. 1084; Provided, however, That the rights and interests of the Construction and Development Corporation of the Philippines pursuant to the aforesaid contract shall be recognized and respected.Henceforth, the Public Estates Authority shall exercise the rights and assume the obligations of the Republic of the Philippines (Department of Public Highways) arising from, or incident to, the aforesaid contract between the Republic of the Philippines and the Construction and Development Corporation of the Philippines.In consideration of the foregoing transfer and assignment, the Public Estates Authority shall issue in favor of the Republic of the Philippines the corresponding shares of stock in said entity with an issued value of said shares of stock (which) shall be deemed fully paid and non-assessable.The Secretary of Public Highways and the General Manager of the Public Estates Authority shall execute such contracts or agreements, including appropriate agreements with the Construction and Development Corporation of the Philippines, as may be necessary to implement the above.Special land patent/patents shall be issued by the Secretary of Natural Resources in favor of the Public Estates Authority without prejudice to the subsequent transfer to the contractor or his assignees of such portion or portions of the land reclaimed or to be reclaimed as provided for in the above-mentioned contract. On the basis of such patents, the Land Registration Commission shall issue the corresponding certificate of title ." (Emphasis supplied)

On the other hand, Section 3 of EO No. 525, issued on February 14, 1979, provides that -

"Sec. 3. All lands reclaimed by PEA shall belong to or be owned by the PEA which shall be responsible for its administration, development, utilization or disposition in accordance with the provisions of Presidential Decree No. 1084.

Page 42: Natural Resources Set2.Classification Cases

42NATURAL RESOURCES: CLASSIFICATIONS CASES

Any and all income that the PEA may derive from the sale, lease or use of reclaimed lands shall be used in accordance with the provisions of Presidential Decree No. 1084."

There is no express authority under either PD No. 1085 or EO No. 525 for PEA to sell its reclaimed lands. PD No. 1085 merely transferred "ownership and administration" of lands reclaimed from Manila Bay to PEA, while EO No. 525 declared that lands reclaimed by PEA "shall belong to or be owned by PEA." EO No. 525 expressly states that PEA should dispose of its reclaimed lands "in accordance with the provisions of Presidential Decree No. 1084," the charter of PEA.

PEA's charter, however, expressly tasks PEA "to develop, improve, acquire, administer, deal in, subdivide, dispose, lease and sell any and all kinds of lands x x x owned, managed, controlled and/or operated by the government." (Emphasis supplied) There is, therefore, legislative authority granted to PEA to sell its lands, whether patrimonial or alienable lands of the public domain. PEA may sell to private parties itspatrimonial properties in accordance with the PEA charter free from constitutional limitations. The constitutional ban on private corporations from acquiring alienable lands of the public domain does not apply to the sale of PEA's patrimonial lands.

PEA may also sell its alienable or disposable lands of the public domain to private individuals since, with the legislative authority, there is no longer any statutory prohibition against such sales and the constitutional ban does not apply to individuals. PEA, however, cannot sell any of its alienable or disposable lands of the public domain to private corporations since Section 3, Article XII of the 1987 Constitution expressly prohibits such sales. The legislative authority benefits only individuals. Private corporations remain barred from acquiring any kind of alienable land of the public domain, including government reclaimed lands.

The provision in PD No. 1085 stating that portions of the reclaimed lands could be transferred by PEA to the "contractor or his assignees" (Emphasis supplied) would not apply to private corporations but only to individuals because of the constitutional ban. Otherwise, the provisions of PD No. 1085 would violate both the 1973 and 1987 Constitutions.

The requirement of public auction in the sale of reclaimed lands

Assuming the reclaimed lands of PEA are classified as alienable or disposable lands open to disposition, and further declared no longer needed for public service, PEA would have to conduct a public bidding in selling or leasing these lands. PEA must observe the provisions of Sections 63 and 67 of CA No. 141 requiring public auction, in the absence of a law exempting PEA from holding a public

auction. Special Patent No. 3517 expressly states that the patent is issued by authority of the Constitution and PD No. 1084, "supplemented by Commonwealth Act No. 141, as amended." This is an acknowledgment that the provisions of CA No. 141 apply to the disposition of reclaimed alienable lands of the public domain unless otherwise provided by law. Executive Order No. 654, which authorizes PEA "to determine the kind and manner of payment for the transfer" of its assets and properties, does not exempt PEA from the requirement of public auction. EO No. 654 merely authorizes PEA to decide the mode of payment, whether in kind and in installment, but does not authorize PEA to dispense with public auction.

Moreover, under Section 79 of PD No. 1445, otherwise known as the Government Auditing Code, the government is required to sell valuable government property through public bidding. Section 79 of PD No. 1445 mandates that –

"Section 79. When government property has become unserviceable for any cause, or is no longer needed, it shall, upon application of the officer accountable therefor, be inspected by the head of the agency or his duly authorized representative in the presence of the auditor concerned and, if found to be valueless or unsaleable, it may be destroyed in their presence. If found to be valuable, it may be sold at public auction to the highest bidder under the supervision of the proper committee on award or similar body in the presence of the auditor concerned or other authorized representative of the Commission, after advertising by printed notice in the Official Gazette, or for not less than three consecutive days in any newspaper of general circulation, or where the value of the property does not warrant the expense of publication, by notices posted for a like period in at least three public places in the locality where the property is to be sold. In the event that the public auction fails, the property may be sold at a private sale at such price as may be fixed by the same committee or body concerned and approved by the Commission."

It is only when the public auction fails that a negotiated sale is allowed, in which case the Commission on Audit must approve the selling price. The Commission on Audit implements Section 79 of the Government Auditing Code through Circular No. 89-296 dated January 27, 1989. This circular emphasizes that government assets must be disposed of only through public auction, and a negotiated sale can be resorted to only in case of "failure of public auction."

At the public auction sale, only Philippine citizens are qualified to bid for PEA's reclaimed foreshore and submerged alienable lands of the public domain. Private corporations are barred from bidding at the auction sale of any kind of alienable land of the public domain.

Page 43: Natural Resources Set2.Classification Cases

43NATURAL RESOURCES: CLASSIFICATIONS CASES

PEA originally scheduled a public bidding for the Freedom Islands on December 10, 1991. PEA imposed a condition that the winning bidder should reclaim another 250 hectares of submerged areas to regularize the shape of the Freedom Islands, under a 60-40 sharing of the additional reclaimed areas in favor of the winning bidder. No one, however, submitted a bid. On December 23, 1994, the Government Corporate Counsel advised PEA it could sell the Freedom Islands through negotiation, without need of another public bidding, because of the failure of the public bidding on December 10, 1991.

However, the original JVA dated April 25, 1995 covered not only the Freedom Islands and the additional 250 hectares still to be reclaimed, it also granted an option to AMARI to reclaim another 350 hectares. The original JVA, a negotiated contract, enlarged the reclamation area to 750 hectares. The failure of public bidding on December 10, 1991, involving only 407.84 hectares, is not a valid justification for a negotiated sale of 750 hectares, almost double the area publicly auctioned. Besides, the failure of public bidding happened on December 10, 1991, more than three years before the signing of the original JVA on April 25, 1995. The economic situation in the country had greatly improved during the intervening period.

Reclamation under the BOT Law and the Local Government Code

The constitutional prohibition in Section 3, Article XII of the 1987 Constitution is absolute and clear: "Private corporations or associations may not hold such alienable lands of the public domain except by lease, x x x." Even Republic Act No. 6957 ("BOT Law," for brevity), cited by PEA and AMARI as legislative authority to sell reclaimed lands to private parties, recognizes the constitutional ban. Section 6 of RA No. 6957 states –

"Sec. 6. Repayment Scheme. - For the financing, construction, operation and maintenance of any infrastructure projects undertaken through the build-operate-and-transfer arrangement or any of its variations pursuant to the provisions of this Act, the project proponent x x x may likewise be repaid in the form of a share in the revenue of the project or other non-monetary payments, such as, but not limited to, the grant of a portion or percentage of the reclaimed land, subject to the constitutional requirements with respect to the ownership of the land: x x x." (Emphasis supplied)

A private corporation, even one that undertakes the physical reclamation of a government BOT project, cannot acquire reclaimed alienable lands of the public domain in view of the constitutional ban.

Section 302 of the Local Government Code, also mentioned by PEA and AMARI, authorizes local governments in land reclamation projects to pay the

contractor or developer in kind consisting of a percentage of the reclaimed land, to wit:

"Section 302. Financing, Construction, Maintenance, Operation, and Management of Infrastructure Projects by the Private Sector. x x xx x xIn case of land reclamation or construction of industrial estates, the repayment plan may consist of the grant of a portion or percentage of the reclaimed land or the industrial estate constructed."

Although Section 302 of the Local Government Code does not contain a proviso similar to that of the BOT Law, the constitutional restrictions on land ownership automatically apply even though not expressly mentioned in the Local Government Code.

Thus, under either the BOT Law or the Local Government Code, the contractor or developer, if a corporate entity, can only be paid with leaseholds on portions of the reclaimed land. If the contractor or developer is an individual, portions of the reclaimed land, not exceeding 12 hectares of non-agricultural lands, may be conveyed to him in ownership in view of the legislative authority allowing such conveyance. This is the only way these provisions of the BOT Law and the Local Government Code can avoid a direct collision with Section 3, Article XII of the 1987 Constitution.

Registration of lands of the public domain

Finally, PEA theorizes that the "act of conveying the ownership of the reclaimed lands to public respondent PEA transformed such lands of the public domain to private lands." This theory is echoed by AMARI which maintains that the "issuance of the special patent leading to the eventual issuance of title takes the subject land away from the land of public domain and converts the property into patrimonial or private property." In short, PEA and AMARI contend that with the issuance of Special Patent No. 3517 and the corresponding certificates of titles, the 157.84 hectares comprising the Freedom Islands have become private lands of PEA. In support of their theory, PEA and AMARI cite the following rulings of the Court:

1. Sumail v. Judge of CFI of Cotabato, where the Court held –"Once the patent was granted and the corresponding certificate of title was issued, the land ceased to be part of the public domain and became private property over which the Director of Lands has neither control nor jurisdiction."

2. Lee Hong Hok v. David, where the Court declared -

Page 44: Natural Resources Set2.Classification Cases

44NATURAL RESOURCES: CLASSIFICATIONS CASES

"After the registration and issuance of the certificate and duplicate certificate of title based on a public land patent, the land covered thereby automatically comes under the operation of Republic Act 496 subject to all the safeguards provided therein."

3. Heirs of Gregorio Tengco v. Heirs of Jose Aliwalas,where the Court ruled -"While the Director of Lands has the power to review homestead patents, he may do so only so long as the land remains part of the public domain and continues to be under his exclusive control; but once the patent is registered and a certificate of title is issued, the land ceases to be part of the public domain and becomes private property over which the Director of Lands has neither control nor jurisdiction."

4. Manalo v. Intermediate Appellate Court, where the Court held –"When the lots in dispute were certified as disposable on May 19, 1971, and free patents were issued covering the same in favor of the private respondents, the said lots ceased to be part of the public domain and, therefore, the Director of Lands lost jurisdiction over the same."

5.Republic v. Court of Appeals, where the Court stated –"Proclamation No. 350, dated October 9, 1956, of President Magsaysay legally effected a land grant to the Mindanao Medical Center, Bureau of Medical Services, Department of Health, of the whole lot, validly sufficient for initial registration under the Land Registration Act. Such land grant is constitutive of a 'fee simple' title or absolute title in favor of petitioner Mindanao Medical Center. Thus, Section 122 of the Act, which governs the registration of grants or patents involving public lands, provides that 'Whenever public lands in the Philippine Islands belonging to the Government of the United States or to the Government of the Philippines are alienated, granted or conveyed to persons or to public or private corporations, the same shall be brought forthwith under the operation of this Act (Land Registration Act, Act 496) and shall become registered lands.'"

The first four cases cited involve petitions to cancel the land patents and the corresponding certificates of titlesissued to private parties. These four cases uniformly hold that the Director of Lands has no jurisdiction over private lands or that upon issuance of the certificate of title the land automatically comes under the Torrens System. The fifth case cited involves the registration under the Torrens System of a 12.8-hectare public land granted by the National Government to Mindanao Medical Center, a government unit under the Department of Health. The National Government transferred the 12.8-hectare public land to serve as the site for the hospital buildings and other facilities of Mindanao Medical Center, which performed a public service. The Court affirmed the registration of the 12.8-hectare public land in the name of Mindanao Medical Center under Section 122 of Act No. 496. This fifth case is an example of a public land being registered under Act No. 496 without the land losing its character as a property of public dominion.

In the instant case, the only patent and certificates of title issued are those in the name of PEA, a wholly government owned corporation performing public as well as proprietary functions. No patent or certificate of title has been issued to any private party. No one is asking the Director of Lands to cancel PEA's patent or certificates of title. In fact, the thrust of the instant petition is that PEA's certificates of title should remain with PEA, and the land covered by these certificates, being alienable lands of the public domain, should not be sold to a private corporation.

Registration of land under Act No. 496 or PD No. 1529 does not vest in the registrant private or public ownership of the land. Registration is not a mode of acquiring ownership but is merely evidence of ownership previously conferred by any of the recognized modes of acquiring ownership. Registration does not give the registrant a better right than what the registrant had prior to the registration. The registration of lands of the public domain under the Torrens system, by itself, cannot convert public lands into private lands.

Jurisprudence holding that upon the grant of the patent or issuance of the certificate of title the alienable land of the public domain automatically becomes private land cannot apply to government units and entities like PEA. The transfer of the Freedom Islands to PEA was made subject to the provisions of CA No. 141 as expressly stated in Special Patent No. 3517 issued by then President Aquino, to wit:

"NOW, THEREFORE, KNOW YE, that by authority of the Constitution of the Philippines and in conformity with the provisions of Presidential Decree No. 1084, supplemented by Commonwealth Act No. 141, as amended, there are hereby granted and conveyed unto the Public Estates Authority the aforesaid tracts of land containing a total area of one million nine hundred fifteen thousand eight hundred ninety four (1,915,894) square meters; the technical description of which are hereto attached and made an integral part hereof." (Emphasis supplied)

Thus, the provisions of CA No. 141 apply to the Freedom Islands on matters not covered by PD No. 1084. Section 60 of CA No. 141 prohibits, "except when authorized by Congress," the sale of alienable lands of the public domain that are transferred to government units or entities. Section 60 of CA No. 141 constitutes, under Section 44 of PD No. 1529, a "statutory lien affecting title" of the registered land even if not annotated on the certificate of title.104 Alienable lands of the public domain held by government entities under Section 60 of CA No. 141 remain public lands because they cannot be alienated or encumbered unless Congress passes a law authorizing their disposition. Congress, however, cannot authorize the sale to private corporations of reclaimed alienable lands of the public domain because of the constitutional ban. Only individuals can benefit from such law.

The grant of legislative authority to sell public lands in accordance with Section 60 of CA No. 141 does not automatically convert alienable lands of the public

Page 45: Natural Resources Set2.Classification Cases

45NATURAL RESOURCES: CLASSIFICATIONS CASES

domain into private or patrimonial lands. The alienable lands of the public domain must be transferred to qualified private parties, or to government entities not tasked to dispose of public lands, before these lands can become private or patrimonial lands. Otherwise, the constitutional ban will become illusory if Congress can declare lands of the public domain as private or patrimonial lands in the hands of a government agency tasked to dispose of public lands. This will allow private corporations to acquire directly from government agencies limitless areas of lands which, prior to such law, are concededly public lands.

Under EO No. 525, PEA became the central implementing agency of the National Government to reclaim foreshore and submerged areas of the public domain. Thus, EO No. 525 declares that –

"EXECUTIVE ORDER NO. 525

Designating the Public Estates Authority as the Agency Primarily Responsible for all Reclamation Projects

Whereas, there are several reclamation projects which are ongoing or being proposed to be undertaken in various parts of the country which need to be evaluated for consistency with national programs;

Whereas, there is a need to give further institutional support to the Government's declared policy to provide for a coordinated, economical and efficient reclamation of lands;

Whereas, Presidential Decree No. 3-A requires that all reclamation of areas shall be limited to the National Government or any person authorized by it under proper contract;

Whereas, a central authority is needed to act on behalf of the National Government which shall ensure a coordinated and integrated approach in the reclamation of lands;

Whereas, Presidential Decree No. 1084 creates the Public Estates Authority as a government corporation to undertake reclamation of lands and ensure their maximum utilization in promoting public welfare and interests; and

Whereas, Presidential Decree No. 1416 provides the President with continuing authority to reorganize the national government including the transfer, abolition, or merger of functions and offices.

NOW, THEREFORE, I, FERDINAND E. MARCOS, President of the Philippines, by virtue of the powers vested in me by the Constitution and pursuant to Presidential Decree No. 1416, do hereby order and direct the following:

Section 1. The Public Estates Authority (PEA) shall be primarily responsible for integrating, directing, and coordinating all reclamation projects for and on behalf of the National Government. All reclamation projects shall be approved by the President upon recommendation of the PEA, and shall be undertaken by the PEA or through a proper contract executed by it with any person or entity; Provided, that, reclamation projects of any national government agency or entity authorized under its charter shall be undertaken in consultation with the PEA upon approval of the President.

x x x ."

As the central implementing agency tasked to undertake reclamation projects nationwide, with authority to sell reclaimed lands, PEA took the place of DENR as the government agency charged with leasing or selling reclaimed lands of the public domain. The reclaimed lands being leased or sold by PEA are not private lands, in the same manner that DENR, when it disposes of other alienable lands, does not dispose of private lands but alienable lands of the public domain. Only when qualified private parties acquire these lands will the lands become private lands. In the hands of the government agency tasked and authorized to dispose of alienable of disposable lands of the public domain, these lands are still public, not private lands.

Furthermore, PEA's charter expressly states that PEA "shall hold lands of the public domain" as well as "any and all kinds of lands." PEA can hold both lands of the public domain and private lands. Thus, the mere fact that alienable lands of the public domain like the Freedom Islands are transferred to PEA and issued land patents or certificates of title in PEA's name does not automatically make such lands private.

To allow vast areas of reclaimed lands of the public domain to be transferred to PEA as private lands will sanction a gross violation of the constitutional ban on private corporations from acquiring any kind of alienable land of the public domain. PEA will simply turn around, as PEA has now done under the Amended JVA, and transfer several hundreds of hectares of these reclaimed and still to be reclaimed lands to a single private corporation in only one transaction. This scheme will effectively nullify the constitutional ban in Section 3, Article XII of the 1987 Constitution which was intended to diffuse equitably the ownership of alienable lands of the public domain among Filipinos, now numbering over 80 million strong.

Page 46: Natural Resources Set2.Classification Cases

46NATURAL RESOURCES: CLASSIFICATIONS CASES

This scheme, if allowed, can even be applied to alienable agricultural lands of the public domain since PEA can "acquire x x x any and all kinds of lands." This will open the floodgates to corporations and even individuals acquiring hundreds of hectares of alienable lands of the public domain under the guise that in the hands of PEA these lands are private lands. This will result in corporations amassing huge landholdings never before seen in this country - creating the very evil that the constitutional ban was designed to prevent. This will completely reverse the clear direction of constitutional development in this country. The 1935 Constitution allowed private corporations to acquire not more than 1,024 hectares of public lands. The 1973 Constitution prohibited private corporations from acquiring any kind of public land, and the 1987 Constitution has unequivocally reiterated this prohibition.

The contention of PEA and AMARI that public lands, once registered under Act No. 496 or PD No. 1529, automatically become private lands is contrary to existing laws. Several laws authorize lands of the public domain to be registered under the Torrens System or Act No. 496, now PD No. 1529, without losing their character as public lands. Section 122 of Act No. 496, and Section 103 of PD No. 1529, respectively, provide as follows:

Act No. 496"Sec. 122. Whenever public lands in the Philippine Islands belonging to the x x x Government of the Philippine Islands are alienated, granted, or conveyed to persons or the public or private corporations, the same shall be brought forthwith under the operation of this Act and shall become registered lands."PD No. 1529"Sec. 103. Certificate of Title to Patents. Whenever public land is by the Government alienated, granted or conveyed to any person, the same shall be brought forthwith under the operation of this Decree." (Emphasis supplied)

Based on its legislative history, the phrase "conveyed to any person" in Section 103 of PD No. 1529 includes conveyances of public lands to public corporations.

Alienable lands of the public domain "granted, donated, or transferred to a province, municipality, or branch or subdivision of the Government," as provided in Section 60 of CA No. 141, may be registered under the Torrens System pursuant to Section 103 of PD No. 1529. Such registration, however, is expressly subject to the condition in Section 60 of CA No. 141 that the land "shall not be alienated, encumbered or otherwise disposedof in a manner affecting its title, except when authorized by Congress." This provision refers to government reclaimed, foreshore and marshy lands of the public domain that have been titled but still cannot be alienated or encumbered unless expressly authorized by Congress. The need for legislative authority prevents the registered land of the public domain from becoming private land that can be disposed of to qualified private parties.

The Revised Administrative Code of 1987 also recognizes that lands of the public domain may be registered under the Torrens System. Section 48, Chapter 12, Book I of the Code states –

"Sec. 48. Official Authorized to Convey Real Property. Whenever real property of the Government is authorized by law to be conveyed, the deed of conveyance shall be executed in behalf of the government by the following:(1) x x x(2) For property belonging to the Republic of the Philippines, but titled in the name of any political subdivision or of any corporate agency or instrumentality, by the executive head of the agency or instrumentality." (Emphasis supplied)

Thus, private property purchased by the National Government for expansion of a public wharf may be titled in the name of a government corporation regulating port operations in the country. Private property purchased by the National Government for expansion of an airport may also be titled in the name of the government agency tasked to administer the airport. Private property donated to a municipality for use as a town plaza or public school site may likewise be titled in the name of the municipality. All these properties become properties of the public domain, and if already registered under Act No. 496 or PD No. 1529, remain registered land. There is no requirement or provision in any existing law for the de-registration of land from the Torrens System.

Private lands taken by the Government for public use under its power of eminent domain become unquestionably part of the public domain. Nevertheless, Section 85 of PD No. 1529 authorizes the Register of Deeds to issue in the name of the National Government new certificates of title covering such expropriated lands. Section 85 of PD No. 1529 states –

"Sec. 85. Land taken by eminent domain. Whenever any registered land, or interest therein, is expropriated or taken by eminent domain, the National Government, province, city or municipality, or any other agency or instrumentality exercising such right shall file for registration in the proper Registry a certified copy of the judgment which shall state definitely by an adequate description, the particular property or interest expropriated, the number of the certificate of title, and the nature of the public use. A memorandum of the right or interest taken shall be made on each certificate of title by the Register of Deeds, and where the fee simple is taken, a new certificate shall be issued in favor of the National Government, province, city, municipality, or any other agency or instrumentality exercising such right for the land so taken. The legal expenses incident to the memorandum of registration or issuance of a new certificate of title shall be for the account of the authority taking the land or interest therein." (Emphasis supplied)

Page 47: Natural Resources Set2.Classification Cases

47NATURAL RESOURCES: CLASSIFICATIONS CASES

Consequently, lands registered under Act No. 496 or PD No. 1529 are not exclusively private or patrimonial lands. Lands of the public domain may also be registered pursuant to existing laws.

AMARI makes a parting shot that the Amended JVA is not a sale to AMARI of the Freedom Islands or of the lands to be reclaimed from submerged areas of Manila Bay. In the words of AMARI, the Amended JVA "is not a sale but a joint venture with a stipulation for reimbursement of the original cost incurred by PEA for the earlier reclamation and construction works performed by the CDCP under its 1973 contract with the Republic." Whether the Amended JVA is a sale or a joint venture, the fact remains that the Amended JVA requires PEA to "cause the issuance and delivery of the certificates of title conveying AMARI's Land Share in the name of AMARI."107

This stipulation still contravenes Section 3, Article XII of the 1987 Constitution which provides that private corporations "shall not hold such alienable lands of the public domain except by lease." The transfer of title and ownership to AMARI clearly means that AMARI will "hold" the reclaimed lands other than by lease. The transfer of title and ownership is a "disposition" of the reclaimed lands, a transaction considered a sale or alienation under CA No. 141, the Government Auditing Code,109 and Section 3, Article XII of the 1987 Constitution.

The Regalian doctrine is deeply implanted in our legal system. Foreshore and submerged areas form part of the public domain and are inalienable. Lands reclaimed from foreshore and submerged areas also form part of the public domain and are also inalienable, unless converted pursuant to law into alienable or disposable lands of the public domain. Historically, lands reclaimed by the government are sui generis, not available for sale to private parties unlike other alienable public lands. Reclaimed lands retain their inherent potential as areas for public use or public service. Alienable lands of the public domain, increasingly becoming scarce natural resources, are to be distributed equitably among our ever-growing population. To insure such equitable distribution, the 1973 and 1987 Constitutions have barred private corporations from acquiring any kind of alienable land of the public domain. Those who attempt to dispose of inalienable natural resources of the State, or seek to circumvent the constitutional ban on alienation of lands of the public domain to private corporations, do so at their own risk.

We can now summarize our conclusions as follows:

1. The 157.84 hectares of reclaimed lands comprising the Freedom Islands, now covered by certificates of title in the name of PEA, are alienable lands of the public domain. PEA may lease these lands to private corporations but may not sell or transfer ownership of these lands to private corporations. PEA may only

sell these lands to Philippine citizens, subject to the ownership limitations in the 1987 Constitution and existing laws.

2. The 592.15 hectares of submerged areas of Manila Bay remain inalienable natural resources of the public domain until classified as alienable or disposable lands open to disposition and declared no longer needed for public service. The government can make such classification and declaration only after PEA has reclaimed these submerged areas. Only then can these lands qualify as agricultural lands of the public domain, which are the only natural resources the government can alienate. In their present state, the 592.15 hectares of submerged areas are inalienable and outside the commerce of man.

3. Since the Amended JVA seeks to transfer to AMARI, a private corporation, ownership of 77.34 hectares of the Freedom Islands, such transfer is void for being contrary to Section 3, Article XII of the 1987 Constitution which prohibits private corporations from acquiring any kind of alienable land of the public domain.

4. Since the Amended JVA also seeks to transfer to AMARI ownership of 290.156 hectares111 of still submerged areas of Manila Bay, such transfer is void for being contrary to Section 2, Article XII of the 1987 Constitution which prohibits the alienation of natural resources other than agricultural lands of the public domain. PEA may reclaim these submerged areas. Thereafter, the government can classify the reclaimed lands as alienable or disposable, and further declare them no longer needed for public service. Still, the transfer of such reclaimed alienable lands of the public domain to AMARI will be void in view of Section 3, Article XII of the 1987 Constitution which prohibits private corporations from acquiring any kind of alienable land of the public domain.

Clearly, the Amended JVA violates glaringly Sections 2 and 3, Article XII of the 1987 Constitution. Under Article 1409 of the Civil Code, contracts whose "object or purpose is contrary to law," or whose "object is outside the commerce of men," are "inexistent and void from the beginning." The Court must perform its duty to defend and uphold the Constitution, and therefore declares the Amended JVA null and void ab initio.

Seventh issue: whether the Court is the proper forum to raise the issue of whether the Amended JVA is grossly disadvantageous to the government.

Considering that the Amended JVA is null and void ab initio, there is no necessity to rule on this last issue. Besides, the Court is not a trier of facts, and this last issue involves a determination of factual matters.

Page 48: Natural Resources Set2.Classification Cases

48NATURAL RESOURCES: CLASSIFICATIONS CASES

WHEREFORE, the petition is GRANTED. The Public Estates Authority and Amari Coastal Bay Development Corporation are PERMANENTLY ENJOINED from implementing the Amended Joint Venture Agreement which is hereby declared NULL and VOID ab initio. SO ORDERED.

G.R. No. 133250             November 11, 2003FRANCISCO I. CHAVEZ vs. PUBLIC ESTATES AUTHORITY10

R E S O L U T I O NCARPIO, J.:

This Court is asked to legitimize a government contract that conveyed to a private entity 157.84 hectares of reclaimed public lands along Roxas Boulevard in Metro Manila at the negotiated price of P1,200 per square meter. However, published reports place the market price of land near that area at that time at a high of P90,000 per square meter. The difference in price is a staggering P140.16 billion, equivalent to the budget of the entire Judiciary for seventeen years and more than three times the Marcos Swiss deposits that this Court forfeited in favor of the government.

Many worry to death that the private investors will lose their investments, at most not more than one-half billion pesos in legitimate expenses, if this Court voids the contract. No one seems to worry about the more than tens of billion pesos that the hapless Filipino people will lose if the contract is allowed to stand. There are those who question these figures, but the questions arise only because the private entity somehow managed to inveigle the government to sell the reclaimed lands without public bidding in patent violation of the Government Auditing Code.

Fortunately for the Filipino people, two Senate Committees, the Senate Blue Ribbon Committee and the Committee on Accountability of Public Officers, conducted extensive public hearings to determine the actual market value of the public lands sold to the private entity. The Senate Committees established the clear, indisputable and unalterable fact that the sale of the public lands is grossly and unconscionably undervalued based on official documents submitted by the proper government agencies during the Senate investigation. We quote the joint report of these two Senate Committees, Senate Committee Report No. 560, as approved by the Senate in plenary session on 27 September 1997:

The Consideration for the Property

PEA, under the JVA, obligated itself to convey title and possession over the Property, consisting of approximately One Million Five Hundred Seventy Eight

10 Classifications

Thousand Four Hundred Forty One (1,578,441) Square Meters for a total consideration of One Billion Eight Hundred Ninety Four Million One Hundred Twenty Nine Thousand Two Hundred (P1,894,129,200.00) Pesos, or a price of One Thousand Two Hundred (P1,200.00) Pesos per square meter.

According to the zonal valuation of the Bureau of Internal Revenue, the value of the Property is Seven Thousand Eight Hundred Pesos (P7,800.00) per square meter. The Municipal Assessor of Parañaque, Metro Manila, where the Property is located, pegs the market value of the Property at Six Thousand Pesos (P6,000.00) per square meter. Based on these alone, the price at which PEA agreed to convey the property is a pittance. And PEA cannot claim ignorance of these valuations, at least not those of the Municipal Assessors’ office, since it has been trying to convince the Office of the Municipal Assessor of Parañaque to reduce the valuation of various reclaimed properties thereat in order for PEA to save on accrued real property taxes.

PEA’s justification for the purchase price are various appraisal reports, particularly the following:

(1) An appraisal by Vic T. Salinas Realty and Consultancy Services concluding that the Property is worth P500.00 per square meter for the smallest island and P750.00 per square meter for the two other islands, or a total of P1,170,000.00 as of 22 February 1995;(2) An appraisal by Valencia Appraisal Corporation concluding that the Property is worth P850 per square meter for Island I, P800 per square meter for Island II and P600 per square meter for the smallest island, or a total of P1,289,732,000, also as of 22 February 1995; and(3) An Appraisal by Asian Appraisal Company, Inc. (AACI), stating that the Property is worth approximately P1,000 per square meter for Island I, P950 per square meter for Island II and P600 per square meter for Island III, or a total of P1,518,805,000 as of 27 February 1995.

The credibility of the foregoing appraisals, however, are [sic] greatly impaired by a subsequent appraisal report of AACI stating that the property is worth P4,500.00 per square meter as of 26 March 1996. Such discrepancies in the appraised value as appearing in two different reports by the same appraisal company submitted within a span of one year render all such appraisal reports unworthy of even the slightest consideration. Furthermore, the appraisal report submitted by the Commission on Audit estimates the value of the Property to be approximately P33,673,000,000.00, or P21,333.07 per square meter.

There were also other offers made for the property from other parties which indicate that the Property has been undervalued by PEA. For instance, on 06 March 1995, Mr. Young D. See, President of Saeil Heavy Industries Co., Ltd.,

Page 49: Natural Resources Set2.Classification Cases

49NATURAL RESOURCES: CLASSIFICATIONS CASES

(South Korea), offered to buy the property at P1,400.00 and expressed its willingness to issue a stand-by letter of credit worth $10 million. PEA did not consider this offer and instead finalized the JVA with AMARI. Other offers were made on various dates by Aspac Management and Development Group Inc. (for P1,600 per square meter), Universal Dragon Corporation (for P1,600 per square meter), Cleene Far East Manila Incorporated and Hyosan Prime Construction Co. Ltd. which had prepared an Irrevocable Clean Letter of Credit for P100,000,000.

In addition, AMARI agreed to pay huge commissions and bonuses to various persons, amounting to P1,596,863,050.00 (P1,754,707,150.00 if the bonus is included), as will be discussed fully below, which indicate that AMARI itself believed the market value to be much higher than the agreed purchase price. If such commissions are added to the purchase price, AMARI’s acquisition cost for the Property will add-up to P3,490,992,250.00 (excluding the bonus). If AMARI was willing to pay such amount for the Property, why was PEA willing to sell for only P1,894,129,200.00, making the Government stand to lose approximately P1,596,863,050.00?

x       x       x

Even if we simply assume that the market value of the Property is half of the market value fixed by the Municipal Assessors Office of Parañaque for lands along Roxas Boulevard, or P3,000.00 per square meter, the Government now stands to lose approximately P2,841,193,800.00. But an even better assumption would be that the value of the Property is P4,500.00 per square meter, as per the AACI appraisal report dated 26 March 1996, since this is the valuation used to justify the issuance of P4 billion worth of shares of stock of Centennial City Inc. (CCI) in exchange for 4,800,000 AMARI shares with a total par value of only P480,000,000.00. With such valuation, the Government’s loss will amount to P5,208,855,300.00.

Clearly, the purchase price agreed to by PEA is way below the actual value of the Property, thereby subjecting the Government to grave injury and enabling AMARI to enjoy tremendous benefit and advantage. (Emphasis supplied)

The Senate Committee Report No. 560 attached the following official documents from the Bureau of Internal Revenue, the Municipal Assessor of Parañaque, Metro Manila, and the Commission on Audit:

1. Annex "M," Certified True Copy of BIR Zonal Valuations as certified by Antonio F. Montemayor, Revenue District Officer. This official document fixed the market value of the 157.84 hectares at P7,800 per square meter.2. Annex "N," Certification of Soledad S. Medina-Cue, Municipal Assessor, Parañaque, dated 10 December 1996. This official document fixed the market value at P6,000 per square meter.3. Exhibit "1-Engr. Santiago," the Appraisal Report of the Commission on Audit. This official document fixed the market value at P21,333.07 per square meter.

Whether based on the official appraisal of the BIR, the Municipal Assessor or the Commission on Audit, the P1,200 per square meter purchase price, or a total of P1.894 billion for the 157.84 hectares of government lands, is grossly and unconscionably undervalued. The authoritative appraisal, of course, is that of the Commission on Audit which valued the 157.84 hectares at P21,333.07 per square meter or a total of P33.673 billion. Thus, based on the official appraisal of the Commission on Audit, the independent constitutional body that safeguards government assets, the actual loss to the Filipino people is a shocking P31.779 billion.

This gargantuan monetary anomaly, aptly earning the epithet "Grandmother of All Scams,"4 is not the major defect of this government contract. The major flaw is not even the P1.754 billion in commissions the Senate Committees discovered the private entity paid to various persons to secure the contract, described in Senate Report No. 560 as follows:

A Letter-Agreement dated 09 June 1995 signed by Messrs. Premchai Karnasuta and Emmanuel Sy for and in behalf of AMARI, on the one hand, and stockholders of AMARI namely, Mr. Chin San Cordova (a.k.a. Benito Co) and Mr. Chua Hun Siong (a.k.a. Frank Chua), on the other, sets forth various paymentsAMARI paid or agreed to pay the aforesaid stockholders by way of fees for "professional efforts and services in successfully negotiating and

securing for AMARI the Joint Venture Agreement",as follows:

Form of Payment Paid/Payable On Amount

Manager’s Checks 28 April 1995 P 400,000,000.00

Manager’s Checks Upon signing of letter 262,500,000.00

10 Post Dated Checks (PDCs)

60 days from date of letter 127,000,000.00

24 PDCs 31 Aug. ’95 to 31 Jan. ’98 150,000,000.00

48 PDCs Monthly, over a 12-month pd. 357,363,050.00

Page 50: Natural Resources Set2.Classification Cases

50NATURAL RESOURCES: CLASSIFICATIONS CASES

from date of letter

Cash bonus When sale of land begins

not exceeding

    157,844,100.00

Developed land from Project

Upon completion of each phase

Costing

        300,000,000.00

  TOTAL P1,754,707,150.00

    ==============

Mr. Luis Benitez of SGV, the external auditors of AMARI, testified that said Letter-Agreement was approved by the AMARI Board.6 (Emphasis supplied)

The private entity that purchased the reclaimed lands for P1.894 billion expressly admitted before the Senate Committees that it spent P1.754 billion in commissions to pay various individuals for "professional efforts and services in successfully negotiating and securing" the contract. By any legal or moral yardstick, the P1.754 billion in commissions obviously constitutes bribe money. Nonetheless, there are those who insist that the billions in investments of the private entity deserve protection by this Court. Should this Court establish a new doctrine by elevating grease money to the status of legitimate investments deserving of protection by the law? Should this Court reward the patently illegal and grossly unethical business practice of the private entity in securing the contract? Should we allow those with hands dripping with dirty money equitable relief from this Court?

Despite these revolting anomalies unearthed by the Senate Committees, the fatal flaw of this contract is that it glaringly violates provisions of the Constitution expressly prohibiting the alienation of lands of the public domain.

Thus, we now come to the resolution of the second Motions for Reconsideration7 filed by public respondent Public Estates Authority ("PEA") and private respondent Amari Coastal Bay Development Corporation ("Amari"). As correctly pointed out by petitioner Francisco I. Chavez in his Consolidated Comment,8 the second Motions for Reconsideration raise no new issues.

However, the Supplement to "Separate Opinion, Concurring and Dissenting" of Justice Josue N. Bellosillo brings to the Court’s attention the Resolutions of this Court on 3 February 1965 and 24 June 1966 in L- 21870 entitled"Manuel O. Ponce, et al. v. Hon. Amador Gomez, et al." and No. L-22669 entitled "Manuel O. Ponce, et al. v. The City of Cebu, et al." ("Ponce Cases"). In effect, the Supplement to the Dissenting Opinion claims that these two Resolutions serve as authority that a single private corporation like Amari may acquire hundreds of hectares of submerged lands, as well as reclaimed submerged lands, within Manila Bay under the Amended Joint Venture Agreement ("Amended JVA").

We find the cited Ponce Cases inapplicable to the instant case.

First, as Justice Bellosillo himself states in his supplement to his dissent, the Ponce Cases admit that"submerged lands still belong to the National Government.  The correct formulation, however, is thatsubmerged lands are owned by the State and are inalienable. Section 2, Article XII of the 1987 Constitution provides:

All lands of the public domain, waters, minerals, coal, petroleum, and other mineral oils, all forces of potential energy, fisheries, forests or timber, wildlife, flora and fauna, and other natural resources are owned by the State. With the exception of agricultural lands, all other natural resources shall not be alienated. x x x. (Emphasis supplied)

Submerged lands, like the waters (sea or bay) above them, are part of the State’s inalienable natural resources. Submerged lands are property of public dominion, absolutely inalienable and outside the commerce of man.This is also true with respect to foreshore lands. Any sale of submerged or foreshore lands is void being contrary to the Constitution.

This is why the Cebu City ordinance merely granted Essel, Inc. an "irrevocable option" to purchase the foreshore lands after the reclamation and did not actually sell to Essel, Inc. the still to be reclaimed foreshore lands. Clearly, in the Ponce Cases the option to purchase referred to reclaimed lands, and not to foreshore lands which are inalienable. Reclaimed lands are no longer foreshore or submerged lands, and thus may qualify as alienable agricultural lands of the public domain provided the requirements of public land laws are met.

In the instant case, the bulk of the lands subject of the Amended JVA are still submerged lands even to this very day, and therefore inalienable and outside the commerce of man. Of the 750 hectares subject of the Amended JVA, 592.15 hectares or 78% of the total area are still submerged, permanently under the waters of Manila Bay. Under the Amended JVA, the PEA conveyed to Amari the

Page 51: Natural Resources Set2.Classification Cases

51NATURAL RESOURCES: CLASSIFICATIONS CASES

submerged lands even before their actual reclamation, although the documentation of the deed of transfer and issuance of the certificates of title would be made only after actual reclamation.

The Amended JVA states that the PEA "hereby contributes to the Joint Venture its rights and privileges to perform Rawland Reclamation and Horizontal Development as well as own the Reclamation Area."12 The Amended JVA further states that "the sharing of the Joint Venture Proceeds shall be based on the ratio of thirty percent (30%) for PEA and seventy percent (70%) for AMARI."13 The Amended JVA also provides that the PEA "hereby designates AMARI to perform PEA’s rights and privileges to reclaim, own and develop the Reclamation Area."14 In short, under the Amended JVA the PEA contributed its rights, privileges and ownership over the Reclamation Area to the Joint Venture which is 70% owned by Amari. Moreover, the PEA delegated to Amari the right and privilege to reclaim the submerged lands.

The Amended JVA mandates that the PEA had "the duty to execute without delay the necessary deed of transfer or conveyance of the title pertaining to AMARI’s Land share based on the Land Allocation Plan."15 The Amended JVA also provides that "PEA, when requested in writing by AMARI, shall then cause the issuance and delivery of the proper certificates of title covering AMARI’s Land Share in the name of AMARI, x x x."

In the Ponce Cases, the City of Cebu retained ownership of the reclaimed foreshore lands and Essel, Inc. only had an "irrevocable option" to purchase portions of the foreshore lands once actually reclaimed. In sharp contrast, in the instant case ownership of the reclamation area, including the submerged lands, was immediately transferred to the joint venture. Amari immediately acquired the absolute right to own 70% percent of the reclamation area, with the deeds of transfer to be documented and the certificates of title to be issued upon actual reclamation. Amari’s right to own the submerged lands is immediately effective upon the approval of the Amended JVA and not merely an option to be exercised in the future if and when the reclamation is actually realized. The submerged lands, being inalienable and outside the commerce of man, could not be the subject of the commercial transactions specified in the Amended JVA.

Second, in the Ponce Cases the Cebu City ordinance granted Essel, Inc. an "irrevocable option" to purchase from Cebu City not more than 70% of the reclaimed lands. The ownership of the reclaimed lands remained with Cebu City until Essel, Inc. exercised its option to purchase. With the subsequent enactment of the Government Auditing Code (Presidential Decree No. 1445) on 11 June 1978, any sale of government land must be made only through public bidding. Thus, such an "irrevocable option" to purchase government land would now be void being contrary to the requirement of public bidding expressly required in Section 79 of PD No. 1445.

This requirement of public bidding is reiterated in Section 379 of the 1991 Local Government Code. Obviously, the ingenious reclamation scheme adopted in the Cebu City ordinance can no longer be followed in view of the requirement of public bidding in the sale of government lands. In the instant case, the Amended JVA is a negotiated contract which clearly contravenes Section 79 of PD No. 1445.

Third, Republic Act No. 1899 authorized municipalities and chartered cities to reclaim foreshore lands. The two Resolutions in the Ponce Cases upheld the Cebu City ordinance only with respect to foreshore areas, and nullified the same with respect to submerged areas. Thus, the 27 June 1965 Resolution made the injunction of the trial court against the City of Cebu "permanent insofar x x x as the area outside or beyond the foreshore land proper is concerned."

As we held in the 1998 case of Republic Real Estate Corporation v. Court of Appeals, citing the Ponce Cases, RA No. 1899 applies only to foreshore lands, not to submerged lands. In his concurring opinion inRepublic Real Estate Corporation, Justice Reynato S. Puno stated that under Commonwealth Act No. 141, "foreshore and lands under water were not to be alienated and sold to private parties," and that such lands "remained property of the State." Justice Puno emphasized that "Commonwealth Act No. 141 has remained in effect at present." The instant case involves principally submerged lands within Manila Bay. On this score, the Ponce Cases, which were decided based on RA No. 1899, are not applicable to the instant case.

Fourth, the Ponce Cases involve the authority of the City of Cebu to reclaim foreshore areas pursuant to a general law, RA No. 1899. The City of Cebu is a public corporation and is qualified, under the 1935, 1973, and 1987 Constitutions, to hold alienable or even inalienable lands of the public domain. There is no dispute that a public corporation is not covered by the constitutional ban on acquisition of alienable public lands. Both the 9 July 2002 Decision and the 6 May 2003 Resolution of this Court in the instant case expressly recognize this.

Cebu City is an end user government agency, just like the Bases Conversion and Development Authority or the Department of Foreign Affairs. Thus, Congress may by law transfer public lands to the City of Cebu to be used for municipal purposes, which may be public or patrimonial. Lands thus acquired by the City of Cebu for a public purpose may not be sold to private parties. However, lands so acquired by the City of Cebu for a patrimonial purpose may be sold to private parties, including private corporations.

However, in the instant case the PEA is not an end user agency with respect to the reclaimed lands under the Amended JVA. As we explained in the 6 May 2003 Resolution:

Page 52: Natural Resources Set2.Classification Cases

52NATURAL RESOURCES: CLASSIFICATIONS CASES

PEA is the central implementing agency tasked to undertake reclamation projects nationwide. PEA took the place of the Department of Environment and Natural Resources ("DENR" for brevity) as the government agency charged with leasing or selling all reclaimed lands of the public domain. In the hands of PEA, which took over the leasing and selling functions of DENR, reclaimed foreshore (or submerged lands) lands are public lands in the same manner that these same lands would have been public lands in the hands of DENR. (Emphasis supplied)

Our 9 July 2002 Decision explained the rationale for treating the PEA in the same manner as the DENR with respect to reclaimed foreshore or submerged lands in this wise:

To allow vast areas of reclaimed lands of the public domain to be transferred to PEA as private lands will sanction a gross violation of the constitutional ban on private corporations from acquiring any kind of alienable land of the public domain. PEA will simply turn around, as PEA has now done under the Amended JVA, and transfer several hundreds of hectares of these reclaimed and still to be reclaimed lands to a single private corporation in only one transaction. This scheme will effectively nullify the constitutional ban in Section 3, Article XII of the 1987 Constitution which was intended to diffuse equitably the ownership of alienable lands of the public domain among Filipinos, now numbering over 80 million strong. (Emphasis supplied)

Finally, the Ponce Cases were decided under the 1935 Constitution which allowed private corporations to acquire alienable lands of the public domain. However, the 1973 Constitution prohibited private corporations from acquiring alienable lands of the public domain, and the 1987 Constitution reiterated this prohibition. Obviously, the Ponce Cases cannot serve as authority for a private corporation to acquire alienable public lands, much less submerged lands, since under the present Constitution a private corporation like Amari is barred from acquiring alienable lands of the public domain.

Clearly, the facts in the Ponce Cases are different from the facts in the instant case. Moreover, the governing constitutional and statutory provisions have changed since the Ponce Cases were disposed of in 1965 and 1966 through minute Resolutions of a divided (6 to 5) Court.

This Resolution does not prejudice any innocent third party purchaser of the reclaimed lands covered by the Amended JVA. Neither the PEA nor Amari has sold any portion of the reclaimed lands to third parties. Title to the reclaimed lands remains with the PEA. As we stated in our 9 July 2002 Decision:

In the instant case, the only patent and certificates of title issued are those in the name of PEA, a wholly government owned corporation performing public as well as proprietary functions. No patent or certificate of title has been issued to any private party. No one is asking the Director of Lands to cancel PEA’s patent or certificates of title. In fact, the thrust of the instant petition is that PEA’s certificates of title should remain with PEA, and the land covered by these certificates, being alienable lands of the public domain, should not be sold to a private corporation.

As we held in our 9 July 2002 Decision, the Amended JVA "violates glaringly Sections 2 and 3, Article XII of the 1987 Constitution." In our 6 May 2003 Resolution, we DENIED with FINALITY respondents’ Motions for Reconsideration. Litigations must end some time. It is now time to write finis to this "Grandmother of All Scams."

WHEREFORE, the second Motions for Reconsideration filed by Public Estates Authority and Amari Coastal Bay Development Corporation are DENIED for being prohibited pleadings. In any event, these Motions for Reconsideration have no merit. No further pleadings shall be allowed from any of the parties. SO ORDERED.

G.R. No. 130906 February 11, 1999REPUBLIC OF THE PHILIPPINES vs. FELIX S. IMPERIAL JR.11

DAVIDE, JR., C.J.:

In this petition for review on certiorari, petitioner seeks to reverse and set aside the (1) Resolution  of 30 July 1997 of the Court Appeals in CA-G.R. CV No. 53972 granting petitioner until 11 August 1997 within which to file its appellant's brief, and the (2) Resolution  of 29 September 1997 dismissing petitioner's appeal. The appeal was taken from the Order  of Branch I,. Regional Trial Court of Legaspi City in Civil Case No. 9176, which petitioner instituted to cancel the title to some lots issued to private respondents for the reversion thereof to the mass of the public domain.

The facts of the case, as found by the trial court, are as follows:

On September 12, 1917, the late Elias Imperial was issued Original. Certificate of Title (OCT) 408 (500) pursuant to Decree No. 55173 of the then Court of First Instance of Albay, covering a parcel of land identified as Lot No. 1113 of the Cadastral Survey of Legazpi, G.L. Cad. Rec. No. 88, containing an area of fifty eight thousand and twenty six square meters (58,026), more or less, situated in Legazpi City.

11 Classifications

Page 53: Natural Resources Set2.Classification Cases

53NATURAL RESOURCES: CLASSIFICATIONS CASES

Original Certificate of Title No. 408 (500) was subdivided and further subdivided resulting in the issuance of several titles, which are now the subject of this case, in the name of the following defendants:

TCT NO. LOT NO. AREA(sq. m.) REGISTERED OWNER1. 978 1113-M-3 5,853 Ellias S. Imperial2. 31054 1113-M-4-A 1,200 Felix S. Imperial3. 213055 1113-M-4-B 4,653 Felix S. Imperial4. 35508 1113-M-2-A 1,355 EANCRA CORPORATION5. 35509 1113-M-2-B 4,518 Feliza S. Imperial6. 35213 1113-M-1-A 1,500 Lolita Alcazar andSalvador Alcazar7. 35214 1113-M-1-B 4,353 Merriam S. Imperial

The plaintiff seeks to judicially declare the transfer certificate of titles described in the preceding paragraphs null and void; to order the said defendants to surrender the owner's duplicate of their aforesaid titles to the Register of Deeds of Legazpi City and directing [sic] the latter to cancel them as well as the originals thereof and to declare the reversion of the lots covered by the aforesaid titles to the mass of the public domain.

In support of its stand, the plaintiff contends among others that on letter request addressed to the Honorable Solicitor General datedMarch 20, 1994, residents of Purok No. 1 and Bgy. 24, Legazpi City, represented by Antonio F. Aguilar, requested that Original Certificate of Title No. 408 (500) in the name of Elias Imperial be canceled and the land covered thereby reverted back to the State on the ground that the land subject thereof is a foreshore land. Subsequent investigation conducted by the Department of Environment and Natural Resources (DENR), Region V, Legazpi City, upon the request of the Office of the Solicitor General (OSG) disclosed that OCT No. 408 (500), from whence the transfer certificate of titles of the defendants were derived is null and void, and was, thus, acquired to the prejudice of the State, considering that:

a. the parcel of land covered by OCT No. 408 (500) has the features of a foreshore land;b. natural ground plants such as mangroves and nipas thrive on certain portions of the land in question;c. some portions of the same land are permanently submerged in seawater even at low tide;d. some portions of the same land are not anymore inundated by seawater due to the considerable amount of improvements built thereon and the placing of boulders and other land-filling materials by the actual residents therein.

The plaintiff alleged that consequently on the basis of said findings, the Director, Lands Management Bureau recommended to the Director, Lands Services, DENR, the cancellation of OCT No. 406 [sic] (500) as well as its derivative titles through appropriate proceedings.

The plaintiff contended that since the land in question is a foreshore land, the same cannot be registered under the Land Registration Act (Act No. 496, now P.D. No. 1529) in the name of private persons since it is non-alienable and belongs to the public domain, administered and managed by the State for .the benefit of the general public.

The plaintiff further contended that under Public Land Act No. 141, as amended, such land shall be disposed of to private parties by lease only and not otherwise as soon as the President upon recommendation of the Secretary of Agriculture and Natural Resources, now DENR, shall declare that the same are not necessary for public services and are open to disposition.

Within the time for pleading, defendants EANCRA Corporation, Lolita Alcazar and Salvador Alcazar filed their answer with cross-claim, while the rest of the defendants, namely, Felix S. Imperial Jr., Feliza S. Imperial, Elias S. Imperial and Miriam S. Imperial filed a motion to dismiss.

The aforesaid motion to dismiss was anchored on the following grounds: (a) the lands covered by the defendants' transfer certificate of titles which were derived from OCT No. 408 (500) was already the subject of the cadastral proceedings in 1917 and which has been implemented by the issuance of OCT No. 408 (500) under the Torrens system.

The adjudication by the cadastral court is binding against the whole world including the plaintiff since cadastral proceedings are in rem and the government itself through the Director of Lands instituted the proceedings and is a direct and active participant. OCT No. 408 (500) issued under the Torrens system has long become incontrovertible after the lapse of one year from the entry of decree of registration; (b) OCT No. 408 (500) was judicially reconstituted in 1953 in accordance with Republic Act [No.] 26 in the then Court of First Instance of Albay, by Jose R. Imperial Samson in Court Case No. RT-305, entitled. The Director of Lands vs. Jose R. Imperial Samson. The proceedings in the judicial reconstitution in said case No. RT-305 is one in rem and has long become final and gave rise to res judicata and therefore can no longer legally be assailed; (c) the findings of the Director of Lands dated February 22, 1983 [sic] from which no appeal was taken in said administrative investigation that Lot No. 1113, Cad. 27 and a portion of it covered by Lot No. 1113-M-5 in the name of Jose Baritua cannot be considered as part of the shore or foreshore of Albay Gulf. This finding of the Director of Lands has become final and thus constitute res judicata, and

Page 54: Natural Resources Set2.Classification Cases

54NATURAL RESOURCES: CLASSIFICATIONS CASES

finally moving defendants contended that several interrelated cases have been decided related to OCT No. 408 (500), specifically Civil Cases Nos. 6556, 6885, 6999 and 7104, all of the Regional Trial Court, Legazpi City which have been brought by several squatters [sic] family against Jose Baritua attacking the latter's title over Lot No. 1113-M-5 which was derived from OCT No. 408 (500) which cases were all decided in favor of Jose Baritua, hence, the decisions rendered therein have become final and executory and constitute res judicata.

The plaintiff through the Office of the Solicitor General filed an objection to the motion to dismiss based on the following grounds: (1) the purported decision issued by the Court of First Instance of Albay in G.R. Cad. Rec. No. 88 supposedly resulting in the issuance of OCT No. 408 (500) pursuant to Decree No. 55173 does not constitute res judicata to the present case; (2) the incontestable and indisputable character of a Torrens certificate of title does not apply when the land thus covered, like foreshore land, is not capable of registration; (3) a certificate of title judicially reconstituted from a void certificate of title is, likewise, void; (4) administrative investigation conducted by the Director of Lands is not a bar to the filing of reversion suits; and (5) the filing of the motion to dismiss carries with it the admission of the truth of all material facts of the complaint 4

After hearing the motion to dismiss, or on 9 August 1996, the trial court dismissed the complaint on the ground that the judgment rendered by the cadastral court in G.R. Cad. Rec. No. 88 and our resolution in the petition to quiet title, G.R. No. 85770, both decreed that the parcel of land covered by OCT No. 408 (500) was not foreshore. The 1917 cadastral proceeding was binding upon the government, which had initiated the same and had been an active and direct participant thereon. Likewise, the 1982 petition to cancel OCT No. 408 (500) filed by the claimants of Lot No. 1113, Cad-47, and resolved by the Director of Lands in his 22 February 1984 letter 5to the effect that "Original Certificate of Title No. 408 (500) 2113 in the name of Elias Imperial and its derivative title[s] were legally issued" was res juridicata to the instant case. Petitioner's contention that the judicially reconstituted certificate of title was void since the land covered by OCT No. 408 (500) was foreshore land was a mere assumption contrary to existing physical facts. The court further considered as forum shopping petitioner's attempt to seek a favorable opinion after it was declared in related cases questioning the title of a certain Jose Baritua, which was also derived from OCT NO. 408(500), that the land in question was foreshore.

On 28 October 1996, petitioner filed a notice of appeal.

On 18 April 1997, the Court of Appeals required petitioner to file its appellant's brief within forty-five (45) days from receipt of the notice. Petitioner received said notice ten (10) days later, or on 28 April 1997.

Due to the alleged heavy workload of the solicitor assigned to the case, petitioner moved for an extension of thirty (30) days from 12 June 1997, or until 12 July 1997, within which to file the appellant's brief. The Court of Appeals granted petitioner's motion for extension in a resolution dated 26 June 1997.

On the same ground of continuing heavy pressure of work, petitioner filed, on 12 July 1997, its second motion for extension of thirty (30) days or 11 August 1997 within which to file the appellant's brief.

On 11 August 1997, petitioner asked for a third extension of thirty (30) days, or until 10 September 1997, within which to file appellant's brief citing the same ground of heavy pressure of work.

Meanwhile, on 30 July 1997, the Court of Appeals issued a resolution, the full text of which reads:

The Office of the Solicitor General is GRANTED a LAST EXTENSION of thirty (30) days from July 12, 1997, or until August 11, 1997, within which to file the oppositor-appellant's brief. Failure to file said brief within the said period will mean dismissal of the appeal. 

On 12 August 1997, petitioner received a copy of the aforesaid resolution.

On 26 August 1997, petitioner moved to reconsider the 30 July 1997 resolution and, despite the appellate court's warning, reiterated its third motion for extension of another thirty (30) days to file the appellant's brief.

On 10 September 1997, petitioner filed a manifestation and motion requesting another extension of five (5) days, or until 15 September 1997, within which to file appellant's brief, reasoning that the brief, although finalized, was yet to be signed by the Solicitor General.

On 15 September 1997, petitioner filed the required appellant's brief.

On 29 September 1997, the Court Of Appeals denied petitioner's motion for reconsideration for lack of merit and sustained its Resolution of 30 July 1997 dismissing the case for failure to file the appellant's brief within the extended period.

Hence, petitioner filed this petition assailing the dismissal of its appeal on purely technical grounds. It alleges that it "has raised meritorious grounds in support of its appeal which, if not allowed to be laid down before the proper Court, will result to the prejudice of, and irreparable injury to, public interest, as the Government would lose

Page 55: Natural Resources Set2.Classification Cases

55NATURAL RESOURCES: CLASSIFICATIONS CASES

opportunity to recover what it believes to be non-registrable lands of the public domain." Minor lapses in adherence to procedural rules should be condoned in order not to frustrate the ends of justice. Thus, petitioner begs indulgence, enumerating the cases that had occupied its time and attention which prevented the filing of the required brief within the extended periods granted by the Courts of Appeals.

Petitioner maintains that our resolution of 8 May 1989 in G.R. No. 85770 entitled "Spouses Espiritu v. Baritua" does not constitute res judicata to the instant case because there is no identity of parties, causes of action, and subject matter between the two cases. The Supreme Court case was instituted by Spouses Jose and Maura Espiritu and others against Jose Baritua, while the instant case was filed by no less than the Republic of the Philippines against herein respondents. The former arose from a proceeding to quiet title, while the latter is an action for reversion.

Anent the "unappealed letter-decision" of the Director of Lands, petitioner contends that the same was "reversible mistake" which did not bar the filing of a reversion suit, as the government is never estopped by the mistakes of its officials or agents.

Petitioner also argues that the 1953 reconstitution case only involved the restoration of the title which case only involved supposed to have been lost or destroyed. The issue as to the nature of the land covered by OCT No. 408 (500) was never delved into by the court. Petitioner insists that the parcels of land in question are foreshore lands, and hence, inalienable and incapable of registration. Consequently, the certificates of title covering said land are void ab initio.

As regards the trial court's finding of forum shopping, petitioner asserts that the same is without basis. It is the first time that petitioner instituted an action against herein respondents concerning the lands in. question.

On the other hand, respondents maintain that the dismissal of the appeal for failure to file brief on time was not an abuse of discretion on the part of the Court of Appeals. Petitioner failed to present special circumstances or good reasons to justify its motions for extension. Moreover, that the parcels of land involved the 1917 cadastral and 1953 reconstitution proceedings. This finding attained finality through our resolution in the (G.R. No. 85770), and was further affirmed through the administrative investigation conducted by the Director of Lands. Thus, the instant case is now barred by res judicata.

We have long observed that the Office of the Solicitor General (OSG) regularly presents motions for extension of time to file pleadings, taking for granted the court's leniency in granting the same. Instead of contributing to the swift administration of justice as an instrumentality of the State, the OSG contributes to

needless delays in litigation. Despite the numerous cases that need the OSG's time and attention, equal importance should be allotted to each and every case. Deadlines must be respected and court warnings not taken lightly.

However, after a thorough reexamination of this case, we are of the view that the challenged resolutions should be reconsidered.

The rules of court governing practice and procedure were formulated in order to promote just, speedy, and inexpensive disposition of every action or proceeding without sacrificing substantial justice and equity considerations. 

The filing of appellant's brief in appeals is not a jurisdictional requirement. Nevertheless, an appeal may be dismissed by the Court of Appeals on its own motion or on that of the appellee upon failure of the appellant to serve and file the required number of copies of the brief within the time provided, 

If the appeal brief cannot be filed on time, extension of time may be allowed provided (1) there is good and sufficient cause, and (2) the motion for extension is filed before the expiration of the time sought to be extended. The court's liberality on extensions notwithstanding, lawyers should never presume that their motions for extension would be granted as a matter of course or for the length of time sought; their concession lies in the sound discretion of the Court exercised in accordance with the attendant circumstances. 

What constitutes good and sufficient cause that will merit suspension of the rules is discretionary upon the court. The court has the power to relax or suspend the rules except a case from their operation when compelling reasons so warrant or when the purpose of justice requires it.  Among the reasons which the court allowed in suspending application of the rules on filing an appeal brief were the following: (1) the cause for the delay was not entirely attributable to the fault or negligence of the party favored by the suspension of the rules;  (2) there was no objection from the State,  and the brief was filed within the period requested; (3) no material injury was suffered by the appellee by reason of the delay in filing the brief;  (4) the fake lawyer failed to file the brief; (5) appellant was represented by counsel de oficio; 16 (6) petitioner's original counsel died;  and (7) the preparation of the consolidated brief involved a comparative study of many exhibits. 

At the core of the controversy is whether the parcels of land in question are foreshore lands. Foreshore land is a part of the alienable land of the public domain and may be disposed of only by lease and not otherwise. It was defined as "that part (of the land) which is between high and low water and left dry by the flux and reflux of the tides."  It is also known as "a strip of land that lies between the high and low water marks and, is alternatively wet and dry according to the flow of the tide." 

Page 56: Natural Resources Set2.Classification Cases

56NATURAL RESOURCES: CLASSIFICATIONS CASES

The classification of public lands is a function of the executive branch of government, specifically the director of lands (now the director of the Lands Management Bureau). The decision of the director of lands when approved by the Secretary of the Department of Environment and Natural Resources (DENR)  as to questions of fact is conclusive upon the court. The principle behind this ruling is that the subject has been exhaustively weighed and discussed and must therefore be given credit. This doctrine finds no application, however, when the decision of the director of lands is revoked by, or in conflict with that of, the DENR Secretary. 

There is allegedly a conflict between the findings of the Director of Lands and the DENR, Region V, in the present case. Respondents contend that the Director of Lands found Jose Baritua's land covered by TCT No.18655, which stemmed from OCT 408(500), to be "definitely outside of the foreshore area." 23 Petitioner, on the other hand, claims that subsequent investigation of the DENR, Region V, Legazpi City, disclosed that the land covered by OCT No. 408 (500) from whence the titles were derived "has the features of a foreshore land." 24The contradictory views of the Director of Lands and the DENR, Region V, Legazpi City, on the true nature of the land, which contradiction was neither discussed nor resolved by the RTC, cannot be the premise of any conclusive classification of the land involved.

The need, therefore, to determine once and for all whether the lands subject of petitioner's reversion efforts are foreshore lands constitutes good and sufficient cause for relaxing procedural rules and granting the third and fourth motions for extension to file appellant's brief. Petitioner's appeal presents an exceptional circumstance impressed with public interest and must then be given due course.

WHEREFORE, the instant petition is hereby GRANTED; the Resolutions of 30 July 1997 and 29 September 1997 of the Court of Appeals are SET ASIDE; petitioner's appeal is reinstated; and the instant case is REMANDED to the Court of Appeals for further proceedings. SO ORDERED.

G.R. No. L-43938 April 15, 1988REPUBLIC OF THE PHILIPPINES vs. HON. COURT OF APPEALS 12 

12 Classifications

CRUZ, J.:

The Regalian doctrine reserves to the State all natural wealth that may be found in the bowels of the earth even if the land where the discovery is made be private. 1 In the cases at bar, which have been consolidated because they pose a common issue, this doctrine was not correctly applied.

These cases arose from the application for registration of a parcel of land filed on February 11, 1965, by Jose de la Rosa on his own behalf and on behalf of his three children, Victoria, Benjamin and Eduardo. The land, situated in Tuding, Itogon, Benguet Province, was divided into 9 lots and covered by plan Psu-225009. According to the application, Lots 1-5 were sold to Jose de la Rosa and Lots 6-9 to his children by Mamaya Balbalio and Jaime Alberto, respectively, in 1964. 

The application was separately opposed by Benguet Consolidated, Inc. as to Lots 1-5, Atok Big Wedge Corporation, as to Portions of Lots 1-5 and all of Lots 6-9, and by the Republic of the Philippines, through the Bureau of Forestry Development, as to lots 1-9. 

In support of the application, both Balbalio and Alberto testified that they had acquired the subject land by virtue of prescription Balbalio claimed to have received Lots 1-5 from her father shortly after the Liberation. She testified she was born in the land, which was possessed by her parents under claim of ownership.  Alberto said he received Lots 6-9 in 1961 from his mother, Bella Alberto, who declared that the land was planted by Jaime and his predecessors-in-interest to bananas, avocado, nangka and camote, and was enclosed with a barbed-wire fence. She was corroborated by Felix Marcos, 67 years old at the time, who recalled the earlier possession of the land by Alberto's father.  Balbalio presented her tax declaration in 1956 and the realty tax receipts from that year to 1964,  Alberto his tax declaration in 1961 and the realty tax receipts from that year to 1964. 

Benguet opposed on the ground that the June Bug mineral claim covering Lots 1-5 was sold to it on September 22, 1934, by the successors-in-interest of James Kelly, who located the claim in September 1909 and recorded it on October 14, 1909. From the date of its purchase, Benguet had been in actual, continuous and exclusive possession of the land in concept of owner, as evidenced by its construction of adits, its affidavits of annual assessment, its geological mappings, geological samplings and trench side cuts, and its payment of taxes on the land. 

For its part, Atok alleged that a portion of Lots 1-5 and all of Lots 6-9 were covered by the Emma and Fredia mineral claims located by Harrison and Reynolds on December 25, 1930, and recorded on January 2, 1931, in the office of the mining recorder of Baguio. These claims were purchased from these locators on November

Page 57: Natural Resources Set2.Classification Cases

57NATURAL RESOURCES: CLASSIFICATIONS CASES

2, 1931, by Atok, which has since then been in open, continuous and exclusive possession of the said lots as evidenced by its annual assessment work on the claims, such as the boring of tunnels, and its payment of annual taxes thereon. 

The location of the mineral claims was made in accordance with Section 21 of the Philippine Bill of 1902 which provided that:

SEC. 21. All valuable mineral deposits in public lands in the philippine Islands both surveyed and unsurveyed are hereby declared to be free and open to exploration, occupation and purchase and the land in which they are found to occupation and purchase by the citizens of the United States, or of said islands.

The Bureau of Forestry Development also interposed its objection, arguing that the land sought to be registered was covered by the Central Cordillera Forest Reserve under Proclamation No. 217 dated February 16, 1929. Moreover, by reason of its nature, it was not subject to alienation under the Constitutions of 1935 and 1973. 

The trial court * denied the application, holding that the applicants had failed to prove their claim of possession and ownership of the land sought to be registered. 11 The applicants appealed to the respondent court, * which reversed the trial court and recognized the claims of the applicant, but subject to the rights of Benguet and Atok respecting their mining claims.  In other words, the Court of Appeals affirmed the surface rights of the de la Rosas over the land while at the same time reserving the sub-surface rights of Benguet and Atok by virtue of their mining claims.

Both Benguet and Atok have appealed to this Court, invoking their superior right of ownership. The Republic has filed its own petition for review and reiterates its argument that neither the private respondents nor the two mining companies have any valid claim to the land because it is not alienable and registerable.

It is true that the subject property was considered forest land and included in the Central Cordillera Forest Reserve, but this did not impair the rights already vested in Benguet and Atok at that time. The Court of Appeals correctly declared that:

There is no question that the 9 lots applied for are within the June Bug mineral claims of Benguet and the "Fredia and Emma" mineral claims of Atok. The June Bug mineral claim of plaintiff Benguet was one of the 16 mining claims of James E. Kelly, American and mining locator. He filed his declaration of the location of the June Bug mineral and the same was recorded in the Mining Recorder's Office on October 14, 1909. All of the Kelly claims ha subsequently been acquired by Benguet Consolidated, Inc. Benguet's evidence is that it had

made improvements on the June Bug mineral claim consisting of mine tunnels prior to 1935. It had submitted the required affidavit of annual assessment. After World War II, Benguet introduced improvements on mineral claim June Bug, and also conducted geological mappings, geological sampling and trench side cuts. In 1948, Benguet redeclared the "June Bug" for taxation and had religiously paid the taxes.

The Emma and Fredia claims were two of the several claims of Harrison registered in 1931, and which Atok representatives acquired. Portions of Lots 1 to 5 and all of Lots 6 to 9 are within the Emma and Fredia mineral claims of Atok Big Wedge Mining Company.

The June Bug mineral claim of Benguet and the Fredia and Emma mineral claims of Atok having been perfected prior to the approval of the Constitution of the Philippines of 1935, they were removed from the public domain and had become private properties of Benguet and Atok.

It is not disputed that the location of the mining claim under consideration was perfected prior to November 15, 1935, when the Government of the Commonwealth was inaugurated; and according to the laws existing at that time, as construed and applied by this court in McDaniel v. Apacible and Cuisia (42 Phil. 749), a valid location of a mining claim segregated the area from the public domain. Said the court in that case: The moment the locator discovered a valuable mineral deposit on the lands located, and perfected his location in accordance with law, the power of the United States Government to deprive him of the exclusive right to the possession and enjoyment of the located claim was gone, the lands had become mineral lands and they were exempted from lands that could be granted to any other person. The reservations of public lands cannot be made so as to include prior mineral perfected locations; and, of course, if a valid mining location is made upon public lands afterwards included in a reservation, such inclusion or reservation does not affect the validity of the former location. By such location and perfection, the land located is segregated from the public domain even as against the Government. (Union Oil Co. v. Smith, 249 U.S. 337; Van Mess v. Roonet, 160 Cal. 131; 27 Cyc. 546).

"The legal effect of a valid location of a mining claim is not only to segregate the area from the public domain, but to grant to the locator the beneficial ownership of the claim and the right to a patent therefor upon compliance with the terms and conditions prescribed by law. Where there is a valid location of a mining claim, the area becomes segregated from the public domain and the property of the locator." (St. Louis Mining & Milling Co. v. Montana Mining Co., 171 U.S. 650; 655; 43 Law ed., 320, 322.) "When a location of a mining claim is perfected it has the effect of a grant by the United States of the right of present and exclusive possession, with the right to the exclusive enjoyment of all the surface ground as well as of all the minerals within the lines of the claim , except as limited by the extralateral right of adjoining locators; and this is the locator's right before as well as after the issuance of the patent. While a lode locator acquires a vested property right by virtue of his location made in compliance with the mining laws, the fee remains in the government until patent issues."(18 R.C.L. 1152) (Gold Creek Mining Corporation v. Hon. Eulogio Rodriguez, Sec. of

Page 58: Natural Resources Set2.Classification Cases

58NATURAL RESOURCES: CLASSIFICATIONS CASES

Agriculture and Commerce, and Quirico Abadilla, Director of the Bureau of Mines, 66 Phil. 259, 265-266)

It is of no importance whether Benguet and Atok had secured a patent for as held in the Gold Creek Mining Corp. Case, for all physical purposes of ownership, the owner is not required to secure a patent as long as he complies with the provisions of the mining laws; his possessory right, for all practical purposes of ownership, is as good as though secured by patent.

We agree likewise with the oppositors that having complied with all the requirements of the mining laws, the claims were removed from the public domain, and not even the government of the Philippines can take away this right from them. The reason is obvious. Having become the private properties of the oppositors, they cannot be deprived thereof without due process of law. 

Such rights were not affected either by the stricture in the Commonwealth Constitution against the alienation of all lands of the public domain except those agricultural in nature for this was made subject to existing rights. Thus, in its Article XIII, Section 1, it was categorically provided that:

SEC. 1. All agricultural, timber and mineral lands of the public domain, waters, minerals, coal, petroleum and other mineral oils, all forces of potential energy and other natural resources of the Philipppines belong to the State, and their disposition, exploitation, development, or utilization shall be limited to citizens of the Philippines or to corporations or associations at least 60% of the capital of which is owned by such citizens, subject to any existing right, grant, lease or concession at the time of the inauguration of the government established under this Constitution. Natural resources with the exception of public agricultural lands, shall not be alienated, and no license, concession, or lease for the exploitation, development or utilization of any of the natural resources shall be granted for a period exceeding 25 years, except as to water rights for irrigation, water supply, fisheries, or industrial uses other than the development of water power, in which case beneficial use may be the measure and the limit of the grant.

Implementing this provision, Act No. 4268, approved on November 8, 1935, declared:

Any provision of existing laws, executive order, proclamation to the contrary notwithstanding, all locations of mining claim made prior to February 8, 1935 within lands set apart as forest reserve under Sec. 1826 of the Revised Administrative Code which would be valid and subsisting location except to the existence of said reserve are hereby declared to be valid and subsisting locations as of the date of their respective locations.

The perfection of the mining claim converted the property to mineral land and under the laws then in force removed it from the public domain.  By such act, the locators acquired exclusive rights over the land, against even the government, without

need of any further act such as the purchase of the land or the obtention of a patent over it.  As the land had become the private property of the locators, they had the right to transfer the same, as they did, to Benguet and Atok.

It is true, as the Court of Appeals observed, that such private property was subject to the "vicissitudes of ownership," or even to forfeiture by non-user or abandonment or, as the private respondents aver, by acquisitive prescription. However, the method invoked by the de la Rosas is not available in the case at bar, for two reasons.

First, the trial court found that the evidence of open, continuous, adverse and exclusive possession submitted by the applicants was insufficient to support their claim of ownership. They themselves had acquired the land only in 1964 and applied for its registration in 1965, relying on the earlier alleged possession of their predecessors-in-interest.  The trial judge, who had the opportunity to consider the evidence first-hand and observe the demeanor of the witnesses and test their credibility was not convinced. We defer to his judgment in the absence of a showing that it was reached with grave abuse of discretion or without sufficient basis. 17

Second, even if it be assumed that the predecessors-in-interest of the de la Rosas had really been in possession of the subject property, their possession was not in the concept of owner of the mining claim but of the property as agricultural land, which it was not. The property was mineral land, and they were claiming it as agricultural land. They were not disputing the lights of the mining locators nor were they seeking to oust them as such and to replace them in the mining of the land. In fact, Balbalio testified that she was aware of the diggings being undertaken "down below" 18 but she did not mind, much less protest, the same although she claimed to be the owner of the said land.

The Court of Appeals justified this by saying there is "no conflict of interest" between the owners of the surface rights and the owners of the sub-surface rights.

This is rather doctrine, for it is a well-known principle that the owner of piece of land has rights not only to its surface but also to everything underneath and the airspace above it up to a reasonable height.  Under the aforesaid ruling, the land is classified as mineral underneath and agricultural on the surface, subject to separate claims of title. This is also difficult to understand, especially in its practical application.

Under the theory of the respondent court, the surface owner will be planting on the land while the mining locator will be boring tunnels underneath. The farmer cannot dig a well because he may interfere with the operations below and the miner cannot blast a tunnel lest he destroy the crops above. How deep can the farmer, and how high can the miner, go without encroaching on each other's rights? Where is the dividing line between the surface and the sub-surface rights?

Page 59: Natural Resources Set2.Classification Cases

59NATURAL RESOURCES: CLASSIFICATIONS CASES

The Court feels that the rights over the land are indivisible and that the land itself cannot be half agricultural and half mineral. The classification must be categorical; the land must be either completely mineral or completely agricultural. In the instant case, as already observed, the land which was originally classified as forest land ceased to be so and became mineral — and completely mineral — once the mining claims were perfected. As long as mining operations were being undertaken thereon, or underneath, it did not cease to be so and become agricultural, even if only partly so, because it was enclosed with a fence and was cultivated by those who were unlawfully occupying the surface.

What must have misled the respondent court is Commonwealth Act No. 137, providing as follows:

Sec. 3. All mineral lands of the public domain and minerals belong to the State, and their disposition, exploitation, development or utilization, shall be limited to citizens of the Philippines, or to corporations, or associations, at least 60% of the capital of which is owned by such citizens, subject to any existing right, grant, lease or concession at the time of the inauguration of government established under the Constitution.

SEC. 4. The ownership of, and the right to the use of land for agricultural, industrial, commercial, residential, or for any purpose other than mining does not include the ownership of, nor the right to extract or utilize, the minerals which may be found on or under the surface.

SEC. 5. The ownership of, and the right to extract and utilize, the minerals included within all areas for which public agricultural land patents are granted are excluded and excepted from all such patents.

SEC. 6. The ownership of, and the right to extract and utilize, the minerals included within all areas for which Torrens titles are granted are excluded and excepted from all such titles.

This is an application of the Regalian doctrine which, as its name implies, is intended for the benefit of the State, not of private persons. The rule simply reserves to the State all minerals that may be found in public and even private land devoted to "agricultural, industrial, commercial, residential or (for) any purpose other than mining." Thus, if a person is the owner of agricultural land in which minerals are discovered, his ownership of such land does not give him the right to extract or utilize the said minerals without the permission of the State to which such minerals belong.

The flaw in the reasoning of the respondent court is in supposing that the rights over the land could be used for both mining and non-mining purposes simultaneously. The correct interpretation is that once minerals are

discovered in the land, whatever the use to which it is being devoted at the time, such use may be discontinued by the State to enable it to extract the minerals therein in the exercise of its sovereign prerogative. The land is thus converted to mineral land and may not be used by any private party, including the registered owner thereof, for any other purpose that will impede the mining operations to be undertaken therein, For the loss sustained by such owner, he is of course entitled to just compensation under the Mining Laws or in appropriate expropriation proceedings. 

Our holding is that Benguet and Atok have exclusive rights to the property in question by virtue of their respective mining claims which they validly acquired before the Constitution of 1935 prohibited the alienation of all lands of the public domain except agricultural lands, subject to vested rights existing at the time of its adoption. The land was not and could not have been transferred to the private respondents by virtue of acquisitive prescription, nor could its use be shared simultaneously by them and the mining companies for agricultural and mineral purposes.

WHEREFORE, the decision of the respondent court dated April 30, 1976, is SET ASIDE and that of the trial court dated March 11, 1969, is REINSTATED, without any pronouncement as to costs. SO ORDERED.

G.R. No. L-69997 September 30, 1987UNGAY MALOBAGO MINES, INC. vs. HON. INTERMEDIATE APPELLATE COURT 13

 GUTIERREZ, JR., J.:

Before us is a petition which seeks to set aside the decision of the then Intermediate Appellate Court affirming the dismissal of the petitioner's action for annulment and cancellation of free patents granted to the private respondents on the ground that the petitioner has no personality to file an action for reversion, the lands involved being public In character.

13 Classifications

Page 60: Natural Resources Set2.Classification Cases

60NATURAL RESOURCES: CLASSIFICATIONS CASES

On July 20, 1962, the President of the Philippines granted the following mining patents on mineral claims located at Ungay Malobago, Rapu-Rapu Albay.

1. lode patent No. V-52 to John Canson, Jr., on mineral claim known as "Catanduandes;"2. lode patent No. V-48 to petitioner, on mineral claims known as "Junior;"3. lode patent No. V-53 to John Canson, Jr., on mineral claims known as "Oas;"4. lode patent No. V - 46 to petitioner on mineral claim known as "Ester;"5. lode patent No. V - 51 to Carlos Stilianopulos on mineral claim known as "Jovellar;"6. lode patent No. V - 49 to petitioner, in mineral claim known as "Manila;"7. lode patent No. V - 50 to Carlos Stilianopulos on mineral claim known as "Polangui;" and8. lode patent No. V - 47 to petitioner on mineral claim known as "Ligao;"(pp. 5-7, Decision Annex 1, Petition)

Way back on October 30, 1959, John Canson, Jr. and Carlos Stilianopulos assigned their rights to their mining claims in favor of the petitioner. The assignment of rights was recorded in the Office of the Mining Recorder of Albay on December 2, 1959.

The aforestated mining patents, after their issuance on July 20, 1962, were all recorded in the Office of the Mining Recorder of Albay on August 28, 1962 and transcribed on September 4, 1962 in the Registration Book of the Registry of Deeds of Albay. Consequently, the Register of Deeds of Albay issued the respective original certificates of titles pursuant to Section 122 of Act No. 496 in the names of John Canson, Jr., Carlos Stilianopulos, and the petitioner.

Subsequently, or from 1968 to 1974, the following free patents were granted by the respondent Director of Lands and the corresponding original certificates of titles were issued by the Register of Deeds of Albay:

1. Free Patent No. 458143 dated October 3, 1968 and corresponding Certificate of Title No. VH-12195 to appellee Felix Detecio;2. Free Patent No. 427824 dated November 21, 1968 and corresponding Certificate of Title No. VH-12256 to appellee Melencio Asuncion;3. Free Patent No. 433318 dated January 10, 1969 and corresponding Certificate of Title No. VH-12198 to appellee Jesus Asuncion;4. Free Patents No. 422847 dated November 11, 1968 and No. 421947 dated October 28, 1969 and corresponding Certificates of Title Nos. VH-12185 and 12186, respectively, to appellee Maria Bernal;5. Free Patent No. 408568 dated July 8, 1968 and corresponding Certificate of Title No. VH-11591 to appellee Gregorio Bolanos; and

6. Free Patent No. 0663 dated March 25, 1974 and the corresponding Certificate of Title No. VH-19333 to appellee Bienvenido Asuncion. (Rollo, pp. 200-201)

All of the above patents covered portions of the lots covered by the patents belonging to the petitioner.

The petitioner filed a complaint for annulment and cancellation of patents against the private respondents and prayed that all the free patent titles issued in their favor for properties over which original certificates of title had already been issued in its favor be declared null and void.

The Director of Lands, who was impleaded as a formal defendant, filed his answer alledging, among others, that the petitioner has no personality to institute the cancellation proceedings inasmuch as the government is the grantor and not the petitioner, and it should be the grantor who should institute the cancellation proceedings.

On January 25, 1980, the trial court rendered a decision dismissing the complaint. It ruled that since the disputed properties form part of disposable land of the public domain, the action for reversion should be instituted by the Solicitor General in the name of the Republic of the Philippines and that, therefore, the petitioner lacks personality to institute the annulment proceedings.

The petitioner appealed to the then Intermediate Appellate Court.

On April 5, 1984, the appellate court affirmed the decision of the trial court. It ruled that the titles issued to the petitioner cover mineral lands which belong to the public domain and that these cannot be the subject of private ownership. According to the Court, under Section 101 of the Public Land Law, only the Solicitor General or the officer acting in his stead has the authority to institute an action on behalf of the Republic for the cancellation of the respondents' titles and for reversion of their homesteads to the Government.

In this instant petition, the petitioner raises two issues: a) Whether or not the appellate court committed an error of law when it ruled that the lands in question belong to the public domain; and b) whether or not the appellate court erred in discussing the complaint on the ground that the petitioner had no personality to institute the same.

With regard to the first issue, the petitioner maintains that since its mining claims were perfected prior to November 15, 1935, the date when the 1935 Constitution took effect, the applicable law is the Philippine Bill of 1902 and that under

Page 61: Natural Resources Set2.Classification Cases

61NATURAL RESOURCES: CLASSIFICATIONS CASES

this Act, a valid location of a mining claim segregates the area from the public domain. (Gold Creek Mining Corporation v. Rodriguez, 66 Phil. 259).

The Solicitor-General, on the other hand, argues that the petitioner's mining patents covered by Torrens Titles were granted only in 1962 by the President of the Philippines, by authority of the Constitution of the Philippines. Under the then Constitution, except for public agricultural lands, natural resources which includes all mineral lands, shall not be alienated. (Art. XIII, Section 1, 1935 Constitution) Therefore, what the mining patents issued in 1962 conveyed to petitioner was only the ownership of, and the right to extract and utilize, the minerals within the area covered by the petitioner's Torrens Titles but not the ownership of the land where the minerals are found.

We rule for the private respondents.

The petitioner has been beguiling, less than candid, and inexplicably silent as to material dates in the presentation of its case. Nowhere in the records of this petition is there any mention of a date before November 15, 1935 as to when essential acts regarding its mining claims were executed. It is silent as to when the land was entered, measured, and plotted; when the legal posts and notices were put up; when the claim was registered with the mining recorder; whether or not the annual amount of labor or development, and other requirements under the Philippine Bill of 1902 were followed. These may have been complied with but not necessarily before 1935.

A mere mention in the Torrens title that the provisions of the Philippine Bill of 1902 were followed is not sufficient. The Philippine Bill provides the procedures for the perfection of mining claims but not the dates when such procedures were undertaken by any prospector or claimant. The same procedures would have to be followed even after the Jones Law of 1916 and the Constitution of 1935 were promulgated, but subject to the restrictions of the fundamental law. The petitioner has failed to state if and when new procedures, different from the 1902 procedures, were provided by law to give a little substance to its case. The petitioner is completely and strangely silent about these vital aspects of its petition.

Petitioner has not established by clear and convincing evidence that the locations of its mining claims were perfected prior to November 15,1935 when the Government of Commonwealth was inaugurated. In fact neither the original complaint nor the amended one alleged the perfection of petitioner's mining rights prior to November 15, 1935. All that petitioner offers as evidence of its claims were the original certificates of titles covering mining patents which embodied a uniform "WHEREAS" clause stating that the petitioner "has fully complied with all the conditions, requirements, and provisions of the Act of the United States of Congress of July 1, 1902, as amended, ..." In the absence of proof that the petitioner's claims

were perfected prior to the 1935 Constitution, the provision of the latter with regard to inalienable lands of the public domain will apply.

Article XIII, Section I of the 1935 Constitution provides:

All agricultural timber, and mineral lands of the public domain, waters, minerals, coal, petroleum, and other mineral oils, all forces of potential energy, and other natural resources of the Philippines belong to the State, and their disposition, exploitation, development, or utilization shall be limited to citizens of the Philippines, or to corporations or associations at least sixty per centum of the capital of which is owned by such citizens, subject to any existing right, grant, lease, or concession at the time of the inauguration of the Government established under this Constitution. Natural resources, with the exception of public agricultural land, shall not be alienated and no license, concession, or lease for the exploitation, development, or utilization of any of the natural resources shall be granted for a period exceeding twenty-five years, renewable for another twenty-five years, except as to water rights for irrigation, water supply, fisheries, or industrial uses other than the development of water power, in which cases beneficial use may be the measure and the at of the grant. (Emphasis supplied)

Therefore, applying the aforequoted provision to the case at bar, we conclude that the issuance of the lode patents on mineral claims by the President of the Philippines in 1962 in favor of the petitioner granted to it only the right to extract or utilize the minerals which may be found on or under the surface of the land. On the other hand, the issuance of the free patents by the respondent Director of Lands in 1979 in favor of the private respondents granted to them the ownership and the right to use the land for agricultural purposes but excluding the ownership of, and the right to extract or utilize, the minerals which may be found on or under the surface.

There is no basis in the records for the petitioner's stand that it acquired the right to the mineral lands prior to the effectivity of the 1935 Constitution, thus, making such acquisition outside its purview and scope.

Every application for a concession of public land has to be viewed in the light of its peculiar circumstances. (Director of Lands v. Funtilar 142 SCRA 57, 69).

In the case at bar, although the original certificates of titles of the petitioner were issued prior to the titles of the private respondents, the former cannot prevail over the latter for the provisions of the Constitution which governed at the time of their issuance prohibited the alienation of mineral lands of the public domain.

Page 62: Natural Resources Set2.Classification Cases

62NATURAL RESOURCES: CLASSIFICATIONS CASES

In the case of Republic v. Animas (56 SCRA 499), this Court ruled that a grantee does not become the owner of a land illegally included in the grant just because title has been issued in his favor:.

A patent is void at law if the officer who issued the patent had no authority to do so (Knight v. Land Ass. 142 U.S. 161, 12 Sup. Ct., 258, 35L ED. 974; emphasis supplied). If a person obtains a title under the Public Land Act which includes, by mistake or oversight, lands which cannot be registered under the Torrens System, or when the Director of Lands did not have jurisdiction over the same because it is a public forest, the grantee does not, by virtue of said certificate of title alone, become the owner of the land illegally included. (See Ledesma vs. Municipality of Iloilo, 49 Phil. 769)

Moreover, patents and land grants are construed favorably in favor of the Government, and most strongly against the grantee. Any doubt as to the intention or extent of the grant, or the intention of the Government, is to be resolved in its favor. (See Republic v. Court of Appeals, 73 SCRA 146, 156). Hence, as earlier stated, in the absence of proof that the petitioner acquired the right of ownership over the mineral lands prior to the 1935 Constitution, the titles issued in its favor must be construed as conveying only the right to extract and utilize the minerals thereon.

The appellate court did not likewise err in concluding that the petitioner has no personality to institute the action below for annulment and cancellation of patents. The mineral lands over which it has a right to extract minerals remained part of the inalienable lands of the public domain and thus, only the Solicitor General or the person acting in his stead can bring an action for reversion. (See Sumail v. Judge of the Court of First Instance of Cotabato, et al., 96 Phil. 946; and Heirs of Tanak Pangawaran Patiwayan v. Martinez, 142 SCRA 252).

WHEREFORE, the petition is hereby DISMISSED for lack of merit. The decision of the Intermediate Appellate Court is AFFIRMED. Costs against the petitioner. SO ORDERED.

G.R. No. 95608 January 21, 1997SPOUSES IGNACIO PALOMO vs. THE HONORABLE COURT OF APPEALS 14

ROMERO, J.:

The issue in the case at bar pertains to ownership of 15 parcels of land in Tiwi, Albay which form part of the "Tiwi Hot Spring National Park." The facts of the case are as follows.

14 Classifications

On June 13, 1913, then Governor General of the Philippine Islands, William Cameron Forbes issued Executive Order No. 40 which reserved for provincial park purposes some 440,530 square meters of land situated in Barrio Naga, Municipality of Tiwi, Province of Albay pursuant to the provisions of Act 648 of the Philippine Commission.

Subsequently, the then Court of First Instance of Albay, 15th Judicial District, United States of America, ordered the registration of 15 parcels of land covered by Executive Order No. 40 in the name of Diego Palomo on December 9, 1916;  December 28,  and January 17, 1917.  Diego Palomo donated these parcels of land consisting of 74,872 square meters which were allegedly covered by Original Certificates of Title Nos. 513, 169, 176 and 173  to his heirs, herein petitioners, Ignacio and Carmen Palomo two months before his death in April 1937. 

Claiming that the aforesaid original certificates of title were lost during the Japanese occupation, Ignacio Palomo filed a petition for reconstitution with the Court of First Instance of Albay on May 30, 1950. 7 The Register of Deeds of Albay issued Transfer Certificates of Title Nos. 3911, 3912, 3913 and 3914 sometime in October 1953.

On July 10, 1954 President Ramon Magsaysay issued Proclamation No. 47 converting the area embraced by Executive Order No. 40 into the "Tiwi Hot Spring National Park," under the control, management, protection and administration of the defunct Commission of Parks and Wildlife, now a division of the Bureau of Forest Development. The area was never released as alienable and disposable portion of the public domain and, therefore, is neither susceptible to disposition under the provisions of the Public Land Law (CA 141) nor registrable under the Land Registration Act (Act No. 496).

The Palomos, however, continued in possession of the property, paid real estate taxes thereon  and introduced improvements by planting rice, bananas, pandan and coconuts. On April 8, 1971, petitioner Carmen vda. de Buenaventura and spouses Ignacio Palomo and Trinidad Pascual mortgaged the parcels of land covered by TCT 3911, 3912, 3913 and 3914 to guarantee a loan of P200,000 from the Bank of the Philippine Islands.

In May 7, 1974 petitioner Carmen vda. de Buenaventura and spouses Ignacio Palomo and Trinidad Pascual filed Civil Case No. T-143 before the then Court of First Instance of Albay for Injunction with damages against private respondents Faustino J. Perfecto, Raffy Santillan, Boy Ariado, Lorenzo Brocales, Salvador Doe and other Does who are all employees of the Bureau of Forest Development who entered the land covered by TCT No. 3913 and/or TCT 3914 and cut down bamboos thereat, totally leveling no less than 4 groves worth not less than P2,000.00.

Page 63: Natural Resources Set2.Classification Cases

63NATURAL RESOURCES: CLASSIFICATIONS CASES

On October 11, 1974, the Republic of the Philippines filed Civil Case No. T-176 for annulment and cancellation of Certificates of Title involving the 15 parcels of land registered in the name of the petitioners and subject of Civil Case T-143. Impleaded with the petitioners as defendants were the Bank of the Philippine Islands, Legazpi Branch and the Register of Deeds of Albay.

The case against the Bank of Philippine Islands was dismissed because the loan of P200,000 with the Bank was already paid and the mortgage in its favor cancelled.

A joint trial of Civil Case T-143 and T-176 was conducted upon agreement of the parties and on July 31, 1986, the trial court rendered the following decision:

WHEREFORE, premises considered, judgment is hereby rendered:IN CIVIL CASE No. T-143, in favor of the defendants and against the plaintiffs, dismissing the complaint for injunction and damages, as it is hereby DISMISSED.Costs against the plaintiffs.In CIVIL CASE No. T-176, in favor of the plaintiffs and against the defendants:(1) Declaring null and void and no force and effect the Order dated September 14, 1953, as well as the Original Certificate of Titles Nos. 153, 10 169, 173 and 176 and Transfer Certificates of Titles Nos. 3911, T-3912, T-3913, and T-3914, all of the Register of Deeds of Albay and all transactions based on said titles.(2) Forfeiting in favor of the plaintiff Government any and all improvements on the lands in question that are found therein and introduced by the defendants;(3) Declaring Lot Nos. 1, 2, 3, 4, 5, 6, 7 8, 9,10, 11 and 12, Plan II-9299 and Lots 1, 21, 11 3 and 4 of Plan II-9205 as part of the Tiwi Hot Spring National Park;(4) and Finally, the Register of Deeds of Albay is hereby ordered to cancel the alleged Original Certificates of Titles Nos. 513, 169, 173 and 176, Transfer Certificates of Title Nos. T-3911, T-3912, T-3913 and T-3914.Costs against the defendants.So Ordered. 

The court a quo in ruling for the Republic found no sufficient proof that the Palomos have established property rights over the parcels of land in question before the Treaty of Paris which ended the Spanish-American War at the end of the century. The court further stated that assuming that the decrees of the Court of First Instance of Albay were really issued, the Palomos obtained no right at all over the properties because these were issued only when Executive Order No. 40 was already in force. At this point, we take note that although the Geodetic Engineer of the Bureau of Lands appointed as one of the Commissioners in the relocation survey of the properties stated in his reamended report that of the 3,384 square meters covered by Lot 2, Plan II-9205, only 1,976 square meters fall within the reservation area,  13 the RTC ordered TCT 3913 covering the entire Lot 21 (sic) Plan II-9205 cancelled.

The petitioners appealed to the Court of Appeals which affirmed in toto the findings of the lower Court; hence this petition raising the following issues:

1. The respondent Court of Appeals committed grave abuse of discretion in affirming in toto the decision of the lower court.2. The declaration of nullity of the original certificates of title and subsequent transfer certificates of titles of the petitioners over the properties in question is contrary to law and jurisprudence on the matter.3. The forfeiture of all improvements introduced by the petitioners in the premises in favor of the government is against our existing law and jurisprudence.

The issues raised essentially boil down to whether or not the alleged original certificate of titles issued pursuant to the order of the Court of First Instance in 1916-1917 and the subsequent TCTs issued in 1953 pursuant to the petition for reconstitution are valid.

Petitioners contend that the Treaty of Paris which ended the Spanish-American War at the end of the 19th century recognized the property rights of Spanish and Filipino citizens and the American government had no inherent power to confiscate properties of private citizens and declare them part of any kind of government reservation. They allege that their predecessors in interest have been in open, adverse and continuous possession of the subject lands for 20-50 years prior to their registration in 1916-1917. Hence, the reservation of the lands for provincial purposes in 1913 by then Governor-general Forbes was tantamount to deprivation of private property without due process of law.

In support of their claim, the petitioners presented copies of a number of decisions of the Court of First Instance of Albay, 15th Judicial District of the United States of America which state that the predecessors in interest of the petitioners' father Diego Palomo, were in continuous, open and adverse possession of the lands from 20 to 50 years at the time of their registration in 1916.

We are not convinced.

The Philippines passed to the Spanish Crown by discovery and conquest in the 16th century. Before the Treaty of Paris in April 11, 1899, our lands, whether agricultural, mineral or forest were under the exclusive patrimony and dominion of the Spanish Crown. Hence, private ownership of land could only be acquired through royal concessions which were documented in various forms, such as (1) Titulo Real or Royal Grant," (2) Concesion Especial or Special Grant, (3) Titulo de Compra or Title by Purchase and (4) Informacion Posesoria or Possessory Information title obtained under the Spanish Mortgage Law or under the Royal Decree of January 26, 1889.

Page 64: Natural Resources Set2.Classification Cases

64NATURAL RESOURCES: CLASSIFICATIONS CASES

Unfortunately, no proof was presented that the petitioners' predecessors in interest derived title from an old Spanish grant. Petitioners placed much reliance upon the declarations in Expediente No. 5, G.L.R.O. Record Decision No. 9820, dated January 17, 1917; Expediente No. 6, G.L.R.O. Record No. 9821, dated December 28, 1916; Expediente No. 7, G.L.R.O. Record No. 9822, dated December 9, 1916; Expediente No. 8, G.L.R.O. Record No. 9823, dated December 28, 1916 and Expediente No. 10, G.L.R.O. Record No. 9868, dated December 9, 1916 of the Court of First Instance of Albay, 15th Judicial District of the United States of America presided by Judge Isidro Paredes that their predecessors in interest were in open, adverse and continuous possession of the subject lands for 20-50 years.  The aforesaid "decisions" of the Court of First Instance, however, were not signed by the judge but were merely certified copies of notification to Diego Palomo bearing the signature of the clerk of court.

Moreover, despite claims by the petitioners that their predecessors in interest were in open, adverse and continuous possession of the lands for 20 to 50 years prior to their registration in 1916-1917, the lands were surveyed only in December 1913, the very same year they were acquired by Diego Palomo. Curiously , in February 1913 or 10 months before the lands were surveyed for Diego Palomo, the government had already surveyed the area in preparation for its reservation for provincial park purposes. If the petitioners' predecessors in interest were indeed in possession of the lands for a number of years prior to their registration in 1916-1917, they would have undoubtedly known about the inclusion of these properties in the reservation in 1913. It certainly is a trifle late at this point to argue that the government had no right to include these properties in the reservation when the question should have been raised 83 years ago.

As regards the petitioners' contention that inasmuch as they obtained the titles without government opposition, the government is now estopped from questioning the validity of the certificates of title which were granted. As correctly pointed out by the respondent Court of Appeals, the principle of estoppel, does not operate against the Government for the act of its agents. 

Assuming that the decrees of the Court of First Instance were really issued, the lands are still not capable of appropriation. The adverse possession which may be the basis of a grant of title in confirmation of imperfect title cases applies only to alienable lands of the public domain.

There is no question that the lands in the case at bar were not alienable lands of the public domain. As testified by the District Forester, records in the Bureau of Forestry show that the subject lands were never declared as alienable and disposable and subject to private alienation prior to 1913 up to the present. 16 Moreover, as part of the reservation for provincial park purposes, they form part of the forest zone.

It is elementary in the law governing natural resources that forest land cannot be owned by private persons. It is not registrable and possession thereof, no matter how lengthy, cannot convert it into private property,  unless such lands are reclassified and considered disposable and alienable.

Neither do the tax receipts which were presented in evidence prove ownership of the parcels of land inasmuch as the weight of authority is that tax declarations are not conclusive proof of ownership in land registration cases. 

Having disposed of the issue of ownership, we now come to the matter regarding the forfeiture of improvements introduced on the subject lands. It bears emphasis that Executive Order No. 40 was already in force at the time the lands in question were surveyed for Diego Palomo. Petitioners also apparently knew that the subject lands were covered under the reservation when they filed a petition for reconstitution of the lost original certificates of title inasmuch as the blueprint of Survey Work Order Number 21781 of Plan II-9299 approved by the Chief of the Land Registration Office Enrique Altavas in 1953 as a true and correct copy of the Original Plan No. II-9299 filed in the Bureau of Lands dated September 11, 1948  contains the following note, "in conflict with provincial reservation."  In any case, petitioners are presumed to know the law and the failure of the government to oppose the registration of the lands in question is no justification for the petitioners to plead good faith in introducing improvements on the lots.

Finally, since 1,976 square meters of the 3,384 square meters covered by TCT 3913 fall within the reservation, TCT 3913 should be annulled only with respect to the aforesaid area. Inasmuch as the bamboo groves leveled in TCT 3913 and subject of Civil Case T-143,  were within the perimeter of the national park,  no pronouncement as to damages is in order.

WHEREFORE, the decision of the Court of Appeals is hereby AFFIRMED with the modification that TCT 3913 be annulled with respect to the 1,976 square meter area falling within the reservation zone. SO ORDERED.